Of Particular Significance

Why the Higgs and Gravity are Unrelated

POSTED BY Matt Strassler

POSTED BY Matt Strassler

ON 10/15/2012

One of the questions I get most often from my readers is this:

  • Since gravity pulls on things proportional to their mass, and since the Higgs field is responsible for giving everything its mass, there obviously must be a deep connection between the Higgs and gravity… right?

It’s a very reasonable guess, but — it turns out to be completely wrong. The problem is that this statement combines a 17th century notion of gravity, long ago revised, with an overly simplified version of a late-20th century notion of where masses of various particles comes from.  I’ve finally produced the Higgs FAQ version 2.0, intended for non-experts with little background in the subject, and as part of that, I’ve answered this question.  But since the question is so common, I thought I’d also put the answer in a post of its own.

As preface, let me bring out my professorial training and correct the question above with a red pen:

  • Since gravity pulls on things proportional to their mass to a combination of their energy and momentum, and since the Higgs field is responsible of giving everything not everything, just the known elementary particles excepting the Higgs particle itself its mass, there obviously must be a deep connection between the Higgs and gravity… right? wrong.

Now let me explain these corrections one by one.

When you first learn about gravity in school, you learn Newton’s law: that the force of gravity between two objects, one of mass M1 and one of mass M2, has a strength proportional to the product M1 M2.

But that was true before Einstein. It turns out that Newton’s law needs to be revised: the Einsteinian statement of the law is (roughly) that for two objects that are slow-moving (i.e. their speed relative to one another is much less than c, the speed of light) and have energy E1 and E2, the gravitational force between them has a strength proportional to the product E1 E2.

How are these two statements, the Newtonian and the Einsteinian, consistent? They are consistent because Einstein and his followers established that for any ordinary object, the relation between its energy E, momentum p and mass M [sometimes called “rest mass”, but just called `mass’ by particle physicists] is

  • E2 = (p c)2 + (M c2)2

For a slow-moving object, p ≈ Mv (where v is the object’s velocity) and pc ≈ Mvc is much smaller than Mc2. And therefore

  • E2 ≈ (M c2)2    (i.e., E ≈ M c2 for slow objects)

Since planets, moons, and artificial satellites all move with velocities well below 0.1% of c relative to each other and to the sun, the gravitational forces between them are proportional to

  • E1 E2 ≈ M1 M2 c4

And since c is a constant, for such objects Einstein’s law of gravity and Newton’s law of gravity are completely consistent; the force law is proportional to the product of the energies and to the product of the masses, because the two are proportional to one another.

But for objects that have high speeds relative to one another, or for objects subject to extremely strong gravitational pulls (which will quickly develop high speeds if they don’t have them already), the Einsteinian law of gravity involves a complicated combination of momentum and energy, in which mass does not explicitly appear. This is why Einstein’s version of gravity even pulls on things like light, which is made from photons that have no mass at all. (And it is why gravitational waves — waves in space and time, massless just like light — can be formed by objects that are orbiting one another.) Simply put, the Einsteinian view of gravity (now reasonably well confirmed by experiment) differs significantly from the Newtonian view, and in particular, it is not mass but energy and momentum which are primary. And all objects, not matter what they are made from or how they are moving from your point of view, have energy — so everything in the universe exerts a gravitational effect on everything else. We say “gravity is a universal force” (here the term is not referring not to the universe but to the notion of universality — of complete generality.)

What about the Higgs field being the source for all mass in the universe? This statement, though you will often find it in the press or in glib articles written for the public, is false.

What is the true statement? Well, here is a list of the elementary particles that we know about so far. The massless ones are

  • photons, gluons, gravitons (the latter presumed to exist)

while the ones with mass are

  • W and Z particles
  • quarks: top, bottom, charm, strange, up, down
  • charged leptons: electrons, muons, taus
  • neutrinos: three types (at least two and probably all three with small masses)
  • the recently discovered new particle with a mass of 125 GeV/c2 (which I will assume for now is a Higgs particle of some type)

Now it is true that the W and Z particles, the quarks, the charged leptons and the neutrinos must get their mass from a Higgs field. It’s not possible for them to have masses any other way. But this is not true of the Higgs particle itself.

The mass of the Higgs particle does not entirely come from the Higgs field!

Where does its mass come from? Oh, that’s a long story that ends in a question rather than an answer. I will try to explain it someday. For now, suffice it to say that the mass of the Higgs particle does not have a single, simple, understood source, and the curious feature is that its mass is so small — this is one aspect of the enormous puzzle called the hierarchy problem.

But in any case, the Higgs field is not the universal giver of mass to elementary particles. The Higgs particle itself gets its mass, at least in part, from elsewhere. And it probably isn’t alone. It is very possible that dark matter is made from particles, and these too probably get at least part of their mass from another source. Dark matter is believed by most physicists and astronomers to be the majority of the matter in the universe; it is believed to provide the majority of the mass of the Milky Way Galaxy that we inhabit. The Higgs field likely provides little of that mass.

Other things get their masses from sources other than the Higgs particle. The majority of the mass of an atom is its nucleus, not its lightweight electrons on the outside. And nuclei are made from protons and neutrons — bags of imprisoned or “confined” quarks, antiquarks and gluons. These quarks, antiquarks and gluons go roaring around inside their little prison at very high speeds, and the masses of the proton and neutron are as much due to those energies, and to the energy that is needed to trap the quarks etc. inside the bag, as it is due to the masses of the quarks and antiquarks contained within the bag. So the proton’s and neutron’s masses do not come predominantly from the Higgs field. [Experts: There is a subtlety here, having to do with how the Higgs field affects the confinement scale; but even when it is accounted for, the statement remains essentially true.] So the mass of the earth, or the mass of the sun, would change, but not enormously, if there were no Higgs field… assuming they could hold together at all, which would not be true of the earth.

And black holes, which are some of the most massive objects in the universe, holding court at the centers of most galaxies, can in principle be made entirely from massless things. You can make a black hole entirely out of photons, in principle. In practise most black holes are made from ordinary matter, but ordinary matter’s mass is mostly from atomic nuclei, and as we just noted, that doesn’t come entirely from the Higgs field.

No matter how you view it, the Higgs field is not the universal giver of mass to things in the universe: not to ordinary atomic matter, not to dark matter, not to black holes. To most known fundamental particles, yes — and it is crucial in ensuring that atoms exist at all. But there would be just as much interesting gravitational physics going on in the universe if there were no Higgs field. There just wouldn’t be any atoms, or any people to study them.

Finally, you can ask more technically whether, in the equations that physicists study, there is any mathematical connection between gravity and the Higgs field. The answer is no. Gravitational fields have spin 2 and are described as part of space and time; they interact with all particles and fields in nature. The Higgs field, which has spin 0, only interacts directly with elementary particles and fields that also participate in the electromagnetic and weak nuclear forces.

So — the guess that the Higgs has something to do with gravity is natural for a non-expert, but I am afraid it is naive; it comes from misunderstanding both

  1. the Higgs field, which is not universal: it gives masses to most of the known elementary particles but not to the Higgs particle itself, and not to protons and neutrons, dark matter (most likely), or black holes,
  2. and Einstein’s gravity, which is universal and has to do with energy and momentum but not mass directly, and most certainly does pull on protons and neutrons, dark matter and black holes even though their masses don’t come entirely from the Higgs field.

It’s really true: despite appearances at first glance, the relation between gravity and the Higgs is just skin deep.

Share via:

Twitter
Facebook
LinkedIn
Reddit

656 Responses

  1. “Gravitational fields have spin 2 and are described as part of space and time; they interact with all particles and fields in nature. The Higgs field, which has spin 0, only interacts directly with elementary particles and fields that also participate in the electromagnetic and weak nuclear forces.”

    Preample: If there is unification, one fundamental “force”, then there must be a link between gravity and the Higgs (and all the other mass giving fields).

    So, what concept can be created to put both (all) in the same equation? … Can the Fourier transform be used to explain this relationship?

    Here is one idea; Looking at the overall picture of the universe, the “cosmic web” you see the structure as an array of filaments of various lengths and shapes and energies but a web all connected together (by gravity?). And this system of galaxies and everything else moving and interact like any other system we have analyzed with the use of the Fourier transform. And like any system there is a fundamental frequency and many (almost infinite) harmonics (at lower energies).

    So, can the gravitational field be the fundamental and the Higgs (and others) are the “lower” harmonics of the same “universal field”. And it is these lower harmonics, Higgs, which are giving mass (low velocities energies packets)?

  2. Mass is an illusion just as the appearance of solidity. The Higgs field imparts a “drag” on the particle giving the appearance of what we know as mass. Gravity is still not a “given” as a force, No one has generated a gravity wave yet in a laboratory yet.

    1. There is much more to inertial mass than drag, much more to QFT than topology, and much more to learn about the dimension known as time in the vacuum.

  3. Undeniably consider that which you said. Your favorite justification appeared to be on the internet the simplest factor to have in mind of.
    I say to you, I definitely get irked while folks consider issues that they just
    don’t understand about. You controlled to hit the nail upon the highest as well as defined out the entire thing without
    having side-effects , other people could take a signal.
    Will probably be again to get more. Thanks

  4. Frequently I would not find out report on websites, nonetheless would like to say that this write-up really pushed me for you to do therefore! A person’s writing style has been stunned myself. Cheers, really excellent write-up.

  5. Hi to every body, it’s my first pay a visit of this web site;
    this web site consists of awesome and really fine data in favor of readers.

  6. I believe that is among the mostt vital information for me.
    And i’m satisfied reading yolur article. However should observation on few common
    things, The site taste is great, the articles is in reality nice : D.
    Good task, cheers

  7. Hey guys, get a load of this:

    http://www.scientificamerican.com/article.cfm?id=splitting-time-from-space

    A new (quantum) law of gravity is being put through its paces at the Perimeter institute. This one is worth a look-see, seems to do everything that General Relativity does and much more. Looks like we might not need theories of Dark Matter and / or Dark Energy after all.

    Go back and check out the Penrose rings in the Cosmic Background Radiation again. Penrose is Stephen Hawking’s mentor, the way Minkowski was Einstein’s calculus teacher. It really pays to learn from a master, evidently.

    1. Thanks for the alert – yes sounds interesting. Personally as an amateur I am more attracted to the realization of a Universal constant time( space ), Which I think was Newton’s notion.

    2. Science fiction Warning:

      Supernovae do not seem to be as rare as once thought thanks to the advance in cosmological telescopic arrays & robotic searches. So the thinking is if any intelligent life forms were in the vicinity of a sun which was about to go caput would it not be in their interest to make themselves known in the hope to escape their inevitable fate?

      Supposing our sun was 200 years away from such an event how would we spend those last years? Take no action or pursue outrageous attempts to ensure that our species should exist in some way? And it they were substantially more tech advanced than us would they not send such a sun down through a worm hole to move it away? If such a thing could be possible. Or modify the supposed Higss field around it to switch the tired sun off? Supposing that such an intelligence had 1+ million years of technological insight vs our tiny 5000 years they should have solved the problems which we spend all our time on? Then if they could master the Higgs Field they would be using many locations in space today to further develop and expand their species to provide them with habitable choices ensuring the longevity of the species and not dependant upon their local solitary sun. In essence they would be planetary nomadic empire builders, curving space at will checking out potential places to pitch their tents. No evidence of that is apparent so I guess E’Rosen worm holes and Higgs Field physical fantasies, and we are the only intelligent species in a place called Universe! Which is most unlikely. Surely their work would be evident in some way i.e curving large areas of the universe or some other major manipulation of Space. We have not sighted any such occurances so presumably not possible or we have not looked for it?

      Or, they have mastered the Higgs Field and now enjoy a massless existence?

    3. At 9.44 his plate states that a photon or electron may be moved around by an underlying undetected wave. This caught my attention as it sort of smacks with my repetitive intuition that the fabric of space is constantly emerging and as I have stated in my 2012 essay, where it is necessary to constantly produce New Space for anything to move and not just an infinitesimal. Space used to be thought of as a static void. Now we are coming to terms with the fact that is constantly enlarging ( expanding ) and not constructed from nothing.

      His laser light polarization seems compelling but I do not fully understand the outcome and final statement on how it brings new light on the historic 2 slit experiment.

    1. Gravitons are not emitted by the earth, any more than electrically charged objects emit photons. To get an electrically charged object to emit a photon, you have to wiggle it back and forth. Same for gravitons.

      Constant gravitational or electrical fields are sometimes said to be due to “virtual gravitons” or “virtual photons”, but these are not gravitons or photons. http://profmattstrassler.com/articles-and-posts/particle-physics-basics/virtual-particles-what-are-they/ In fact, the energy of the gravitational field of the earth is *negative*.

      Zero-point energy is a completely different issue… it is an issue that is present even in empty space, and depends fundamentally on the fact that all fields are really quantum fields.

    2. How can you say Gravitons exist where as far as I understand not in the text books, but only exist in the minds of people who theorsize about such entities. It still may be the result of a completely different force, field or just a medium of information as yet undetected.

  8. I missed you guys too.

    Another really good reason that gravity theories like Le Sage’s or Nordstrom’s have nothing whatsoever to do with the recent Higgs discovery has to do with this:

    http://en.wikipedia.org/wiki/Bertrand%27s_theorem

    Mathematically, a scalar field (even an exotic one like Higgs) cannot possibly produce stable planetary orbits. Only inverse square law fields can do this on large scales. Knowing about the Higgs mechanism and Higgs field, it basically gets astrophysics nowhere to look for better answers than to rely on General Relativity.

    Sorry to have bothered anyone.

    Still, it would be nice if there were a bridge between the two domains of some sort, or at least a solid number that wasn’t ± 120 orders of magnitude for the vacuum energy.

    Keep up the great work, and hopefully we will see some more results to knock our socks back off after the LHC upgrade in 2015.

  9. The graviton does have zero spin since it is a scalar particle and a scalar field does not have a sense of direction or any spin. All particles above 100 Gev appear to be massless which is why sometimes they say the graviton has no mass so they are both right. It has mass but it is also energy.

    1. This is completely wrong. The graviton (like the gravitational waves in Einstein’s theory) are spin two. A theory with a spin-zero graviton would be Nordstrom’s theory of gravity, http://en.wikipedia.org/wiki/Nordstr%C3%B6m's_theory_of_gravitation, which is ruled out by data in favor of Einstein’s.

      “All particles above 100 GeV appear to be massless” — that’s wrong no matter how you interpret it. A top quark has a mass of 175 GeV/c^2; it isn’t massless, nor does it appear to be massless when it has kinetic energy above 100 GeV, nor can it ever have total energy smaller than 175 GeV. So I don’t know what you’re talking about, and my conclusion (on behalf of my readers) is that you don’t either.

      1. Hi Matt,

        I am happy that you finally stop all these non-sense postings on this long thread, from people who mis-understand physics and want to expose their own non-scientfic theories, misleading other peoples.

  10. James, speed does effect the mass because acceleration is felt independently of mass. According to the equivalence principle acceleration and inertial mass equal gravity so inertial force, acceleration equal gravity in a free fall. The kinetic energy (energy of motion) of the relativistic protons in the LCH have seven thousand times the rest mass of a proton at rest. E = MC squared Energy equals matter and mass which is expressed by many extra virtual quarks and gluons in the proton close to light speed in the LCH. The extra mass allows for higher more massive particles to be discovered; the virtual quarks and gluons collide with each other.

    1. You’re playing into a misconception about how the term “mass” is used in particle physics. Let me suggest you read http://profmattstrassler.com/articles-and-posts/particle-physics-basics/mass-energy-matter-etc/more-on-mass/the-two-definitions-of-mass-and-why-i-use-only-one/

      With the definition of mass you are using, all photons have a mass. With the definition that particle physicists use, all photons are massless. If you don’t keep careful track of which definition you’re using, you’ll get completely confused. [It’s really awful that history left us with this mess! Einstein used both definitions at different points in his career.]

      In particle physics, the equation E = mc^2 is taken to ONLY be true for particles at rest, not to be true always. The equivalence principle is more subtly defined than as merely the equivalence of inertial and gravitational mass.

      On this website I ALWAYS use the above definition, where mass = what some call “rest mass” [and not the one you are using, where mass = what some call “relativistic mass” and I call “Energy, divided by a constant”].

  11. Only 100 km from the LHC? Lucky guy! I’d have to drive over 350 miles to get to Fermilab.

    I think we are about the same age.

    I have read some of your website ‘gravity explained’.

    Thanks profusely for your work in thin films and flat panel display technology, Edward. The number of folks who know the first thing about birefringence, even among quite technical people, is so small, it really should embarrass them! If it suddenly all went away tomorrow, I wonder how long it would take them to regain such capability, since they seem to understand nothing about it. Like my two year old granddaughter putting small fingerprints all over my flat screen monitor for no good reason, they seem to have no idea how long it took to develop the technology that makes their fancy iPhones possible. Anyone can buy fancy stuff; making it from scratch takes more functional grey matter than 99.99% of them will ever have, or even meet in their short lives.

    It’s nice to see that someone else here appreciates that gravitational, inertial masses must be related, or else both Newton and GR carry about as much weight as anyone else’s cracked pot ideas about the subject, including my own. It is their predictions (mostly accurate ones) that set their theories apart from the untested ones. Thanks to Newton, we know the Earth will not spiral into the sun any time soon. Thanks to Einstein, we can have GPS.

    I don’t expect that knowing about the Higgs mechanism or Higgs field will lead to any breakthroughs in ‘inertial dampeners’ or star drive technology any time soon either, but who knows?

    My own ideas about gravity have changed throughout my life, but until seeing the Higgs Lagrangian, I never dreamed that someone would come so close to explaining the whole enchilada. I’m assuming you wrote your book before the Higgs discovery. Am I right?

    I think Einstein may have had the last word after all when he quipped that compound interest is the most powerful force in the universe. Compound interest indeed. I’m now trying to work out the math so that floating circular sea vessels can actually orbit each other in an appropriately energetic virtual waves. At the quantum scale it obviously works, so why not?

    1. Thanks for noting Gravity Explained – I am not impressed by it and it does nothing but embarrasses me for all kinds of reasons – I was surprised you found it as I cannot !. I became obsessed March 2012 after purchasing an antique clock which had a small bronze statue of Newton supporting it ! Then the idea came to me that perhaps gravity is nothing more than an exchange of information typ ‘0’s and ‘1’s. Which includes the paradoxical clock which speeds up when moving away from a larger gravitational field. It having its own gravity but we only think of the surface which it is moving away from.

      Yes the TFT work was an exciting period which lead to so many other research topics I was quite overwhelmed. My partner and I developed a laser metal organic CVD pyrolosis process which enabled the writing of transparent electrical conductors – an opposite approach to acid lithography. My current interest is of course Graphene films alike 1000’s of other people. When we can overcome the technical difficulties of producing this stuff in sheet form I think will cause the next evolution of man as it will open up so many technical applications. Including god forbid a TV you can roll up like a poster off the wall or foldable telephone etc. In medical applications could well be the most staggering as substrate for growing human cells. Then one thinks of the possibility of retinal cells and where that could take us!

      Matt got me onto the idea that radial momentum can cause gravity – which then got me thinking about the dynamics of a black hole and thinking it could simply be a gravity multiplier and void of any solid material in the conventional sense. The lead in to that is the functionality of the Magnetron and Klystron devices for generation of microwave fields. Yes I wrote the essay before the so called Higgs particle was detected – but in it I am technically cynical. When I published it I discovered Prof Erik Verlinke – whereas I used the word Production of space his word was far superior word and referred to it as Emergence of space which is far more appropriate. He published his first paper at the end of 2011. It encouraged me somewhat as I had found another person who could imagine Space as something which is constantly being Produced/Emerging. And in my thinking with the value C hence this is what limits its upper value as you cannot move into a space which does not yet exist. Also a medium in which information may be transmitted from one body to another irrespective to scale.

      I am following as best I can the Higgs work it is not a subject I fully understand as it is extremely detailed. And the interactions of all those infinitesimal particles for me is almost impossible to comprehend. You caught my interest with your “ Compound Interest” quotation I should like to know more what his meaning was. Also your work sounds very interesting relating to circular sea vessels and link with the quantum arena.

      1. You have absolutely nothing to be embarrassed about with respect to the gravity explained essay. Scott Adams (Dilbert fame) has submitted a similar idea in one of his books I read. Since no one has a better explanation of what the mysterious Dark Energy might be, production or emergence of space is as valid an idea as any other at this point. Now all we need is someone like Stephan Hawking to make a prediction about what virtual particles near such a process would do, and we’re in business.

        As for the 1’s and 0’s idea: nothing wrong with that either. In fact, Hawking has recently lost TWO bets. The latest was of course betting against the Higgs discovery, which cost him $100. The other bet he lost was an argument over whether black holes completely obliterate all data (1’s and 0’s) that was contained within it when it collapsed. Because in cosmological time black holes eventually evaporate, a small amount of information could theoretically be recovered, most likely in the case of an energy black hole with only a small amount of matter remaining inside of it.

        Fascinating stuff being done with grapheme (and Scotch tape!!) these days. Nobel Prize winning stuff, too. Thin films are not going away any time soon. The graphene movie the researchers eventually produced certainly knocked my socks off. Actual carbon atoms in a graphene lattice are exotic and eerie looking; like it was something no human was supposed to ever see. Yet there they were; several layers, tetravalent bonds and all.

        As for the circular ships on a sea of virtual particles, the idea that the sea is calmer between particles that have mass so as to draw them weakly together, and calmer still between collections of particles with a lot of mass is an idea that for me just refuses to go away. The Casimir effect might be bogus, but there is something going on with part of the explanation that has merit. There isn’t anything like the Higgs mechanism going on with anything immersed in air or water, of course, but a mathematical model should be possible.

        So, Edward, you see, there is really nothing for you to be embarrassed about. Einstein himself revised General Relativity so many times during the eight years it took for him to develop it, his colleagues were beginning to doubt he would ever settle on its final form.

        If you want to read something that is really embarrassing, try Chris Langan’s Cognitive Theoretic Model of the Universe, or John Doan’s two decade rant about Special Relativity’s Twin Paradox. Chris Langan has a measured IQ over 200, but the education of your basic imbecile. Plenty of cognitive ability, totally gone to waste there. You’re gravity ideas are genius compared to either of those guys.

        1. Once again thanks Daniel I get the impression you are a quality educator a rare form these days. Your supportive comments are very well received but a danger to me: 1. Resurrection of obsession on these subjects, 2. No one has interest in my rantings 3. Such congnition requires a solid math foundation in order for it to be taken with any kind of seriousness – here I fail completely. Having said that I do have a referee for the Xrchiv publication site but until I can present paper/s in the correct form it will not be considered by them for publication. Erick Verlinde succeeded with his as his was correctly presented and is a working Prof at Princeton.

          Being ex RN I noticed the similar phenomena in which your mind is focused. When two ships are on the same track with a short distance between them the sea state was a lot calmer and the navigation problem was overcoming the subsequent suction between them. However, I am sure that on the quantum level such phenomena is a mystery to be solved. AE & SIN have solved the predictive affects of G but what actually causes it and what is responsible for it’s transit is the big apple remaining. I understand that it is still believed to be a fundamental particle of some form or other. However once I thought that space was constantly emerging it may well not be necessary to have such a particle as the carrier is the emergence of space itself is causing its transmission. I realize this is vaguely a 19C outlook but if MM work had not come to a standstill maybe they could have invented a more appropriate experiment? Hence my delight with the NASA planned work. We can see a tree grow and measure it but how could we know if a tree is growing if it is invisible to everything? Other than the Horizon problem, Clock affects, Time dilation, Legnth shortening, Value of C, Dilution of matter in the universe, Gravity, Entangled particles – etc. ( Synchronicity of sun spots ?) Maybe these jointly are the clues.

          Regarding SH black hole thermal entropy how will we ever know that such a theory is correct? And if it is correct why can we not see the slightest glimmer of the lost heat which it is supposed to be losing? If you have the will to indulge me further I would like to present you with a diagram on same subject or it can be viewed via you tube, but alas having created it I can never go back and find it. The title is ‘Black Hole as a gravity Engine’. You having a generous spirit with a lot of specialist knowledge I would like you to take a first glance for amusement purposes only and receive your opinion. Also to receive advice from you how such a proposal could be written up into math form. Not included in my further essay ‘Gravity Explained’.

          I will research your book refs – thanks.

          1. That’s the ticket, Edward.

            Very few people understand that OCD (obsessive-compulsive ‘disorder’) is anything but debilitating for certain occupations. Engineering and science for sure, because for an engineer the risk of not being obsessive about your designs means you forget to consider important details, and the risk for a scientist is that if he or she may overlook small anomalous effects in the everyday drudgery of data collection that are often key to discovering new and important science.

            Even OCD can be focused to good effect. SIN and AE are both cases in point.

            Lee Smolin, whose science writings are misunderstood by a great many on this website and elsewhere) has written extensively on ideas about black hole physics and string theory.

            At least one beautiful thing which Smolin did not write about regarding string theory is the ‘amplitudihedron’. It does for quantum probability calculations what the slide rule did for engineering mathematics. Gone are Feynman’s tortured line integrals (literally) tracking around the all of the moons of Jupiter just to find out the probability that a photon in Young’s double slit experiment is displayed (or not) on a screen. And good riddance to that. Feynman diagrams are great, but that variety of math just sucked.

            The other thing Smolin writes extensively about, a sort of Origin of Species for the evolution of universes with black holes, gets a bad rap mainly because it is rejected out of hand by other astrophysicists and ‘Intelligent Design’ proponents alike. That’s too bad, because even though most hard core scientists (such as myself) are loathe to admit it, the aspiration of our finite minds to grasp just a glimpse of what an infinite mind might know is probably the most important motivation for doing science at all, as piecemeal and ‘trial and error’ prone as the process needs to be for us. Faith should never be thought of as an obstacle to doing good science, unless yours demands a dogmatic approach to the written word, which is one form of idolatry. Gregor Mendel was both a friar and the first true geneticist. Newton’s posthumously discovered writings about the Christian faith being misdirected into polytheism by a simple mistranslation of the Greek (‘holy trinity’) shows that he thought as deeply about his adopted faith as he did about physics, optics, or the form of calculus he invented. My point is, one should never estimate the value of science or a scientist based on his or her motivation for doing it.

            Smolin’s writings about black holes, in particular, have merit. An energy black hole is every bit as possible as a black hole created when a massive star collapses into one. This means all of the mass-energy of the black hole could (eventually) be concentrated at the event horizon. This means it might also be hollow, or contain a residual amount of matter (whole stars, galaxies, or whatever). A mostly energy black hole as viewed from the inside would have an awfully strong pull on matter approaching its event horizon from the inside, but like matter falling into black holes from the outside, the relativistic effects would make falling into the even horizon take quite literally forever. In other words, it would appear to be a perfectly normal infinite universe with increasing acceleration of anything and everything approaching its event horizon from the inside. An event horizon is quite capable of accreting everything and anything that falls into it. Even if the interior were made of a solid mass of neutronium to begin with, eventually the mass of the interior would thin out. I’m not saying (and neither does Smolin) that this is the case in our universe, mind you, but the moment of the Big Bang could just as easily be interpreted as something that happened interior to a black hole, just after it collapsed. Smolin seems to think that the nature of black holes we can observe in our universe is a clue to the creation of a whole hierarchy of such encapsulated universes.

  12. Sunspot activity is mostly random, and likely just correlated with concentrations of different heavier elements present in the corona at any given moment. I was not aware there might be anything like superluminal velocities (or even quantum entanglement?) associated with sunspots.

    There is a marvelously detailed supercomputer model of the inner workings of the sun (the core workings at least), which, like the search for the Higgs, has some interesting history.

    The search for the missing neutrinos is another drama in physics with which I am intimately familiar, and other accounts of it are readily available on the internet. Ray Davis’ detector, based on counting the number of argon atoms produced from solar neutrinos bombarding the chlorine atoms surrounding moelcules of perchlorethane, was only able to count about 1/3 of the expected neutrino flux. This was alarming to the physics community, because either the computer model was wrong, or our sun was a lot older than we thought.

    The SNO Sudbury Neutrino Observatory in Ontario, still the deepest such underground detector, resolved the issue some thirty years later using a large tank of heavy water and photomultiplier type detectors. These could count the number of neutrinos (as well as their general direction) by capturing images of the Cherenkov radiation photons they produced when they collided. The mystery was solved, and our computer model of the core of the sun vindicated. And so we finally know the age of our sun to a very high degree of accuracy, and it is about middle-aged.

    Both Jack Ulmann and I are expecting big things to be discovered with AMANDA (Antarctic Muon and Neutrino Detector Array) and Ice Cube in the next few years. Much better physics is happening there than anywhere else until 2015 at least.

    1. Sunspot activity is cyclical with well documented frequency. My suggestion is has this data ever been compared with our neighbouring suns such as Cephei,HD 12545 and Draconis etc? If the frequencies are synchronous or near synchronous would cause some scientific interest.

      1. Nikola Tesla wrote about recorded increased sunspot activity being correlated with wars on this planet. Carl Sagan commented on the futility of such activity on so small a speck of real estate.

        If they were correlated, it might just mean they were synchronized in an area of the galaxy subject to similar magnetic fields.

        1. Indeed but do we actually know if they are synchronous with or without martial conflict? A typical sun has a diameter of 1M miles and makes for a significant experimental detection face.

          1. That would be tough (determining synchronicity or coincidence of sunspots on different stars). Depending on which part of the cycle (and magnetic pole polarity), you might have better luck correlating hemispheric numbers of sunspots to start. With our own sun, the whole cycle is 22 years, give or take. With other stars, it depends on masses, composition (which is also a measure of age).

            The surface area of a sphere is: A = 4\pi r^2. A = area of sphere, r = radius of sphere, pi =, well, you know.

            You’d be amazed at how many esoteric bits of statistical data like this astronomers and astrophysicists have amassed over the last few decades. I once wondered, for example, whether there might be a ‘net’ spin to observable spiral galaxies, and whether their angular momenta favored any particular direction in space? Turns out, that study / survey has already been done, and the answer seems to be that it is statistically random and evenly distributed in every direction to a high degree of certainty. The angular momentum of the expansion of the universe at large does not seem to have a preferred direction or orientation, within the parameters of the survey.

            I kind of suspect what you are suggesting would already have been done, if we could actually view the sunspots in nearby stars. Fact is, we can’t, and it’s difficult enough even to determine whether or not there are large planets orbiting them. Kepler was doing that while looking for extrasolar Earth candidates, before its supply of liquid helium ran out. The result was that as many as 20% of the nearby systems it had time to survey have either Earths or super-Earths in the requisite Goldilocks zone to support carbon based life / liquid water at or near its surface. You may have recently read about this.

            How am I doing so far? I noticed, many folks who were previously answering your many questions were running out of steam over here.

            Glad to help out with answering your questions Edward, but if Matt objects to us getting side tracked too far from particle physics (and sunspots is stretching this a little) we may need to take this discussion to some venue other than his blog. I’m assuming he will object or moderate if he sees anything untoward in the discussions also.

            I’m here to learn what I can about the Higgs discovery before going back to answer more questions at a local magnet middle school in the near future. Are you a science educator or related interest?

            1. Thanks – yes I suppose i am rather repetitive and surprised Matt has not already deleted my history on this site. When I was researching sun spots I did turn up some data relating to other suns. A few examples showed vast sun spot activity which encompassed a high percentage of the sun area. I found various graphs showing quite clearly the frequencies etc but nothing relating the activity of one sun to another ( The correlation as you rightly describe). Putting aside the obvious possible causes if a link could be found would raise a lot of interest – why! Thank you for indulging me I appreciate your scientific curiosity has a different centre. We can detect the force of gravity but cannot identify the entity which causes its transmission – if indeed it is an entity in the first place. So if there is communication between objects – suns in this case could provide another insight? Kind regards

            2. My interest is centred around Nature which involves satisfaction at all levels of science. I am not an educator with a prime career in electronic engineering with specialization optoelectronics namely TFT’s and Thin films & material sciences study for low and room temp superconducting films etc. My claim to non fame being involved with the evolution of flat panel display tech which caused the redundancy of CRT tech in the late 80’s. Now living 100kms from the LHC – not that that has any bearing on anything whatsoever. Kind regards

  13. I once replied to a review of a book about the gravity wave experiments performed in the 1970’s at the University of Maryland College Park, where I received my batchelor’s in physics.

    Joe Weber was a Navy engineer turned physicist who set up an array of gravity wave detectors in College Park based on multi-ton solid aluminum cylinders suspended from an ‘acoustically isolated’ ceiling with piano wire. Each gravity wave sensor was surrounded by rings of piezoelectric detectors, which at the time were believed (by Weber) to be the fastest and most sensitive pressure detection instruments available.

    Right idea; wrong detectors. Joe Weber was not well respected in the physics community, and there were many false alarms about gravity waves which were likely the result of Earthquake activity (because the building’s foundation was acoustically coupled to its ceiling, right?), or perhaps trucks rolling up and down nearby US route 1.

    Jack Ulmann, assistant to Ray Davis who won the Nobel prize for his perchloroethane neutrino detector, told me that other ideas of Weber, such as using a single silicon crystal inside an NMR machine to detect neutrinos, was not an original idea, and never worked, just like his gravity wave experiments.

    To date, the most sensitive functional gravity wave detectors in existence are the Earth’s tides, and even they are difficult to analyze for that function, due mostly to weather and geography.

    The problem with using an interferometer in the way that NASA intends relies too heavily on gravity waves affecting an awful lot of empty space between the source and the mirrors (like the empty space between their ears, more like it!). There isn’t very much mass there for them to interact with, is there? Think hard about it. Planets are better gravity wave detectors, and you don’t have to build those. NASA would be better off correlating Marsquakes with Earthquakes or something along those lines. As I said, Joe Webers idea was better (not worth funding, mind you, but “better”) in most ways than an interferometer based experiment. What he lacked as an engineer, at least Joe usually made up for in imagination. A better man than Thomas Edison (or the folks ripping off physics for things like LIGO), he most assuredly was. His only failing in engineering was predicting the expected strength of the effect he was trying to measure. A difference of only a few Angstroms requires much better engineering to detect than, like LIGO, requesting that someone get you a flatter or lighter mirror to bounce your laser beam off of.

    1. Personally I would like to see the outcome of a study which measures the periodicy of sun spot on various suns looking for any evidence of synchronicity. If sun spots are synchronous would infer that some form of common thing is acting between them and possibly at a velocity >C.

  14. To wrap up: No one will ever see a “graviton”. Evidently, they don’t exist. Gravity “waves”, if there is such a thing, can only be detected by tidal interaction (and not interferometry).

    But the Higgs mechanism and quark interactions with the scalar Higgs field evidently provides the necessary energy exchanges for gravitational attraction to exist, albeit indirectly. Tiny virtual particle “waves” or energy in the quantum foam can cause particles with mass/energy in whatever state of motion to be attracted to other particles with mass/energy. Even the paths of photons will bend to this interaction, so it actually does what GR does, even if all of the math to show this unification isn’t quite “ready” yet. And why should it be? The Higgs was only just discovered. Even frame dragging and gravitomagnetics should work out just fine.

  15. L’Album du Marin, Charpentier, Nantes (1836, not 15th century) by P.C Causee is usually cited as support for the “Casimir Effect”, with an explanation that boundary conditions on the ocean by the vessels explain why there is infinite vacuum energy between two ‘perfectly smooth’ idealized metal plates, wormholes, Kip Thorne, and other science fiction. Well, the calming effect of the sea also has to do with the size of the waves and the sizes and masses of the ships (how much water is drawn), and not just parallel lines and boundary conditions. which is why it is a much better analog of Higgs than it is for the associated science fiction of the so-called Casimir effect. Sometimes, a distinction needs to be made between science and science fiction for the sake of advancing science.

    And stop polishing the mirrors on LIGO also, because it was never anything other than a repeat of the Michaelson-Morely experiment.

    1. MME was a very good attempt at bridging the difference between SIN & AE.

      SIN was dead and could not comment upon it- AE was not very interested and sunk it – until some 20 years later when he reinvented SIN. In his own way.

      1. MME = Michaelson Morely Experiment
        SIN = Sir Isaac Newton
        AE – Albert Einstein

        MME (Interferometry) survives as the best tool astrophysics has to detect gravitational lensing. Suggest reading Evalyn Gate’s ‘Einstein’s Telescope’ for details. I’m reading it now, in fact. A little dated since the Higgs discovery, however.

        I often worry that things like Kip Thorne’s ‘wormholes’ and the like are the vestiges of aether theory (because, why should either end of a wormhole have any particular frame of reference?). That would definitely be a giant step backward.

        1. I am awaiting news for the planned NASA experiment to occur approximately 10 years from now. They intend a sort of developed laser based experiment using 3 separated devices over vast distances. The technique is not that far removed from the MME concept. I am sure NASA scientists will gain some valuable information from the data collected. However, from my point of view I am not sure if will be able to detect if space is something which is constantly emerging at the speed of light. Although I hope it may cause people to think about it especially in conjunction with other mysterious phenomenom such as mass increase at values close to C, plus the many other so called paradoxes.

  16. Oh, and as to your question about the speed of light. That derives of the Higgs mechanism simply because any material object or energy going faster than that would be unable to interact with the Higgs field, which is exactly equivalent to saying that it cannot exist. Divide that one by zero and see what you get.

  17. Very good insight, Edward. There still exist 15th century accounts (in a French manual on seamanship) that when two vessels are in close proximity to each other, a turbulent and foamy sea will tend to push the ships together because the sea between them is calmer. Although the manual never quantifies the magnitude of this force, its origin makes perfect sense to anyone who isn’t too caught up in the mathematical machinations of their own manifestly finite minds.

    This effect is indeed a candidate for explaining why the Higgs imparting inertial mass also imparts gravitational mass, despite the public disassociations by members of the CERN team which have made this momentous discovery.

    1. Thanks for giving some validation to the concept. I can’t let it go as it seems to make sense. After all, why does light move at the speed of light. Is the space itself expanding at that rate, are there ripple the light rides on? And if so, what role does mass play on the expansion/ripples. It would seem that a drag might occur similar to the effect of the boats on turbulent water. Although, you and I don’t move at the speed of light, the ripples are sort of like air pressure. Reduce the pressure on one side and what happens, you move in that direction. The greater the reduction, the faster the movement.

      1. Such an effect also would obey the inverse square law and depend on the density (not just mass) of the gravitating bodies. In addition, interaction with the Higgs field explains very nicely where the energy for gravitational interaction actually comes from. Looks like a nautical analog of the Higgs mechanism to me.

        Mathematicians, like anyone else, easily become invested in a particular approach to a problem when they have worked with a system for a while.

        Edward Witten and Michio Kaku both point out that Newton and Einstein’s descriptions of gravity break down as r goes to zero. Einstein eliminated mass in General Relativity by co-opting math formerly used for fluid dynamics to describe space curvature, but General Relativity very much depends on the equivalence of inertial and gravitational mass as much as Newton does. General Relativity, for all of its flaws, still remains the most rigorously tested pieces of science ever conceived.

        Mathematical symbology is a wonderful tool if one is not overly dependent on it. Imagining that a finite mind such as ours can capture all of the possible interactions in the universe is as vain as it is almost certainly wrong. It was wrong for Newton and Einstein, and it’s just as wrong for theories of supersymmetry, string theory, or anything else. It’s like thinking you could theoretically derive the Schrodinger wave equation or Pauli’s exclusion principle without ever seeing how quantum mechanics actually works, or ever seeing a periodic table. Good luck with that.

        1. Thanks again Daniel. I understand I am a complete layman in this matter (with the exception of reading popular physics books such as the Hawking’s book, etc. which is of course the highest tip of the iceberg). I’m wondering, though, if anyone has explored the theories I expressed. That is, light could be thought of as standing still while space ripples through the universe at C, and gravity is a result of the drag that mass produces on the rippling. So, objects with mass that are otherwise equalized by this rippling would tend to move towards other masses (similar to how a wing moves up due to air pressure differentials, an object will move towards mass due to the differentials in the rippling that result from the drag). Thanks again for your thoughts.

          1. I like your idea of a stationary photon and everything else is moving passed it ! Sounds a bit off the wall but ajoins my own idea and promotion of a constantly emerging space which provides the transport mechanism by virtue of New Space ! I am a layman and curious about the Higgs. I was ignorant and somewhat lost with all this discussion about particles all the time. I watched an interesting presentation where the presenter simply used the analogy: One’s hand is totally massless until one moves through space ( or higgs field through it) whereupon it shall gain mass. Another bizarre account the mass of space ( converting energy into mass by equivalence e=mc^2 shall cause every cm^3 to weight 1 trillion tones !!! ?

  18. As a complete layman, I have a question. Do massless particles tend to travel at the speed of light? If so, is it possible that they do so because they are essentially standing still (i.e. the speed of light is non-motion and everything else is moving)? And could it be that this is the case because every point in the universe is expanding, or compressing, so to speak, at the speed of light in such a way so that if you were a proton, you would just be standing still riding a wave of expanding space? And possibly, gravity exists because objects with mass disrupt the flow of this spatial expansion so that when one is close to a large object, the uninhibited space in the directions other than where the mass is, exert more expansion on the particles that one is made of, thus pushing them towards the mass?

  19. General Relativity cannot be formulated AT ALL without reference to mass, specifically the principle of equivalence of inertial and gravitational mass. Furthermore, this relationship is tested against reality each and every time someone uses a GPS satellite to find their location on the surface of the Earth. The universe is filled with gravitational lenses, and every one of those bends light in accordance with this theory. This theory (General Relativity) has the highest standard of accuracy of any scientific theory in the history of the human race. It still has plenty of detractors, too, just like flat-earthers and moon landing deniers.

    You have deliberately mislead readers by stating that GR theory makes no explicit mention of mass within its mathematical form. It doesn’t need to; it’s assumed. Those other theories (Standard Model, M-Theory) are all very much works in progress, and the discovery of Higgs has not changed that (yet). But there is still hope.

  20. Just read this entire thread, it was a real eye opener. I must say that my physics knowledge base stems entirely from watching minutephysics on youtube, and now I am going back to my reality with a few less holes in my ideas of how the universe interacts, I still don’t understand it, I’m not going to, I do thank you Professor Strassler for enabling open sourced information such as this to the general public, regardless of whether we fully interpret it.

      1. By saying “exist” you enter the mine-field … eg. ‘exist’ before or after you measure?? The quantum potential of the particle – a unitary gizmo in phase space – clearly must “pre-exist” to get mapped into 3+1 (when it would get its mass). But I’d say that anything more detailed is likely theory-dependent, in that there is really no agreed upon connection between QM and GR.

  21. It seems to me to say gravity “pulls” on anything is inherently misleading as Einsteins idea was that gravity only “pulls” on space and not on objects at all, gravity warps space bending it such that objects appear to be “pulled” but that this is just an illusion. Dark matter and dark energy are meant to account for most of the mass and energy in the universe, Without dark matter to assist in curving space galaxies would fly apart from the momentum of their own observed spin and without dark energy to overcome the universes gravitational curvature of space the observed acceleration of the universes expansion would not be possible. The percentages of dark matter and dark energy required are quite simply the percentage of failure of the current gravitational model to conform to observation, a failure rate of around 90%. This is a massive refutation of the current model. No one knows why the current model fails to be verified by observation of the universe at large. In such a circumstance It strikes me as unscientific to make absolute claims about gravitation, With the 90% failure rate of the current model any and all ideas about gravitation are potentially on the table or on the chopping block and teachers are revealed to actually be 90% students who should be listening more and speaking less.

  22. Dear Mr. Strassler,
    I have heard it said many times that the Higgs field is like treacle in that it provides resistance to the motion of those particles with which it couples. But I was wondering why then doesn’t a particle’s mass increase in proportion with its speed, in the same way that drag on an object moving through a fluid is speed dependent? Surely a particle moving quickly would encounter more of the Higgs field per second and thus experience a greater resistance. Instead it seems that only acceleration is resisted! How is it possible that acceleration can be resisted and speed isn’t?
    Sincerely,
    James.

    1. Is there any such thing as higgs field?

      I thought the LHC had a problem with both field and particle and that is why they are investigating?

      So both are now part of official physics law?

      Now perhaps we can move onto other things.

  23. I’m not the one making up my own theories. My statements and criticisms are supportable by the principles of quantum physics which can be found online by anyone.

    1. You don’t know what you’re talking about. And I won’t have you confusing my readers. So put up, or shut up (or I’ll ban you, as I’ve banned other crackpots.)

        1. It’s also true that every now and then, someone with fixed but wrong ideas gets them set straight here. Essentially, I’m giving Mr. Hillend a chance to learn something from real physicists (and real data).

          But if he refuses to listen and continues to make obviously wrong statements, he’ll be banned, because this site is not the place for him.

          I have quite often allowed people who disagree with me to remain as commenters. There is a difference between (a) people who have a different point of view from me and (b) people who have a point of view that contradicts experiment.

          1. It’s your site – you must manage it and deal with some unusual people. I have visited my similar sites most of which operated by crankies. Let him play for the moment – and give him a chance to substantiate who he is or what his ideas are then say thanks and goodbye – don’t over react Professor. Just keep working.

      1. Mr. Johnson, Mr. Hilland has yet to prove he has any credentials at all. This is not an argument between equals, yet. Mr. Hilland hasn’t demonstrated he has any background in the subject. And he has made multiple statements that would get him laughed out of a particle physics discussion.

          1. The problem isn’t me; it’s other readers. I can’t have random people coming in, claiming to be experts, and making false statements about physics. How is a 17-year-old kid, or a retired engineer, to know who to believe? So I feel I have to fight this kind of fight; it’s to protect the integrity of this website, so that people will have confidence that they get the highest-quality information here.

            1. Of course not – it’s your idea – your site and you must manage it – whoever he is flies in and will fly out again. Some people are more interested in creating warfare in this environment – not very serious they get off with being controversial that’s what they specialize in.

  24. you said: ” the number of quarks is not well defined” and “I say “many, but it is not sensible to say precisely how many.” In some sense, the number is constantly changing (though the average is constant) since quarks and anti-quarks are annihilating into gluons and gluons are annihilating into quarks and antiquarks.” A quantum physicist would not say that. It’s wrong.

    [Editor’s note: since the author is lecturing me on quantum physics, and claiming to know more about than I do, I am not going to publish what he says until he proves he has the experience and background to do it. See below.]

    1. Ok, Mr. Hillend. You’re lecturing me now in my area of expertise. I have a Ph. D. in quantum field theory from Stanford; I was a full professor at Rutgers from 2007-2013, and have done stints at other universities as well; I am often invited to speak at international conferences on particle physics; my research has led to new particle searches at the Tevatron and Large Hadron Collider; my most-cited paper, on the links between quantum field theory and string theory, has over a thousand citations by my quantum physics colleagues, and I have several other papers in the 250-500 range. Now. You say “a quantum physicist would not say that”. Well, I’m a quantum physicist, and I Say That. Note that I’m speaking to the public on this blog, so I often make statements that are not precisely rigorous here; it is true that this statement is not precise, and I would not make precisely this statement to graduate students. However, your objections appear obviously wrong to me. But you obviously believe you know more about quantum physics than I do.

      Please state your credentials and let me check them. You can check mine on-line.

      http://www.physics.rutgers.edu/~strassler/
      http://indico.lal.in2p3.fr/conferenceDisplay.py?confId=1747 (Skip to the end)
      http://www.kmi.nagoya-u.ac.jp/workshop/sakata100/program/timetable.html
      http://www-d0.fnal.gov/Run2Physics/WWW/results/final/NP/N09B/N09B.pdf
      http://atlas-service-enews.web.cern.ch/atlas-service-enews/2009/features_09/features_hiddenvalley.php
      http://inspirehep.net/search?ln=en&ln=en&p=a+strassler&of=hcs&action_search=Search&sf=&so=d&rm=&rg=25&sc=0

      1. You also might want to study this plot : http://www.hep.phy.cam.ac.uk/~wjs/partons2008nlo.jpg

        which is taken from the webpage of a leading expert on the proton, Professor James Stirling at Cambridge University. http://www.hep.phy.cam.ac.uk/~wjs/ . I’ve talked to him many times at conferences.

        That plot shows the anti-quarks as well as the quarks inside the proton. Note that professor Stirling helped make this plot; in the MSTW group that made it, the “S” is Stirling.

        And lest you think Professor Stirling is a crackpot, look here https://twiki.cern.ch/twiki/bin/view/LHCPhysics/CrossSectionsCalc

        This is the official webpage used by the Large Hadron Collider experiments in their Higgs particle studies. Look for “MSTW” and you’ll see it appears 14 times on the page.

        So. I’m not the only quantum physicist who “says that”. In fact a whole community does; and this is the community that found the Higgs particle, and appears to know what it is doing.

  25. Quote by Matt Strassler: “Moreover, it is not true that protons have “hundreds” of quarks; what is correct is that the number is not well-defined. This is important, though confusing, because if you try to count the number of quarks you quickly get into paradoxes. I have not written the sophisticated article on this yet. For the general public, I do not attempt to say how many are in there, and when asked, I say “many, but it is not sensible to say precisely how many.” In some sense, the number is constantly changing (though the average is constant) since quarks and anti-quarks are annihilating into gluons and gluons are annihilating into quarks and antiquarks.” This quote is not in any way supported by quantum field theory.
    Baryon number and charge is always conserved. As a result, we do always know how many quarks there are three for baryons and two for hadrons. Also gluons do not turn into quarks and quarks don’t turn into gluons.

    1. You are wasting my time, and that of other readers. How in the world you can claim to know something about quantum field theory is a mystery to me.

      a) baryon number and charge do not change when quark + antiquark –> gluon + gluon, so your first objection is no objection at all. The quark and antiquark have opposite charge, so the total charge is zero before the interaction and zero afterward. The same is true for their baryon number; the quark has baryon number 1/3, the anti-quark has baryon number -1/3, so again the total baryon number is zero beforehand and zero afterward.

      b) “Also gluons do not turn into quarks and quarks don’t turn into gluons.” You do not know what you are talking about. How do you think top quark/antiquark pairs are produced at the Large Hadron Collider?

      https://atlas.web.cern.ch/Atlas/GROUPS/PHYSICS/PAPERS/TOPQ-2011-07/

  26. quote by Kudzu: “Firing a high energy neutrino at a proton is the same as firing a high energy proton at a neutrino.” No way. these are not equivalent maybe low energy proton. I agree with what you say about the energy level though because in neutrino detection in a large tank of water high, if a energy neutrino strikes a water nucleus and it creates a muon of electron.

    1. I am curious as to why you think they are not equivalent. As respects momentum the two cases differ; but otherwise what matters when two particles collide is the total energy available to both particles and not which particles posses it.

      This can be seen in a number of situations. When two objects collide what matters is their difference in speed so that being hit by a car at 100 miles an hour is the same as being fired into a stationary car at 100 miles an hour. In chemistry you ca often create an identical reaction by heating either of the reactants. (A hot iron wire will react with water in the same way as a cold wire with steam.)

  27. The fusion of one element into another only occurs at extremely high energies such as the fusion of hydrogen and helium in stars and in our Sun or a hydrogen bomb which releases large amounts of energy in the form of radiation alpha, beta, gamma, etc. Radioactive decay is based on half life’s where one unstable radioactive element decays into another. We use this for madiometric dating. For example Uranium 235 has a half life of 704 million years and Uranium 238 has a half life of 4.5 billion years, potassium 40 1. 25 billion years etc. These are radioactive elements

  28. quote by Matt Strassler: In this case one typically looks for processes in which the nuetrinos not only delfected but are actually converted to electrons by the weak nuclear force.” The weak nuclear force cannot convert a neutrino into an electron due to the conservation of mass. Weak gauge boson plus a neutrino does not equal the mass of an electron. How can the weak nuclear force convert a neutrino which is very light into and electron which is lighter than any gauge boson? Maybe an up or down quark but an electron?

    1. That’s and interesting point .. so when two atoms fuse they must release mass? In the form of energy is that what a fusion reaction is all about?

    2. That would depend on the energies involved; what matters is the total energy of *all* products vs *all* reactants. Firing a high energy neutrino at a proton is the same as firing a high energy proton at a neutrino. (Since due to relativity you can observe the reaction from two viewpoints, one where each of the particles is not moving.) Such a mechanism caps the energy of cosmic rays; high energy protons interact with microwave radiation to produce an excited proton state (that then decays.)

      In the case of a radioactive or energetic nucleus the conversion of a light particle into a heavier one can occur.

  29. Hi Matt, hope you are still answering this blog.

    You say; “And nuclei are made from protons and neutrons — bags of imprisoned or “confined” quarks, antiquarks and gluons. These quarks, antiquarks and gluons go roaring around inside their little prison at very high speeds, and the masses of the proton and neutron are as much due to those energies, and to the energy that is needed to trap the quarks etc. inside the bag, as it is due to the masses of the quarks and antiquarks contained within the bag.”

    My question is; how do physicists know that these collisions are happening in there when all that can be seen with an electron-microscope is the three dots that we know as quarks? Can gluons be seen?

    1. My ability to keep up with questions comes and goes…

      Your premise is wrong. We ***cannot*** see quarks with an electron microscope, and anyone who told you that this is how we know about the quarks in a proton was mistaken, or was speaking by analogy. The proton is much too small to investigate with an electron microscope.

      We know about the quarks (more than 3!!!) AND THE ANTIQUARKS in the proton by slamming electrons at high energy into a proton (typically hydrogen gas, which has lots of protons in it). The electrons bounce off the quarks and antiquarks. By studying how the electrons come flying out, we can infer something about how likely an electron is to bump into an up quark, up anti-quark, down quark or down anti-quark. To complete the suite of experiments, we also scatter neutrinos off of protons (no mean feat) and electrons off of neutrons (also no mean feat).

      How about the gluons? We can infer that there are gluons inside the proton in a more complicated way. This combines a theory with equations for how quarks, antiquarks, and gluons interact, computer simulations of those equations, and a subtle understanding (through those equations) of the effects of gluons annihilating into quarks and anti-quarks, and vice versa.

      But we can also check that we are correct about the gluons using the Large Hadron Collider and other sorts of experiments. For example, the rate to produce top quark/anti-quark pairs at the LHC depends crucially on the gluons inside the proton. That we get it right confirms we know what we’re doing. See http://profmattstrassler.com/articles-and-posts/largehadroncolliderfaq/whats-a-proton-anyway/checking-whats-inside-a-proton/

        1. An excellent question, and I realize I may not have answered this anywhere on the site.

          “Bounce off of” is really incorrect, and I should have been more careful in my wording. “Are deflected by” is more accurate. Whenever two electrically charged objects approach each other — and electrons, quarks and anti-quarks are all electrically charged — there will be an electric force between them, and this will deflect them. So we send electrons in a beam toward a target full of hydrogen gas, and we look at how the electrons are deflected. This is the same method used by Rutherford to discover that the nucleus is very small compared to an atom.

          Neutrinos will also scatter off of quarks and anti-quarks because of the weak nuclear force between them. This scattering is very rare, but it can be measured. In this case one typically looks for processes in which the neutrinos not only deflected but are actually converted to electrons by the weak nuclear force.

          1. Got it thanks – and obvious had I recalled that sub quantum have charge characteristics too. But nature manages to squeeze atoms/protons together on a daily basis!

            So does the composition (total sub quantum charge ) of up & down quarks found within an atom determine the number of electrons subtending to a parent atom in some form of natural equilibrium law? And in the case of heavy atoms does the proton still only have the 3 quarks? What is the relationship between atom size and quark number?

            1. Quarks bind into hadrons with integer charges (0 or 1 usually.) A proton will bind an electron to an atom because of the electromagnetic force.

              If you read the article on what protons are you will find that protons (and neutrons) have far more than 3 quarks; they have hundreds. I am not sure what you mean by ‘heavy’ atoms; the proton in hydrogen is ‘heavier’ than a proton in uranium. If you ignore the effects of the nuclear force and structure (Which binds protons and neutrons together and varies from nucleus to nucleus.) then the protons in all atoms are identical, quark-wise.

              1. Now that is surprising! Ref Hyd vs Ur protons, I would have to learn more to understand that. I thought protons had 1xup and 2xdown quarks? So what determines the ratio of Q’s found in various elemental atoms?

                1. Kudzu is confusing you on this one. All protons are identical — period. Always.

                  Kudzu, if that weren’t true, the Pauli principle would not apply in nuclei.

                  Protons contain two up quarks, one down quark, many gluons, and many pairs of quarks and antiquarks (up, down, and strange, mostly). Anyone who told you it was just two up quarks and a down quark was white-lying. See http://profmattstrassler.com/articles-and-posts/particle-physics-basics/the-structure-of-matter/protons-and-neutrons/ and http://profmattstrassler.com/articles-and-posts/largehadroncolliderfaq/whats-a-proton-anyway/checking-whats-inside-a-proton/ .

                  What Kudzu was confusingly referring to is that the mass of an atomic nucleus is slightly less than the sum of the masses of what it contains. In fact that’s true of many objects: see http://profmattstrassler.com/articles-and-posts/particle-physics-basics/mass-energy-matter-etc/the-energy-that-holds-things-together/ and related articles. But this, I believe, has nothing to do with your question.

                  When you’ve read about protons and neutrons you can work your way back up (either order will work) to nuclei and atoms.

                  http://profmattstrassler.com/articles-and-posts/particle-physics-basics/the-structure-of-matter/the-nuclei-of-atoms-at-the-heart-of-matter/
                  http://profmattstrassler.com/articles-and-posts/particle-physics-basics/the-structure-of-matter/atoms-building-blocks-of-molecules/

                2. “Protons contain two up quarks, one down quark, many gluons, and many pairs of quarks and antiquarks (up, down, and strange, mostly)”

                  So if all protons are exactly the same how does that engineer for the lightest ones vs the heaviest ones – they still have the same internal component?

                3. All protons have exactly the same mass. There are no lighter and heavier ones. They are identical in all respects. The same is true for all electrons. And for all neutrons. And for all photons. All particles of the same type are precisely identical. The Pauli exclusion principle that determines chemistry, and the Bose-Einstein principle that allows for lasers and for Bose-Einstein condensates, depend upon this fact.

                  See for instance: http://profmattstrassler.com/articles-and-posts/particle-physics-basics/the-structure-of-matter/atoms-building-blocks-of-molecules/atoms-of-an-isotope-are-identical-literally/

                4. From one point of view it is astonishing; but in quantum field theory, the equations that we use to describe the particles of nature, this type of identity is automatic. This is one of the reasons quantum field theory is the cornerstone of particle physics, and why today particle physicists view fields, not particles, as the primary ingredients of nature, with particles being just little ripples (“quanta”) in the fields of nature.

                5. Yes that is very interesting. I suppose it much easier to make sense of it all as fields in the imagination, however if it has a construction we cannot ignore it. Astonishing is a very appropriate word to use. Do you know anything about the NASA plans to launch the laser based detectors – sort of copy of Michelson Morley experiment on the grand scale I think planned in the next 10 years?

                6. Makes me happy to hear you use the Nature word – with so much mathematics and theories and laws one sort of becomes isolated from what it is all about.

                7. Professor if you can tolerate a bit of emotional clap trap for a moment I cannot help but feel that for such organization to occur at the tiny scale which is energy related then there must be some background dynamic which we maybe we missing completley. My easiest explanation is a sand box. Nature does what it does. If we have a solid substrate and couple that with a source of vibrational energy ( as you well know ) the particulates of sand will organize themselves into regular patterns which vary according to the frequency of same. This is not the magic of sand it is a relationship with the surface, where they are and supported,and the energy from the system couple. If nature can do that in such a simple visible experiment why cannot it do the same at the tiny scale?

                8. I don’t know what nature is doing at the tiniest scales. I do know the mathematics of quantum field theory, applied to the particles of the Standard Model, gives predictions that agree with all known experiments. That doesn’t tell me the math of quantum field theory is fundamentally right, but it tells me that it gives me a good way to think about the world. And that’s all I know. Pretty good, if you ask me.

                  As for your specific question — you have to turn feelings like the one you have into equations, before you can do science. Then you have to calculate the implications of those equations, and see if they agree with nature. There is no short-cut.

                  You may want to watch the video of my recent public talk, http://profmattstrassler.com/2013/07/03/my-public-talk-on-the-higgs-now-online/ , if you want to know a bit more about what quantum field theory implies. What I describe in this talk is not “truth”; it’s a way of thinking about nature that agrees with all known experiments. Maybe it’s true, maybe it’s incomplete, but certainly it’s useful and powerful.

                9. I will with interest. As for the maths I think that is a wall too high to climb and not sure if one/me could put such a painting into a set of numbers and for it to retain its beauty by its conversion? A monkey jumps from one tree to the next only to get at the nuts before the next monkey ( by sense of time ). It is not aware it exists in 3D but manages to make very good judgments about it’s space and the amount of quantum leap it has to deploy not to end up on the jungle floor. Before I am corrected I now peanuts are not available in trees. Thanks again but we rely on you guys to provide guidance, like a dentist to pull a tooth or a med doctor to live longer.

                  A ‘contradiction theory’ a wild card like Galileo etc someone who has great insight or a better imagination than the ordinary man to give us incisive vision would be more than good? Apologetically I leave out ‘AA’ because although his numbers may be sound they do not describe how things work only what happens when they do, about to or will do. We may well live in a universe where some things simply cannot be put into creditable set of numbers to make sense of it. Other than to exchange ideas logically through the medium of numerical a language.

                  My old friend was ex Oxford (Old School ) and he would never be drawn into my inane rhetoric – he would say only deal with things which are real which I suppose is indisputable numerical & falsifiable physics. He was correct of course but the Vatican was wrong despite their religious dictative dogma. When Newton said “ What is there in Space where matter does not exist” is something which we still seek – a pity to drop his insight for the past 100 years, and follow a course of numbers which doesn’t answer the question.

                10. I think a bigger problem than not being able to ‘put things into numbers’ is our limited ability to understand numbers. It is insanely easy to reduce a four dimensional cube into numbers and equations, but it is an object that the human mind can never grasp. Often I wonder if we’d be able to understand the ‘theory of everything’ even if we discovered it.

                11. If sound energy can organize sand particulates into regular patterns, why cannot the universe do the same with energy, Or Quarks?

                12. I didn’t say it couldn’t. But lots and lots and lots of things are possible in principle. I can make a million speculations, but that won’t move science forward an inch.

                  The problem is that you’re only giving me words. I can’t test words. I have to test predictions. That’s what science is about. We have to take your words and make a very precise version of them, make precise equations based on them, and use them to make precise, concrete predictions based on your words, ones that we can check or falsify it. Until then, it’s just cocktail party conversation, not science.

                13. Naturally the way to learn how is to study how it’s been done in the past. In this case, one would want to study the math behind the sand example, and see if you can make something analogous for particles that’s consistent with what we already know about nature. This isn’t easy; most speculations are dead on arrival once one starts writing equations down… they violate some well-known experiment, or have internal inconsistencies.

                14. Yes and yes agreed. I am sure without knowing that the dynamics prevailing in a sand box/vibration exp is pretty much over exhausted with historic content conclusions. But on face value there is a cause and action. Speculatively how one jumps from the sand scenario to the universe having a similar resonant affect on sub quantum materials? Then as you comment dead before birth.

                15. The mathematics of sand in a box is still throwing up incredible things and novel surprises. For a good example of this see Sixty Symbols video here: http://www.youtube.com/watch?v=2uWps9LczH8 In physics you never know when one thing will be connected with another or where the math might lead.

                16. Thanks for the link. Yes very good point a monkey is attracted to eat bananas but does not know how they grow or why they grow. Even if you gave him a book to explain it or put him in front of a pc monitor with easy to follow images.

                17. Watched the sand box video with great interest – thankyou. I am not sure why they fixed on 60Hz for the test – one normally executes a range of frequencies but very happy for them that they have an argument for surface tension in this environment – including cows ! I hope they find an application. My thinking is that this idea is not entirely dissimilar to the external affect of the cosmos on the tiny scale. Doing something similar that is why the particles are so good at organizing themselves not by magic but by a background dynamic. Which I prefer to think of as the Primary dimension.

                18. Thanks for your responses I appreciate it. I may well look into the dynamics of the sand box experiment to see what I can dig up – now out of curiosity. It does not create particles but it creates an order of particles or at least one which can think of as order?

              2. Kudzu — Your statement is wrong; even if you account for the effects of nuclear force and structure, all protons are identical. If you were correct, the Pauli exclusion principle would not apply in nuclei.

                Moreover, it is not true that protons have “hundreds” of quarks; what is correct is that the number is not well-defined. This is important, though confusing, because if you try to count the number of quarks you quickly get into paradoxes. I have not written the sophisticated article on this yet. For the general public, I do not attempt to say how many are in there, and when asked, I say “many, but it is not sensible to say precisely how many.” In some sense, the number is constantly changing (though the average is constant) since quarks and anti-quarks are annihilating into gluons and gluons are annihilating into quarks and antiquarks.

          2. I am curious, does not the exclusion principle ensure that protons in an atomic nucleus are distinguishable, being in different quantum states? If they were all truly identical wouldn’t they all be in the same state?

            1. Absolutely not. You’re confusing what things are doing with what they are.

              The issue is: if I switched two of the protons, could you tell the difference?

              Take a Lithium atom; it has three electrons, two in the 1S state, one with spin up and one with spin down, and a second in the 2S state. If the electrons were distinguishable, then if I took the one in the 2S state and switched it with one of the ones in the 1S state, I’d get a different atom. But I don’t.

              Similarly, if you switch two of the protons in a nucleus, you get exactly the same nucleus you started with.

          3. Aaah, then that is why my confusion lies. In future I shall have to word tihngs more precisely. Many thanks.

      1. Thanks. Yes, I’ve read the suggested article and obviously have to do some updating of my text-books. I am amazed about the number of quarks in a proton and that gluons can turn into quarks. My greatest obstacle is being skeptical about theories in general and the person who told me about electron microscope was not a physicist but my professor of chemistry. I’ll do some more reading before I pop another question that is already festering in my head. Deeply grateful for you time and effort. Thanks again.

  30. excuse my dyslexia here, The graviton is 126 Gev. It is energy not mass? The photon has no mass but there still can be photons with very high energy. For example all waves in the electromagnetic spectrum are conveyed by quanta or particles called photons. We can have high energy gamma ray photons, x-ray photons, ultra-violet photons etc. Consequently the graviton can have zero mass just like the photon but it’s quanta still must be expressed in energy as electron volts. When we collide particles with high enough energy we make a particle in the higgs field; the decay of particles such as quarks and gluons which decay into gravitons which can decay into photons, W and Z bozons, bottom quarks which immediately decay into hadrons such as pions.

  31. According to Wikipedia the graviton has a spin of 2 and zero mass. http://en.wikipedia.org/wiki/Graviton I have to agree with this since the only particle of zero mass can be exchanged long range, yet the graviton has a mass of 216 GeV. The reason for this discrepancy is the quantized general relativity is not renormalizable unlike quantum electrodynamics. The gravity is exchanged through virtual particles which have no mass. It is only their quanta which have mass.

  32. Quantum field theory is a gauge theory; gauge means that we have vectors or motion. General relativity is also a classical gauge theory. They all measure through 4 dimensions. 3 of space and one of time.

  33. 1. The graviton has no spin which allows it to act differently in the Langrangian. 2. The graviton does move from one point to another. 3. The field does not emanate from only one point to another. It produces virtual particles at every point so it is spontaneous at every point. Only a particle with detectable energy moves from one point to another. The field is only indirectly detectable. When you make a graviton in LCH you also make a wave in the higgs field. The higgs field is coupled to the other fields e. I. electromagnetic field, strong and weak fields or other forces through symmetry breaking.
    My point is there higgs fields is always there; it represents the potential energy of the zero point energy or vacuum energy which is not really zero but always has a certain energy conveyed by virtual particles. All particles with mass are dependent on the higgs field to give them mass.
    It seems to me the Higgs particle does have sense of direction but the higgs field does not since it is a scalar field. It has not spin but that does not mean it does not have a sense of direction? It must have a sense of direction otherwise there can’t be any gravity waves. A particle is simply the localization of the wave function to a small area, a wave packet, or one quanta of energy, The wave function is low so we see only one crest but not a train of crests with troughs when the wave function is high; With a high wave function and the velocity is known but the position becomes uncertain and vice versa. A low wave function and the velocity becomes very uncertain but the position becomes certain.

  34. On the question of the nature of a graviton, one question that I’ve had for a long time is, How formalism- or theory-dependent is the thing? For example, over the years I’ve seen both spin 2 and spin 3/2 attached to gravitons.

    My main point though is that this whole discussion of a field-borne ‘particle’ tacitly assumes that the field emanates from some point A and the graviton ‘carries’ the effect to point B. This is the way it’s usually viewed, eg. “It would take 8 minutes for us to discover that the sun has disappeared”.

    However, my (computer scientist’s) analysis of causality (using geometric algebra over Zed3 = {0,1,-1}) shows that, unlike weak, strong, and electromagnetic processes, gravitational processes – being *mutual* or “two-sided” – *cannot* possess this kind of ‘from-here-to-there’ particle. [It looks this way, though, because we’re *inside* 3+1 space-time, and “from A to B” is how everything seems to work here.]

    The culprit here is that in the algebra, the possibility of gravity first emerges with 4-vectors like abcd, which commutes with everything (and squares to +1), whereas ab and abc both square to -1, thus forming the basis for all sorts of +- polarities (spin up/down, charge plus/minus, etc.).

    There *is* a nilpotent something connecting A and B, but in this view it’s the (dissipative) space itself, which commutes/communes with itself, so to speak. I was as surprised by this result as anyone, but it makes very good sense, and the mathematics speaks very clearly here. Unfortunately, I don’t know how to calculate spin – the usual “square it and see if you get -1” method seems to fail.

    Anyway, I just wanted to interject the fact of the above-mentioned tacit assumption.

    1. A question on your question. In a hydrogen atom is not the electromagnetic interaction between the proton and electron also mutual or two sided? They should both exert the same force on each other yes? How does this differ from gravitational force?

    2. Interesting thinking.. How does your notion fit in if the very space which the particles considered are located is renewing itself constantly ( emerging )? Where this emergence of primary space is then responsible and performing a carrier task of information across gaps? A to B etc.

  35. The higgs field does not have a sense of direction like the idea that space appears the same in all directions. Reference: The article A unified physics by 2050, The Edge of Physics, Scientific American special addition, 2003. The electro-magnetic field does have a sense of direction. The graviton has a sense of direction only when we add matter or energy into the field like the Earth, a Star, because you have collided two particles in LCH adding energy to the Higgs fields which produce a graviton.. Gravity waves come from the star and have a sense of direction. We know this because the travel time the light from pulsars take to reach our Earth have been off according to classical physics but predicted by Einsteins general relativity which predict that gravity waves are ripples in space-time which have changed the travel time the light from the pulsar takes to reach our Earth.

    1. Einstein’s interpretation of gravity is nothing more than a mathematical and philosophical model of its affect – so how can you use this a yardstick to construct a new idea?

      1. In the end all theories are ‘mathematical and philosophical models’ We base new ideas on theories all the time.

        1. Hi Kudzu – I was having 5 mins of being obtuse! My meaning below was a math model of its affect is not the same as a math model of its cause. Of course the predictions may still hold good but it does not get us any closer to understanding the cuase.

  36. You said, wrongly, that “a scalar particle has only or speed”. A scalar particle has only magnitude, as eg temperature field. Speed has obviously a sense of direction, it is a vector field, so has spin 1.
    All you say is non sense.

    1. Why do you call the Higgs a graviton ? It’s not the same thing. What is commonly referred as the graviton is the quanta of the gravitionnal field (in a quantum theory of gravitation), and it has a spin 2.
      To paraphrase what you said previously, it is common knowledge.. See for example :
      http://en.wikipedia.org/wiki/Graviton

      The graviton must have a zero mass, as the gravtionnal force has an infinite range. The higgs particle is the quanta of the higgs field, is spin 0, and has mass (between 125 and 126 Gev/c² if the recently discovered scalar meson at LHC is indeed the standard Higgs).

  37. The Higgs particle of graviton has no spin, and no charge. “All fields have a property called spin which is an intrinsic property of the angular momentum of particles.” The Mystery of Mass, p. 42, Scientific American, July, 2005. Since the graviton has no spin because the Higgs field is a scalar field. A scalar field has only magnitude or speed but no spin which is why the graviton has no spin. Also the graviton must have mass because it is part of the same force which gives mass to particles, the HIggs field which produces gravitons and interacts strongly with the heavy particles such as W and Z bosons etc and is responsible for spontaneous symmetry breaking; The graviton’s mass is 125 Gev and it does decay into other particles quickly. Same source, P 42, 43, 44 and also Wikipedia:http://en.wikipedia.org/wiki/Higgs_boson

    1. Who says there is such a thing as a graviton. This is a fantasy philosophy same as string theory so lets not get too carried away. We can all imagine our own concepts – which may be more appropriate. Kind regards

      1. The existence of a graviton follows logically from the existence of particles in other fields. It is similar to it having rained over and your neighbor’s houses. You may not have seen it rain over all of the houses on your block but you can make a good guess that it did. If not, something quite strange has happened.

        1. So we call it a shower just so we can comprehend what is going on? But as you go on to say something else strange ( may be ) happening. Referring it to rain affect is a convenient cognitive model for our imagination and may not actually describe its true nature? Similarly Einstein’s thoughts on gravity. Which caused Newton’s ideas to be abandoned! Which I think is a pity as it could have opened further doors for us if pursued.

          1. We call it a shower because that makes logical sense. In science there is generally the principle that it is best not to use a complex explanation when a simple one will do. Gravity fits nicely as being mediated via particles in a field. It is possible it works completely differently but on the whole we have no indication that the ‘graviton method’ need be abandoned.

            However take heart in that if there is one field where a concept is never truly abandoned it is physics. While the ‘mainstream’ may pursue its own theories and ideas you can bet that somewhere there is a dedicated cabal of serious (and not so serious) physicists out there working on wild and unusual alternatives. A good example is the big bang; the (temporally) infinite cyclic universe is much out of favor and has been for decades yet theoretical work on various models has never stopped, merely been pushed to the sidelines.

            1. Thanks for your further insight. The shower theory infers movement which is not such a bad thing I suppose particularly if such a shower is a homogenous isometric ( like a fruit jelly constantly emerging from its source – a particle free wave ) we always have to find a particle fulfilling a particular function, but if I am not incorrect this doctrine is heavy going especially at the microscopic scales and never be able to see any of them first hand and just measure the affect they are having upon the macroscopic scales.

              We are not obliged to think this way even though popular physics does. Why not have a particle free field?

              Regards the cyclical universe. A juvenile one is currently in existence much like today’s newspaper, but tomorrow a new edition will be printed. How could we possibly know whether universes are not cyclical? And what is occurring, when does not?

              1. The problem with a particle free field is that it is hard to crate a model where the field itself acts that does not produce particles. In a way this is what relativity does. It can be thought of as a distortion of spacetime but also simply as a field that directly acts on particles in it. (We can even envisage other forces in this way, though it tends to break down.)

                In regards to the cyclic universe there are two main ways its existence can be inferred. The first is if the physics of the (entire, non-3D) universe suggests it is possible. For example if we come across a working theory that matches experiment and suggests that the universe has more dimensions than we see that would support the idea that our universe is a 3D brane in a higher dimensional space.

                The other way is to look for ‘imprints’ of past universes on our own. In the past this was actually a hindrance to the theory since every preceding universe changed the balance of radiation and matter in the succeeding one, leading to infinitely long cycles. Current theories may look for asymmetries or unexpected statistical effects in our universe.

                1. I think information fields are – could be particle free zones – inc gravity! For example I have to refer back to my earlier reference that being communication between non genetically coupled species. i.e. the floral entomological mimics separated by some 130 million years of evolutionary gap, Despite the impossibility of copy it happened.

                  I cannot vote for a multi interleaved universe otherwise there would be some tangible evidence of it other than a mathematical conundrum, without hope of any verification. Clever that it is.

                  Despite the fact that particle less fields may be difficult to model does that make them permanent outlaws for consideration?

                2. Nothing is ever ‘outlawed from consideration’ in physics. There are people pushing a creationist view where the universe as it is formed from a giant sphere of water. Fields without particles are small fry by comparison.

                  The problem though, as I should have explained a little more deeply, is that a field without particles is very ‘smooth’

                  Think of the photon. The classical view that comes to mind when ‘field and particle’ is mentioned is a sold ball of something being ‘pushed about’ by the field or other solid balls of stuff.

                  In reality our current description of particles and fields is more like waves on the ocean. particles aren’t separate from fields, they’re built *from* them. A photon is a wave in the electromagnetic field. At the very base of it you have something analogous to that guy looking at the computer screen in The Matrix; particles are just numbers in a field of numbers.

                  A theory then that involves fields with no particles is hard to construct. The first question is what particles *are* if they are not waves in a field. We know we can make particles by adding energy to a field or ‘wiggling’ it in the right way. This lends a great deal of evidence tot he two being linked. Particles without fields are difficult to envisage too, we know they are waves, but then what medium are they traveling through?

                  About the only case where it is possible to view particles responding to a ‘smooth’ field is gravity. Since gravity is so weak compared to other forces its field changes smoothly and on large scales. (It is also very hard to detect its particles.) So we can view say, a proton falling to earth as being smoothly influenced by the gravitational field (or curved space.) rather than ‘jiggling about’ as it’s hit by individual gravitons.

                  With all other forces however it is more or less difficult to make the analogy break down. Two magnets act very similar to two masses interacting gravitationally and a proton ‘falling’ towards a negative charge looks very similar to one falling towards a mass, but individual particles often show much more quantized behavior. Especially in the area of ‘virtual particles’ and things like particle decays.

                  If gravity were in any way of comparable strength to the other four forces then relativity or QM would break down, possibly both. Stars would form black holes if they grew larger than cars, objects orbiting each other would decay via gravitational radiation in weeks rather than millions of years and so on.

                  Given all this anyone who wants to build a theory of particle-less fields (whatever they may be.) has their work cut out for them. I am open to anything that’s self-consistent and testable but I have no idea where you’d even start on such an exercise.

                3. Then the nature of space I suppose is key to everything! Particles make the space? or space provides a place for them to exist in?

                4. That is a very interesting question indeed. Certainly in our current preferred understanding fields make a place for particles to exist in. (If ever there was some place where things like the electromagnetic field ‘ended’ many particles would not be able to exist in that area of space.)

                  But fields are thought to fill all of the space there is. Are THEY space? DO they make up space? Are they just something that happens IN space? Those are deep questions indeed.

                5. Then on this stepping stone of reasoning I have to revert back to my earlier position and just think of it as constantly emerging . After all it is expanding and matter getting thinner – our beloved particles separated by an increasing virtual flat dimension

                6. The current thinking illustrates the incompatibility between relativity and QM. From a ‘relativistic’ perspective space is ‘smooth’ on all scales; no matter how closely you examine it there is no structure, no ‘graininess’ It is the backdrop on which things occur.

                  From a ‘quantum’ perspective there comes a point where space can no longer be thought of as smooth but instead as a ‘grainy’ or foam-like set of constantly fluctuating fields. This has consequences even at the scale of things like protons and neutrons.

                  The fact that two theories give two different descriptions of the same thing (space, gravity.) tells us that both are incomplete and that we need another theory that encompasses them both. The search for a theory that unites the ‘very big’ and ‘very small’ is one of the hottest topics in physics today.

                7. One must also return to the idea of what exists in quantum gaps? Other than any local field caused by their existence. Do the gaps exist because of the quantum behaviour or does it gap space exist it in absence of them anyways?

                8. I am not entirely sure of what you mean by ‘quantum gap’; could you give a reference to something explaining this, or possibly just try to re-word it?

        2. Referring to the Gravity probe B measurement of frame dragging/twisting: I see that in a different light – to me it measured historic gravitation as the measuring device was at some distance from the surface – and if the gravity field moves at C the detected information was historic not real time hence the Earth has moved and appears twisted?

  38. The vacuum energy includes the energy of all fields, the electromagnetic field, gauge fields, hermionic fields, and the Higgs field. As a wrote here before, it does not have zero energy because of the Heisenberg uncertainty principle due to the wave particle duality and the wave nature of all fields. The best way to see this is unlike the electromagnetic field which has a ground energy of zero, the Higgs fields lowest state is less than zero; It can have particles with negative energy. Wikipedia, Zero point energy http://en.wikipedia.org/wiki/Zero-point_energy

  39. If we go by the name graviton, it is a particle with zero mass and a spin of two. According to general relativity, the graviton must have a spin of 2 because its vector potential, spin 2 is a symmetric tensor potential which is required by general relativity. I suggest you check out Feynman’s Lectures on Gravitation. Also the angular momentum is conserved in quantum field theory which also shows that the graviton must have a spin of two otherwise it could not decay into two photons, each with a spin of one.

    1. You don’t have to “suggest” I “check out Feynman’s lectures on gravitation”; one does not get a Ph.D. in this field without knowing these things, and I got mine 20 years ago. In your previous comment you said the graviton *does* have a mass [which is wrong] so I’m glad you’ve corrected yourself in this comment.

      However, your statement about angular momentum is wrong. The Higgs particle (with a spin of zero) decays into two photons, each with a spin of one. The Z boson, with a spin of one, can decay (though the rate is too small to measure) into three photons, each with a spin of 1. Angular momentum addition is not linear, and what is conserved is not spin but total angular momentum, which involves orbital motion as well as spin.

      In addition one should note that a massless particle like the graviton doesn’t decay.

      1. Do you have a good resource on this? I must admit particle decays such as those you mention have given me trouble. I seem to have it stuck in my mind somewhere that spin should be conserved like charge or mass\energy.

  40. I stand corrected. You are correct the Higgs particle does have mass. It is the photon that has no mass. I am still right about the Higgs field though. It gives mass to all particles. The Higgs field and the zero point energy are the same and that was my point. At one time the Higgs was thought to be a massless particle with a spin of 2. My book The Quantum World, by kenneth w. Ford shows a table of force carrier particles on page 256 and it show the graviton has zero mass but that it wrong today of course. I should have known that because according to quantum theory a particle with heavier mass must always decay into lighter particles which have a sum that adds up to the original heavier particles mass. One particle can’t decay into a single particle but only into two or more particles. We see this in the LCH where protons with 7,000 times their rest mass collide to produce heavy particles because E= mc squared so the kinetic energy, the energy of motion of such a proton is represented by many extra virtual quarks and gluons as well as it’s original own three quarks. These quarks and gluons collide together and produce W bosons, Z bosons which emit a Higgs. The Higgs then decays into lighter particles such as two photons, W bosons, Z bosons, quarks etc.

    1. 1) The Higgs field is a field that is charged under isospin and under hypercharge; see http://profmattstrassler.com/articles-and-posts/particle-physics-basics/the-known-apparently-elementary-particles/the-known-particles-if-the-higgs-field-were-zero/ . If this were not the case, the Higgs field could not give masses to the W and Z particles and to the fermions of the Standard Model without contradicting a wide variety of experimental results. Zero-point energy, by the nature of energy, is not charged under isospin or hypercharge, or indeed under any gauge interaction. The Higgs and zero-point energy are clearly not the same.

      2) The Higgs particle was *never* thought to be a particle of spin 2. Just read Higgs’s original papers for yourself! They’re in the literature.

      Higgs, P. W. (1964). “Broken symmetries, massless particles and gauge fields”. Physics Letters 12 (2): 132–201. doi:10.1016/0031-9163(64)91136-9.

      Higgs, P. (1964). “Broken Symmetries and the Masses of Gauge Bosons”. Physical Review Letters 13 (16): 508. doi:10.1103/PhysRevLett.13.508

      3) Electromagnetism is a long-range force because the photon is massless; gravity is a long-range force because the graviton is massless.

      4) You may want to work on your kinematics, and might find this helpful: http://profmattstrassler.com/articles-and-posts/particle-physics-basics/mass-energy-matter-etc/mass-and-energy/

  41. The Higgs particle has zero mass. This is common knowledge. It has energy but energy equals mass E=MC squared. The Higgs field is a scalar field; it does not have a sense of direction. It is everywhere.

    1. No, the Higgs particle most certainly has mass. A massless particle can be produced with any energy greater than zero. Everything in the universe made of normal matter glows with photons. Even in liquid nitrogen objects emit a small amount of very low energy radio waves.

      The Higgs particle cannot contain less energy than about 125 Gev. This also makes it incredibly hard to produce and give sit a short lifetime like the W and Z bosons.

      Whether or not the Higgs field is scalar does not affect the fact that its particle has mass. If you have a reputable reference that says this I would be most interested indeed.

    2. The Higgs particle has just been discovered, and its mass has been measured as 125-126 GeV/c2. Last time I checked, the equation 125 = 0 was not true. So what you call “common knowlege” is common only to you.

  42. The HIggs fields is an energy field and if you collide particles with enough energy you make a wave or particle in that field. Particles with a lot of mass are deflected by that field.

  43. Excuse me, because of the Heisenberg uncertainty principle, we can’t know both the position and velocity of a particle simultaneously. When we measure the velocity accurately, the position becomes uncertain and when we measure the position, the velocity becomes uncertain.

  44. According to the Heisenberg uncertainty principle, we know both the velocity and position of a particle simultaneously. We can’t have a particle with zero velocity or zero position because they would both position and velocity be would known for certainty; A particle with zero position and zero momentum, the momentum is mass time velocity. Consequently, empty space is seething with a foam of virtual particles which are created and destroyed in a very small fraction of a second; matter and anti-matter particles are created come together and annihilate in a very small fraction of time. Detectible photons are produced as a result of these collisions of quantum virtual effects.

  45. The rest state, the ground state or lowest energy state of all fermions or matter particles, the particle which have mass come from the zero point energy or quantum vacuum energy of empty space. All the mass of matter particles are calculated from the from the kinetic energy of quarks and gluons which protons and neutrons, etc are made. The rest is the ground state energy of the atom. A process of renormalization is used to calculate the mass of particles. “The sum of the negative infinite values and the positive infinite values almost cancel out leaving a small remainder, the finite observed values of mass and charge.” p. 107, 108, The Grand Design, Hawking. These are the infinites of quantum field theory which should be considered a quantum object where virtual particles can appear and be destroyed at any point in space. The Quantum Man, Krauss.
    2. The Higgs field does give mass to all fermions which has now been proven by the LCH through particle decay. Heavier particles like quarks can collide and decay into a W boson and graviton etc.
    3. The HIggs particle has no mass because only particles with zero mass can go long range like photons. W bosons have mass to they can only be exchanged short range. Hawking, A Brief History in Time, pages 90-91. Particles are also waves according to the wave particle duality. Moreover, Energy equals mass time the velocity of light squared E = MC squared.

    1. The Higgs particle most certainly has mass. This restricts the Higgs force to a very short range and it why we don’t observe it. It is also why it is so hard to create Higgs particles.

      The Higgs field is what gives particles mass.

    2. “3. The HIggs particle has no mass because only particles with zero mass can go long range like photons.”

      As I described in my post of 5/31/13, the Higgs particle, unlike a photon, does not go long range. If it did, we would have discovered the Higgs force many decades ago, because the apparent gravitational force between different materials would have been different. So in fact, your own statement actually proves that the Higgs particle DOES have mass; if it did not have mass, then its long-range force would have been observed long ago.

  46. “since two particles may be on a course where they should classically miss, yet, due to their quantum nature, interact”.

    Yes exactly I am trying to think that one through hence my student take on it’s available and non normal energy condition in the collider. I am assuming that the proton in this case has its mass somewhat multiplied by it’s velocity? Under what other condition in nature ( cosmological etc) can we expect to see similar conditions of free proton? or proton only dynamics. Thanks again for not recoiling at every nonsensical idea.

    1. The mass of a proton moving at speed v is equal to p/v * root (1 – v^2/c^2) which at low speeds simplifies to m = p/v but heads to infinity as v approaches c.

      I am not entirely sure what you mean by ‘free proton’, but if you are referring to a proton not bound to an electron then it doesn’t take much energy at all. The solar wind is full of naked protons and they bombard our atmosphere constantly. Hydrogen will ionize if given barely 14 eV of energy (That’s plain electron volts, not the billions of electron volts needed to make a Higgs.) Though they don’t last long on earth due to quickly meeting electrons from other atoms, the universe if filled with them. (For one things stars are made mostly of them.)

  47. Various observations on this exchange, hoping that I haven’t grossly misunderstood:

    * In an expanding universe, where does the “new space” come from?

    Having had to probe this in my own work [, see preceding top-level post], I decided that this is buried in the “connection” criterion, which says (loosely) that if little (well-defined ‘local’) bits of space-time are to be connected together to “tile” and therewith *form* 3+1 space, the complexity of the result cannot exceed the sum of the complexities of the components being joined.

    More specifically, suppose bi-vectors ab+cd and ef+gh, which pairs are ‘dimensional atoms’ of 3+1d, each with complexity 2+2=4, are to be connected. This can be done by forming (eg) ab+ef and cd+gh, which also have complexity 2+2=4. [All these are the precursors of actual 3+1d spacetime.]

    The increase is an increase in dimensionality (since all such pairs are orthogonal to each other) which increases the number of distinctions, which increases the information potential, which encourages entropy creation, which means expansion.

    But maybe the connection criterion is the wrong place to look for this.

    * Is there a timeline of sorts that ‘clocks’ the expansion itself? Gravitons?

    Here I’ve come to differ from the received wisdom, due to my computational/process mind-set. The idea of a bosonic “graviton” particle that carries the gravitational influence from place to place (so to speak) is, for me, very “sequential”, in that it implicitly supposes a linear view of causality: [gravitational] event Y is caused by [gravitational] event X, with the “graviton particle” being the connecting causal link, ie. analogous to photons linking electromagnetic events.

    My analysis surprised me by showing that gravity can’t be like this. Rather, gravity’s causation came out as a non-local “distributed causality”, the reason being grounded in the fact that gravity is non-polar and indifferent to polar attributes like charge. In a nutshell, whereas a standard causal sequence YX cannot just be rewritten XY because this trashes its very meaning, this *can* be done with gravitation because gravity is a *mutual* relationship (“they commute”). So for me, “gravitons” are mythical, and an implicitly misleading concept.

    Popping the stack to the starting question, the timeline of the expansion itself is measured by entropy creation, and this “time” is thus only very indirectly connected to the sequential (ie. time-like) reference frames of relativity theory.

    * Novel theories *must* be accompanied by good mathematics to be credible!

    Oh so true. But, alas, it’s not enough, cf. . The math there is very clean and straightforward, and pretty too. But the referees of physics journals – like all referees – have a very hard time with true novelty. They’re *experts*, after all, and can easily evaluate standard stuff. But the genuinely new gets a different treatment – it takes a lot of time and effort to read unfamiliar mathematics, and what happens is that the expert usually concludes that – lacking familiar signposts – there must be *something* wrong with it somewhere, else some other expert would have found this approach already. But finding that ‘somewhere’ will take a lot of effort, so the referee just lists some complaints about various details here and there, and recommends against publication. After all, it’s probably wrong, right?

    And this is why *I* am so grateful for this site. Hopefully, here I can get some meaningful critique.

    1. You are correct in your description of physics journals, and publishing anywhere is often not easy, even for ‘professionals’; the best bet for a totally new theory it to attempt publication in one of the smaller journals while avoiding those with a poor reputation (Like the Journal of Cosmology.) Knowing a physicist will also help as, as you note, they can offer critique on your theory and its presentation as well as helpful links.

      On the ‘origin’ of new space this is a topic which I am poorly prepared to comment. In the consensual view, where space is smooth things are much simpler since there are no discrete creation events. Models such as the one you propose that also use expansion of space tend to have discrete creation of new ‘nodes’ which must be handled in a logical manner. Sadly I do not know enough on such theories to comment.

      I am curious as to how you would ‘rewrite’ an event X->Y as Y->X; If I have two bodies with opposite electrical charge I cannot say one attracts the other, they are both attracted to each other, each one exerting a force on the other. Similarly two bodies with energy attract each other gravitationally, exerting equal and opposite forces on each other.

      The only way I can think of where you would have a cause and effect relationship would be if a gravitational signal (A gravitational wave.) were sent from one body to affect another in the same way a pulse of light or changing magnetic filed initiated by one object affects another. And I am not entirely sure how this could be ‘rewritten’ so that the affected object was the cause.

      1. Kudzu – I was wondering could we exchange information via private email so my Roman rhetoric does not constantly clutter Matt’s good site ? I have contact in Maine USA who is preparing some geometrics for me. Hopefully this will develop into a moving computer graphic. Would this be possible I would like you to take a look at them and criticize accordingly? thanks

        1. That should be no problem; simply send whatever you want to my gmail at gareth.l.dean (I am sure you can figure out the rest.)

          1. Noted thanks – will be a little while as I want to send developed images centered around my roman rhetoric for opinion thanks again.

          2. PP collision:

            How do we know? for instance the particles produced ( whatever classification in the SM ) are more to do with the Swiss Electricity Board and less to do with the actual composition of a proton? The proton is pumped up to a high velocity it has an artificial and substantial new energy state ( virtue of near C velocity ) thanks to the SEB above.

            So what we detect is the conversion of the SEB energy into particles and separate from what the proton could show us in the classical world if was at rest ( relative ) on the table?

            So in a sense are we are demonstrating that we ‘can’ create matter from energy, rather than expose the units of composition of an existing state of matter? i.e. The LHC or any collider is detecting new matter not exposing it?

            1. Now this is an interesting question.

              The function of the LHC and similar particle accelerators is NOT to fragment ordinary matter to look at its ‘bits, but to create new particles out of energy.

              However these accelerators CAN tell us about the structure of matter because the result of particle collisions is VERY sensitive to what the structure of those particles is.

              Imagine colliding two balls at high speed, say bullet speed. If they are billiard balls, the same all the way through they will either bounce off each other, or shatter into pieces. But if they are instead hollow and filled with small metal pellets, looking, feeling and weighing exactly the same as a billiard ball, the collision will produce a scattering of metal fragments. Many may even pass right by each other.

              Likewise, while quarks were once just a nice mathematical idea the past few decades have given us an incredible amount of data supporting the quark model. (And not the initial ‘3 quarks per hadron’ model, but the more complex one illustrated on this site. You will notice for example that the mass of the up and down quark has been changed downwards in recent physics books compared to older publications.)

              What it pays to remember is that it is not protons that are being collided, but the quarks and gluons within them, and we have overwhelming evidence for this, given on this site. We know there are particles, or something very like them inside protons. We know they contain only a small part of the proton’s mass. We know that they come in several kinds of different properties and energy and that the gluon-like particles are the most common by far.

              We know protons do NOT contain things like electrons and that the particles we’ve ‘seen’ are very, very quarklike. If they were not then the products of literally trillions of experiments would be very different.

              1. “The function of the LHC and similar particle accelerators is NOT to fragment ordinary matter to look at its ‘bits, but to create new particles out of energy.”

                Thanks again that helps to continue to build the picture. I need to return to the energy question: Can I assume that the energy provided by the LHC to accelerate the proton is also part of the conversion into matter in isolation of what is happening to the internal content of the proton?

                1. Well… yes and no.

                  A large amount of the matter produced in a p-p collision will not be the original proton itself, nor will it be anything contained within a proton. In that respect you could neglect the proton entirely and just view it as a carrier of energy that is converted into a wide variety of particles.

                  However the internal structure of the proton greatly influences what the collision will produce; only a small fraction of the energy of the proton will be carried by any one quark\gluon for a start, and the expanding jets of hadrons is seeded by the remaining fragments of the proton. A collision between to equally energetic electrons would look much different.

                  In this way it’s much like growing a tree. All trees are nearly entirely mass drawn out of the air (from carbon dioxide.) with only a little bit being the seed, but that initial seed determines a lot about waht the final tree will look like.

                2. “In that respect you could neglect the proton entirely and just view it as a carrier of energy that is converted into a wide variety of particles.”

                  Great that helps.

                  Ref: P-P vs mass energy equivalence:

                  The proton has it’s mass substantially increased by virtue of its externally applied energy – resultant velocity ).

                  Q: So the LHC enquiry is to determine what is the construct of this increase in mass? ( in isolation for the moment ).

                  Where such additional mass experienced is short lived because as soon as the collision occurs its velocity will be substantially reduced and cause extinction of the mass so created? Hence their rapid decay from being mass into heat?

                3. The LHC was constructed for a number of reasons, notably to detect the properties of the Higgs boson, but also to test the standard model at higher energies, look for signs of supersymmetry, new particles, test the mass of particles like the top quark and more.

                  It was not constructed to determine why velocity increases the ‘mass’ of a particle, since we already know this.

                  The increase in a particle’s ‘mass’ with speed is a direct result of the equivalence of mass and energy. Anything made to move has been given energy, which is equivalent to it gaining mass by a number of measures. Thus anything moving will seem naively to be getting more massive.

                  This means that it will take MORE energy to increase he speed an equal amount increasing the ‘mass’ by more and so on. It is not hard to prove that this will set a limit on how fast an object can go when its energy is infinite. (That depends on the precise relation between ‘mass’ and energy.)

                  The long and short of it is that we know why ‘mass’ increases with speed and why it does so exponentially. It is a basic part of how our universe works. (Though more fundamental questions such as why inertial mass and gravitational mass are the same, meaning all objects fall at the same speed are not clear.)

                  Such increased mass can be quite stable, cosmic rays can spend millions, even billions of years with masses trillions of times their ‘rest mass’ The short life of speedy particles on earth is merely due to the fact that everything moving here quickly bumps into something else.

                4. Yes understood, but in the case of LHC a temporary mass increase is sustained by the uniformity of the external energy applied to the particle. Is this the reason why they degrade ( mass reduction ) on and immediately post collision? They are rapidly de accelerating? ( instantaneous de acceleration ? ). Like the opposite timing of a bomb detonation.

                5. Incorrect. The increased ‘mass’, that is the high velocity is NOT being sustained by anything. The LHC *increases* the particles’ velocity by clever manipulation of magnetic fields, but if all the power were to suddenly fail the particle wouldn’t slow down, it would keep going at its high velocity until it hit something. (Probably the LHC’s wall.)

                  The loss of speed in the particles involved is not as quick as you would think; the two colliding particles are destroyed, but (many of) their fragments are still moving a >0.5c. These energetic particles decay and hit things themselves producing more fragments with velocities around 0.1c and so on.

                  Were the LHC’s detectors not solid matter it would take several seconds to slow things down to sub-sonic speeds. However solid matter is an excellent shield for particles like hadrons so we see them stop ‘quickly’ in the same way a car stops quickly when it hits a solid brick wall.

                6. So the particles once at 9.999…% the LHC can be technically switched off without change to the particles velocity ? What is the energy consumption when they are at max velocity? Is the power reduced?

                  If the fragments are at 0.5c post collision a substantial difference than the above figure. Would or does this not account for their reduction of mass as this lower figure? Just in considering the numbers in the Mass vs energy equivalence?

                7. The LHC consumes power in doing two things to the particles inside it. The first is to keep them magnetically contained, that is, to keep their path circular, instead of straight. This doesn’t give the particle any more energy, but it does require that the particle be constantly accelerated towards the center of the LHC. (Equivalent to how a planet stays in orbit by constantly falling towards its star.)

                  This is why the magnets have to be so powerful; since the particles are moving so fast (And have so much inertia.) a strong magnetic field is required to change their direction. This power consumption *increases* with particle speed.

                  The second thing is to increase the particles’ speed, which is done by tweaking the magnetic fields so that the particles feel a pull forwards or a push from behind. The proportion of power consumed this way depends on how fast you want to accelerate the particles to their top speed; the faster you do so the greater proportion (Since you will spend less time using ‘containment energy’) This power consumption can be kept the same, but in the LHC is increased as particle speed increases so particles build up enough energy quickly.

                  If the LHC were simply to keep a stream of particles at maximum speed then it would require significantly less power (but still a substantial amount since the magnets there are not permanent and need a great deal of power to run and keep cool.) However its purpose is not containment, but acceleration and collision. Therefore as soon as particles are at peak velocity they are used and a new set accelerated. This results in rising energy use, a sudden drop, a rise… in a series of peaks and troughs for each burst. (Though the actual situation is much, much more complex.)

                  The reduced speed of fragments produced in a particle collision is due to a number of factors. Firstly, it is not easy to divide up fragments into ‘generations’; two colliding protons are like two colliding water balloons, more a splash than a clean collision.

                  The initial result of two colliding protons will (mostly) be a series of particle ‘jets’ as the two protons mingle and are torn apart, the color force between their quarks and gluons trying to hold them together, converting kinetic energy into more particles.

                  After the jets have condensed and stabilized into definite particles moving away from the collision in all directions their velocity is reduced simply because the energy is now dispersed over a lot more particles in many different directions and a good portion of it has been converted to mass. These particles of course are still moving fast enough to produce their own messy collisions with anything they collide with.

                8. Don’t worry I don’t seek the rationalization of the earlier question. I have assembled the ideas and concluding the 15 page paper. Respectfully would like you to have a look at it [criticize]. It is very simple, it promotes and reinforces in simple language the validity of the primary dimension and brings argument between Newton and Einstein. Both are relevant but each separated by their vision confined by their ideas of the Absolute vs 3D+temporal of Einstein + Minkowski. I think I have unwrapped a mathematical way of legally presenting this case and cause the opportunity of doubt regards the singularity of 3D in isolation. We have 2 schools, we can believe in both. However, one does not take us forward the other does [ in my view ]. This is not outside the limitations of Matt’s site as it invokes opportunity of detection and measurement over and above the super microscopic scale, and more importantly provides for a fundamental mechanism for gravity – not just the equations of its effect. Please send me an email so I can copy to you in confidence. The acronym of the theory is TONS! By total coincidence. Kind regards

                9. Actually a man of your deep knowledge physics who makes a comment that he thinks that time does not exist, I find rather encouraging. It would be good to get the meaning of time and separated ( if it can be separated in the mind of science ) how it actually participates, or control in my vision of things.

                10. If I have a regular solid spherical which is constantly enlarging/dispersing its volume at value C. is the following expression correct, where I need to illustrate the intensity at any particular R.

                  4/3π r³C x 1/R² ( distance from centre )

                  I seek a simple expression for the entire solid spherical not a quadrant of it. Can I change the inverse ² into a ³ ?? or does it remain as a ², and is C in the right position – which I have doubts about. Is there a simple ideal way to express this dynamic?

                  I tried rotating your email address and using .com but I failed to send an image? I am obviously doing something wrong?

                  thanks

          3. PP collisions: Following my absurd notion for one further step.

            Could it be, that because it is ‘not’ at actual value C ( albeit close ) all we are seeing is the inability of energy to complete its transformation into a persistent form of matter ( particle’s )? And if, we could achieve C using the proton as a conveyor of that energy, do you think that following a collision that the energy is then able complete its transformation and we see a persistent form of matter produced? E.g all we are detecting at the moment is an indication of what that quantum of energy is trying to do and cannot complete the process because it is too low?

            If that could possibly be true then have we evolved the concept of relativity, which illustrates that matter can only be formed providing (energy transmission ) is at energetic value equivalence to C. Or something in the background is, which sustains its persistence once formed? E.g. Light is persistent in the form of photon once transmitted from incandescence. It can be slowed down artificially but remains persistent as an identifiable particle. So are we missing something?

            When the LHC powers up and more quantum energy available and assuming it cannot improve upon its 99.999’s… % then would it still fall short of the above ability? And see the same particles – but much more of them as there is more energy available to be converted. And when we see Higgs P is there any possibility that it is a result of a specific and rare collision?

            1. This would present some problems; recall that cosmic rays are bombarding earth with particles millions of times more energetic than the LHC produces and the big bang itself should surely have had a large enough energy. If we assume that the energy level required is on the scale of the ‘Oh God’ particle then the universe would have billions of times less particles each with the mass of a baseball.

              Of course particles moving faster than light, unable to slow down to light speed can theoretically exist, so called ‘tachyons’, but they seem at present to be unlikely.

              1. Yes the notion of tachyons is not new to me. However, if they did exist then my notion of a constantly populating manifold of space at value C would be destroyed ( assuming it could not be destroyed before that ! ). If a super light speed particle existed in our local universe would upset all kinds of things. E.g a Cosmic particle reacts with our relatively static material atmosphere. Likewise I assume that a tachyon would react with our relatively static manifold and unable to penetrate it, or very quickly degrade into something else? And if that was happening presumably we would have some evidence of it? E.g. Our ionosphere provides the ability of an accidental shield – likewise the construct of the universe protects us against rogue material such as a Tachy??

                1. One of the big things for me would bet he fact that tachyons would be traveling backwards in time; the future could send a signal to the past, which raises all sorts of questions.

                  A tachyon itself could be very low energy, indeed as low energy as you wanted, which would take care of stability issues and shielding. Fortunately we have yet to see any of these, which suggests they are either totally absent, unreactive or very high energy. I thus don’t bother myself with them much.

                2. Yes! You caught me on that one but a subject which is outside the law of this site. Although I have to respond. If we live in a background of constant time then everything referenced to it all happens at the same time! ( In my thinking ) as far as the background ‘0’ is concerned.

                  My understanding is that these imagined particles move at >C this is what makes them different? And if so, how can they have low energy if I am thinking of velocity. Or can something have super C velocity and have low energy? Now I am struggling with the two?? But we live in the HZ where C has an upper fixed value so how could it transit our space?

                3. The behavior of tachyons is what I like to think of as ‘negative normal’ and is a common occurrence in physics. A normal particle has minimum energy at zero speed and a maximum energy at speed c. A tachyon however has minimum energy at infinite speed and maximum energy at c.

                  It can be tempting to think of some things as ‘beyond infinity’, which shouldn’t be possible, but these usually have a ‘mirrored’ behavior (Such as negative temperatures.)

                4. “One of the big things for me would bet he fact that tachyons would be traveling backwards in time”

                  But the time passing in this case would be ‘our 3D experience’ of time. This is going to get me into trouble, if we have a background of zero the future already exists, as does the present and past. ( at the same time reference). There is no going backwards or forwards, it’s just constant ‘0’. In trying to give birth to an idea existing as 2 entities of it! If the background could run into ( –0), then I suppose we could meter our experience ?

                  One further step if time did run at (-0) would change everything including Gravity, on the basis that my concept is that gravity as is light is totally dependent upon it. In as much it may not have a graviton construct but is nothing more than the manifold of space creating new 1D space point for point ( is not moving but getting bigger = more of it. – I am aware of the contradiction) . All it is doing is conveying information at ‘C’ in 3D everywhere – especially the current and variable energy states of bodies hence gravity ( and our meaning of going backwards and forwards in ‘our observed time’ ). For example our milky way may not have a black hole, but as far as ‘gravity’ of the suns concerned it still does. Because the ‘0’ time population between them is still being received from a historic event.

                5. I personally dislike the idea of the entire universe existing all at ‘once’, past and future. Firstly it makes me wonder about random processes, either they would have only one possible result that already existed, or the entire multiverse with all possible results exists. It also makes ‘time travel’ possible if the universe is set up correctly. (Though this ‘time travel’ will not change the way things are, it will always have been.)

                  Indeed I do not believe that ‘time’ exists at all myself, but this is likely something we shouldn’t be discussing here.

                6. Yes fascinating subject. Maybe in some universes ( time-0, 0, 0+), where matter also appears and disappears?)). Varying like a tide? On my original manubo site I put up an image of a cut cauliflower. It just seemed like a good natural candidate shape for a potential multiverse, daft as this first appears. Next time you have one cut it in half and look at the bundle of complex but uniform shapes in profile. It has a central flow which branches out into a complicated internal and external florets each one could be a different universe with its unique conditions. Where the rate at which it is developing is at variance to its neighbor. All the same but all different! The only thing which remains constant is the colour. Then structurally one returns to fractal ideas such as Julia and Mendelbrot sets. Nature is full of these another example being species of fern patterns.

                7. Dear Matt and Kudzu, I am pretty damn sure that I understand the process which causes a clock to increase time as it moves away from a gravitational field, our field or any other field. Would you both like to receive a pdf which describes this and the process involved? I have published a treatise which, will naturally have a very low global profile. However, if you are sufficiently curious I would be happy to send over an email with description. If you can agree the good news is that that you are both implicated in its validity, 1.because I am dedicated to your site and 2. Because kudzu has put up with my endless questioning. Regards.

      2. Proton vs proton collision:

        When two of these items collide how do we know if they share precisely the same axis?, and not slightly offset. E.g on a game of billiards if two of the balls interact exactly head to head their reactive motion will be exactly opposite and the ball will return to the cue. Then if slightly offset the balls will go of at a random angle determined by the collision offset. Under those conditions would It quantify the force in the collision? The proton is tiny to us but relative to each other they are huge. So is the resultant energy of the collision subject to a trigonometric value? If so, would this affect in any way the combination ratio of particles released and observed? And could it be that when we see a Higgs like the protons have experience a 100% head to head contact?

        1. This is an interesting question indeed.

          The first thing to note is that protons are not what is colliding, in the sense of two particles hitting each other, in these ‘p-p collisions’; What is colliding is a quark or gluon *inside* the protons. This does not however change the argument that you bring forward.

          The primary difference between this case and that of billiards is that a classical ball has a solid surface. They cannot pass through each other or hit and miss at the same time or do any of the wonderful things that subatomic particles can.

          Now imagine two particle moving towards each other at a given speed. They are, as you know disturbances in their respective fields, as well as other fields they interact with. These particles are always interacting, even if weakly due to the infinite reach of forces like gravity ad electromagnetism. At some point these interactions will result in the two particles changing their direction of motion.

          Saying two particles ‘collide’ is not the same as saying two classical object collide; there is no definite solid surface involved and position can be a bit iffy. Calling these events ‘interactions’ is, in my opinion more accurate, since two particles may be on a course where they should classically miss, yet, due to their quantum nature, interact.

          Then there is the fact that interactions can well change what particles are, especially at high energies. In this case what emerges from the interaction is not two particles that have ‘bounced off’ each other, but a slew of particles whose total properties are equivalent to those that went into the interaction.

          So when two particles collide, their properties, including their momentum do matter, but it would be misleading to think of them as small solid balls bouncing classically off each other.

    2. “I’m sorry, but this website is NOT where you will get meaningful critique on novel ideas. If I had to provide novel critique on every novel idea that gets thrown at this website, I’d spend all my days critiquing.” …

      Hi Matt yes we understand these terms and conditions and just not practical for you to involve yourself in technically road blocked concepts.

      “Learn to talk with physicists; then you’ll make progress”

      Personally that is why I am here, as your site is a creditable one without all the claptrap of other parallel sites on a similar subject. I have tried to open a meaningful dialogue with many respected physicists around the world. In some instances succeeded to a minor degree. The reluctance always come down to the same reason ‘ time’ and lack of it. E.g “ Even if your intuition is correct I am sorry I cannot involve my time with it” We are all in the same boat, and fully understand. However, I take this opportunity to publically present my sincere gratitude to Kudzu in involving his time to respond to my many questions.

    3. I’m very sorry that I misunderstood your intentions, Dr. Strassler, though upon re-reading you made them clear enough. So please accept my apologies for this. And, since I am committed to a discrete and (unboundedly) finite view, I agree that my posts are “speculative” relative to the main stream. So I expect that I won’t try to contribute again.

      I would though like to address Kudzu’s questions.

      “I am curious as to how you would ‘rewrite’ an event X->Y as Y->X; If I have two bodies with opposite electrical charge I cannot say one attracts the other, they are both attracted to each other, each one exerting a force on the other. Similarly two bodies with energy attract each other gravitationally, exerting equal and opposite forces on each other.”

      The quick reply is we express causal processes as products of idempotents, and so changing the order implicitly changes the causal relationship (which is ultimately thermodynamically governed). Being computer scientists, we’re not used to thinking in terms of ‘fields’; on the other hand, the *role* fields play is taken over by the quantum/causal potential (cf. Parseval’s Identity below) and its evolution.

      The difference (that we found) between electro-magnetic and gravitational attractions arises from the fact that what we experience as gravity in 3+1d rests (in our formulation) upon quantum entanglement. The quaternion isomorphs (“tauquernions”) forming ‘space’ are namely *also* the Bell and Magic entanglement operators (and e-bit states as well). [This doesn’t mean though that gravity propagates instantaneously.]

      So entities are already in pairwise association *before* they are projected (thru the tauquernion frame) into 3+1d. In contrast, electromagnetic influences are woven *into* this framework, ie. “in it” and not “of it”. So the bosonic connection (photon vs ‘graviton’) has a different tone for this reason. Your question is framed from a 3+1 pov, and as if gravity must be understood solely in that frame’s terms. We have more context to work with and can therefore be more detailed.

      “In the consensual view, where space is smooth things are much simpler since there are no discrete creation events.”

      The paradox of discrete vs. continuous has long presented a puzzle. I have found what I believe is a resolution, based on a generalization of the Pythagorean theorem called Parseval’s Identity:

      Theorem (Parseval, 1799). The projection of a function F onto an orthogonal inner-product space *is* the Fourier decomposition of F.

      The space is usually an ordinary vector space, and the basis for the theorem becomes clear when one recalls that the sides of a right triangle can be expressed in terms of sine and cosine, which then easily generalizes to n dimensions, yielding F in terms of a big pile of sines and cosines. Assuming that a Fourier decomposition is unique, however it be calculated, then completes the argument.

      Thus the discreteness of our model is in terms of *dimensions* and not really *numbers*. On the other hand, a Fourier decomposition lives in continuous space and maps the real line. While such matters are very much the province of the the mathematicians, it nevertheless seems that the transition between the two realms is in this way well-defined and sound. At a minumum it shows that every expression in our algebra is a wave operator. [I note that applying the identity to geometric algebra’s graded space means that Pontryagin Duality – a further generalization of the Pythagorean relationship – must undergird the continuity argument.]

  48. For Prof Matt:
    “And if gravity were instantaneous, rather than delayed as in Einstein’s theory,”

    Yes; but why delayed?

    I know ( understand ) that if the sun were to magically disappear in <1 sec. We as a body ( earth ) would continue to rotate as if nothing had happened for 8 minutes following ( approx ). What is your explanation for that ( other than the speed of light )? I would appreciate your insight which does not hang on value C – thanks

    1. The primary reason is that relativity is true; there is a maximum speed that anything can travel through space. Gravity is no exception to this, if it were instantaneous we would need an explanation, as to why it was the exception, as things stand it is behaving normally, just like everything else.

      The view of gravity being mediated by massless particles traveling at light speed is the most conductive to this, since using that model gravity travels at light speed as a matter of course and your question becomes ‘Why can’t gravitons travel indefinitely fast’?

      1. Thanks, yes gravity ( imagined gravitons ) are limited to value C. Hence the action following the action has a value C hysteresis. Sun vanishes, its gravity vanishes, information is transmitted via the manifold – currently preferred gravitons ( spacetime takes temporal time to vary its depression, it flattens ( missing coupled energy in this case ) and the earth flies of at a tangent from its historic orbit.

        Which returns me to my earlier notion, that bodies may be connected through constant time. Where information – has to be transmitted from one body to the next in 3D temporal spacetime in this case. ( Information in both directions simultaneously – like the arm example ) But the above underlying phenomena may have more to do with an upper influence which is determining the above hysteresis, and value C is evidence of its behavior.

        E.g. We have to await for more primary manifold space to be produced in order that the information can be communicated. On the basis that no information can be transmitted through primary space which is static.

        For example.

        1 primary Space frame is contiguous with a body. Something happens to that body and the next frame which is produced records that packet if information. In the next moment another frame is produced so on and so forth. Until eventually sufficient frames of information have been produced in order to pass that information onto the remote body, across the gap of 3D spacetime. Maybe you could find a link with this thinking with the behavior of light in GTR.

        If anything like this is realistic we can forget about 3D spacetime deformation as a visualisation. And Gravity Probe B is only detecting information from the surface which is historic.

        i.e if the satellite moved closer to the earth the twisting affect of spacetime diminishes ( shortening of time relative = less new frames of information ) – and as it approaches closer to the earth surface there would be no twisting detected at all. The Probe is detecting a historic event not the current one.

        i.e. If the earth stopped spinning it would continue to see spacetime with a twist, which would not be true, as this is historic information. The update can only be possible by the production of New Primary Space conveying the information not a graviton? Possibly?

  49. “The problem there is that the main force allowing matter to escape a supernova is not centrifugal, but radiation pressure. (Basically the massive particles are given enough velocity that they can just fly away.)
    Massless particles have no need of this energy boost since they always move at light speed. All that keeps them bounded is interactions with massive particles. (Photons in the star’s core for example are constantly colliding with, being absorbed and being reflected by atomic nuclei and electrons.)”…..

    Sorry I missed this reply in my email and did not look at PMS web comments – I just found it. Thank you for your advice fascinating reply…….

    You have a lot of detailed knowledge. O K so I can discount simplistic centrifugal. I suppose it may participate in a very minor role. But the Rad’P is far more powerful. However, the RP is acting in its own was as a separator – getting rid of the big stuff! As you comment the massive particles. This is obviously extremely important part in the success of the next phase. Defensively’ centrifugal or RP as description is lesser import the main thing is that the new system in order to pass into this next phase presumably must get rid of them? Otherwise it does not inititate?

  50. Dear Prof. Strassler –

    Like so many of your readers, I say many many thanks for this initiative. You certainly nailed me with this posting! I hope we all don’t wear you out with our tortured inquiries.

    I’m a computer scientist, and look at “processes” from the point of view of an operating system: it does not, it *cannot*, matter what the various processes are doing – that’s their business. From the os point of view they all look the same. Just anonymous abstract discrete processes, doing what processes do: Wait and Signal to the outside, maintain stable (often concurrent) state inside.

    These being *discrete* processes, one can imagine doing some combinatorics both along and across various event sequences, and hopefully find some structure. And that’s what we did, starting with processes with one bit of state, and building up from there. We report the results at TauQuernions.org.

    This is where your post about the Higgs and gravity comes in. One would expect that a purely combinatorial approach like ours would produce results unbiased by any other criteria – prior knowledge, theoretical constructions, biases, what-have-you. One pattern that pops out is

    H = (ab-cd) + (ac+bd) + (ad-bc) = Tx + Ty + Tz

    which is nilpotent. In fact, the three pairs are isomorphic to the quaternions except that they are irreversible, and as well, each is an entanglement operator. Quaternions being the very definition of 3d space, we MAYBE made the mistake you describe, of confounding the Higgs, mass creation, and gravity.

    But I don’t *think* so, because we stop at the point where we dot Tx+Ty+Tz with whatever X is to manifest in 3+1d tauquernion space. Ie. we simply punt, not knowing the necessary physics to take it any further. I agree that if this projection disagrees with well-established theory, then we’ve likely figured wrong. But it’s not obvious to me this will be so. Eg. our measure of heft is bits, which translate to energy, not to mass.

    In addition, we found a number of patterns that I don’t really find echoed in the expert discussions. One wonders. Finally, we suggest that it follows that the mechanism underlying gravity is entanglement, and this seems to collide with your argument. Yet our approach, being purely combinatorial, is supposedly theory-neutral.

    So, if our approach *is* truly flawed, I’d sure like to know just where it goes off the rails.

  51. Some comments here :
    – The gravity force your body exerts on the earth is the same as the force the earth exerts on you : F = GMm/r^2. Reaction = action. It is the first newton law. What is different is inertia. With its mass, Earth has much more inertia than you. So the movement of the Earth is very small, your movement is much larger, you fall. My explanation is here within the newtonian dynamic theory framework.
    – Second : Black holes are much better described within the framework of general relativity. The presence of energy and matter bends space-time curvature. The light follows geodesics in space-time (lines of smallest length). In the case of a black hole, these geodesics are closed lines. The light remains confined inside black hole.
    In the case of the sun, the light travelling near the sun surface is deflected. This was famoulsly measured by the british astrnomer Eddington in 1919 duringa a sun eclipse, confirming Einstein gravitation theory.

    [A little correction, as I can’t edit my previous answer : Reation = action is the third newtonian law of motion, not the first.
    The first states that if no force exrts on a body, it will remains at rest or continues at the same speed in the same direction.
    The second states that : F =ma. Acceleration is proportional to the force exerted but inversely proportional to its mass. So Earth with his very big Mass will a have a very low acceleration, while you are falling…]

  52. If matter is made of photons within higgs field, why cant matter escape a black hole then if gravity is not stronger than higgs boson?

    1. I don’t understand your question. Matter is not made of photons (to the contrary, photon is energy, E= h f, whether matter has mass), and higgs boson is not related to gravity, as explained in this article !

    2. I assume that your question would be better worded as ‘If matter particles are all massless without the Higgs field, why can’t matter escape black holes, since the Higgs field interacts so much more with matter than gravity does?’

      The first thing to realize is that while the force of gravity has a certain strength (It is in fact exceedingly weak compared to say, the strong force.) this is a ‘per unit’ strength, an electron say produces a far stronger electromagnetic field than a gravitational one. But this is different from the strength of a gravitational field in a given situation.

      Black holes are objects where enough energy is in a small enough space that and object nearby needs to move faster than light to escape. I am stronger than my dog, but if 50 of them were to jump on me at once, there is no way I can win. Likewise the earth holds me down with its gravity because there is just so much Earth there to gravitate. Gravity’s strength lies in its ubiquity and its cumulative action.

      Secondly, when it comes to gravity what matters is energy, not mass, so photons are in the same situation as anything else when it comes to black holes or gravity in general. This is why black holes are black, not even light can escape their gravity. If we had no Higgs field black holes would still be possible.

      And finally, the Higgs field is not fighting anything. It does not oppose gravity so how strong it is isn’t really an issue here.

  53. Thank you, Alain and Kudzu. Electrons are subject to gravity, are gravity related. May I conclude that the Higgs being gravity unrelated means not directly gravity related? Is this Higgs the Higgs with a mass/energy of 1,25 Gev?

    1. The boson discovered this summer at CERN at about 125 GeV is very likely at least one Higgs boson (some theories as supersymmtery postulate several Higgs). This boson is only the particle associated with the Higgs field. The Higgs boson does not gives mass to other particles (also W, Z…), it is the Higgs field, interacting with these particles, that gives them mass.

      But as you probably know from General Relativity, Gravity is the the result of the bending of spave-time curvature due to to the presence of mass AND energy. Even if the Higgs gives mass to some particles (and not protons, neutrons…), it is not responsible for the gravity due to the preence of different kinds of energies. So, yes, Higgs is not directly related to Gravity, as it is described by General Relativity (and not Newton Law).

      Nevertheless, the Higgs field pervades all space and has a mean vacuum energy (vev) of 246 GeV, so possesses himself potentially a lot of energy that could contributes to Gravity (the problem of “Dark Energy” and cosmological constant).

    1. The idea is that Higgs field gives mass to some particles (electron, muon…), but not all, for example neutron and protons. The latter are composed of quarks, but their mass is not directly related to the masses of quakss. It is difficult to calculate, but a large part is due to interactions between partcles, virtual partcles, and the so called chiral condensate.
      So, the argument of Matt, is that, as neutron and proton are the major part of the mass of atoms, Higgs is not responsible for their mass, so to the gravity of matter…

    2. The Higgs and mass are related. Mass (Or energy) and gravity are related. We can reduce this to something like A -> B and C-> B, does A -> B?

      And in this case, no. This Higgs is responsible for giving some things mass and mass is what gravity acts on, but if there were no Higgs field at all gravity would be unaffected; it would still pull in the same way on mass and energy. And likewise a world without gravity would still see the Higgs field become nonzero and give particles mass i just the same way as it does in this universe.

      1. Kudzu — ONLY Newtonian gravity acts on Mass. In Einstein’s theory, the theory that is widely tested in experiments and widely accepted as much more accurate than Newton’s, gravity acts on energy and momentum. It only acts on mass for slow moving objects because mass is associated with energy; stationary objects carry E = m c^2. I’ve made this point very strongly on this website — please do not confuse my readers by restating this error.

        1. I apologize, I attempted to make a brief reference to it with my bracketed comment on energy, without having to type the full explanation in the reply itself, as I was afraid that it might bloat the reply and make it less readable. However it seems the wording is confusing and should be changed.

            1. Feel free to do so yourself and remove my reply, you have a way with words and concepts that exceeds my own, likely because of all the thought you put into this entire online edifice.

  54. ‘Dark Matter Core Defies Explanation in NASA Hubble Image’
    http://www.nasa.gov/home/hqnews/2012/mar/HQ_12-068_Hubble_Dark_Core.html

    “This technique revealed the dark matter in Abell 520 had collected into a “dark core,” containing far fewer galaxies than would be expected if the dark matter and galaxies were anchored together. Most of the galaxies apparently have sailed far away from the collision. “This result is a puzzle,” said astronomer James Jee of the University of California in Davis, lead author of paper about the results available online in The Astrophysical Journal. “Dark matter is not behaving as predicted, and it’s not obviously clear what is going on. It is difficult to explain this Hubble observation with the current theories of galaxy formation and dark matter.””

    The dark matter core does not defy explanation. The dark matter core is not a puzzle. The dark matter core is not difficult to explain. It is obviously clear what is going on.

    Non-baryonic dark matter and galaxies are not anchored together. There is no such thing as non-baryonic dark matter. Matter moves through and displaces the aether.

    Aether has mass.

    An objects resistance to acceleration is the force of the displaced aether pushing back and exerting inward pressure toward the object.

    Inertial mass is the mass of an object as it is accelerated with respect to the state of the aether in which it exists. The object displaces the aether as it moves through the aether. Since it is accelerating the force exerted by the displaced aether toward and throughout the object is not equally applied to the object.

    Gravitational mass is the force exerted by the displaced aether toward and throughout an object. When you are standing on the surface of the Earth the aether displaced by the Earth is pushing down and exerting inward pressure toward you.

    Both are the same phenomenon. Both are discussing the displaced aether pushing back and exerting inward pressure toward an object.

    Relativistic mass is the mass of the object and the mass of the aether connected to and neighboring the object which is displaced by the object. The faster an object is moving with respect to the state of the aether in which it exists the more the object displaces the aether the greater the relativistic mass of the object.

  55. If gravity is universal and so the graviton couples to the Higgs field, then why does the graviton not have a mass ?

    1. I suppose thinking about it perhaps it is because the Higgs condensate can only give mass via the Higgs mechanism to particles that carry weak hypercharge. Also the Higgs condensate has zero energy so no interaction with gravitons.

      1. I do notice from one of the previous posts on this blog (The known (apprently) elemntary particles, if the Higgs field were zero) he says:

        “What about the force carriers? The Higgs does not affect the gluons, but it rearranges the isospin and hypercharge forces, making the photon out of a mixture of the W3 and X, the Z0 out of a different mixture of the W3 and X and the Higgs particle called the A0, and the W+ and W- out of mixtures of the W1, W2, the H+ and H-.”

        So we know the photon, also massless is affected. (Actually, now I’m curious as to why photons do not have mass when it is a mixture of two particles.)

        1. I think the photon is massless because the Higgs condensate is neutral with respect to electric charge.

          1. Interesting, can you expand on this? If it were not, would all charged particles (but not neutral ones) be heavier?

  56. Prof. Strassler: I noticed that your expression E_1*E_2 = M_1*M_2*c^4 has the wrong operator. You use an equality sign (approximately equal) but when you refer to that expression you read it as E_1*E_2 “is proportional to” M_1*M_2*c^4. Is this an error? If not why are your reading an equality as a proportionality? Since equality and proportionality are not the same thing, please correct the expression to read E_1*E_2 :: M_1*M_2*c^4 or if the equality sign is correct then read it as an equality.

  57. Prof. Strassler writes that, the gravitational force F_grav between two slow moving objects separated by a distance R is proportional to the product of their energies

    F_grav :: E_1 * E_2

    But this is not true for Newtonian gravity

    F_Newton :: 1/R^2

    and it is not true for Einstein’s gravity because a concept of force acting between particles does not exist in Einstein.

    So where did you get this idea that gravitational force between two objects is proportional to the product of their energies?

    1. There is a slight difference between the two expressions. Newton’s classic 1/r^2 equation is for the gravity felt by one object at various distances from another object. Thus if the moon moves twice as far from earth the gravity it feels is 1/2^2 or 1/4 what it feels at its usual distance.

      This tells you how the gravity changes with distance, but not what the actual value is at any distance. *that* is proportional to the energies of the two objects involved. (If I am orbiting the earth at the same distance as the moon the gravity between me and earth is much less than that between the earth and the moon because E(moon) * E(earth) >> (E(me) * E(earth)

      So one tells you how gravity changes with distance, the other how it changes with energy (including mass.)

      In the case of Einstein, gravity as a force and gravity as a curvature of spacetime are equivalent (exactly equivalent); I can model an electric field as curvature in space if I want to. The fact that relativity does not model gravity as a force is a big problem with it and good evidence for relativity being incomplete. This is why we still look out for gravitons despite relativity not mentioning them.

    2. In Newtonian gravity: F_grav = G M1 M2 / R^2 so therefore F_grav :: M1 * M2 and F_grav :: 1/R^2.

      In Einstein’s gravity you are correct that the discussion is much more complicated; but I am trying to explain what I can without going into the details of Einstein’s equations. The points are correctly stated in the text; that the reason Einstein’s more complex equations still give Newton’s law of gravity is that Einstein’s gravity involves pulls on energy and momentum, not on mass, and that we know that E &approx; M c^2 for slow moving objects.

      1. Prof. Strassler writes: “In Newtonian gravity: F_grav = G M1 M2 / R^2 so therefore F_grav :: M1 * M2 and F_grav :: 1/R^2.”

        I don’t agree that F::mM for several reasons.

        (1) F::mM is not an operational relationship. We cannot compute orbits using F::mM alone, a discussion of mM is nothing more than a meaningless academic exercise;

        (2) Orbits are independent of the mass m of the orbiting body; m cancels out. Operational formulas do not contain m therefore mM is a meaningless decorative term;

        (3) multiplication of two masses has no physical meaning; there is no operational formula where two masses are multiplied;

        (4) terms written on both sides of an equation as in ma=GmM/r^2 are decorative and ideological placeholders that have no relevance or effect in computations — they cancel. If you look at 19th century physics textbooks, you will see that physicists wrote Newton’s gravity simply as a=M/r^2. There was no reason to write m on both sides and then cancel it;

        (5) when you state F::mM you secretly assume that M is not part of the proportionality constant in F=GMm/r^2. As I am sure you agree, the statement F::G has no meaning because G is the proportionality constant. But your assumption that M is not a part of the proportionality constant in F=GmM/r^2 is not correct. The effective proportionality constant is GM, because neither G nor M appears on its own in practical astronomy. So, F::mM is equivalent to F::GM and it is as absurd as F::G. GM is probably unique in scientific literature, it appears to be G*M but it is a single quantity; M exists in GM as a decoration to assert Newton’s authority.

        Therefore, neither m nor M exists as active quantities in operational formulas used in orbit computations and F::mM is not a physically meaningful statement and therefore F is not proportional to the product of m and M.

        The important question is why are physicists lowering their scientific prestige to such low levels and corrupt physics with advanced mathematical sophistry to save Newtonian doctrines every physicist admits was superseded by Einstein’s regime? If Newtonian gravity is proved to be wrong then it must be discarded once and for all. But, physicists have chosen the scholastic way and try to save a wrong theory with mathematical sophistry only because it is the sacred theory named after their founder.

        1. Wow! I have received few comments made with such confidence that are so spectacularly wrong in every detail.

          If you have two objects that have both mass and electric charge, then in computing their motion in response to one another, every single statement you have made (about the mass m canceling out) is wrong. Or for instance if you have an object that has a mass and a charge and you subject it to both a gravitational field and an electric field, the mass does not cancel. In fact that is how the electron’s charge and mass were measured [read about Millikan’s experiment.]

          If you were taking freshman physics and made such statements on an exam, I would flunk you and tell you to retake the course.

          As for masses never appearing multiplied, please take a beginning quantum field theory course and learn why in fact they do appear multiplied. It’s a simple consequence of semi-classical field theory, and it is the same reason why charges appear multiplied in Coulomb’s law.

      2. Prof. Strassler wrote on October 26, 2012 at 10:03 AM: “I have received few comments made with such confidence that are so spectacularly wrong in every detail.”

        I am puzzled with your reply. It reads like you are replying to another comment. Just to make sure I understand this correctly, we are talking, as you mention in your post, about “slow-moving objects obeying Newton’s laws”. To me this means that ma=GmM/r^2 is the applicable equation. No electricity or quantum mechanics, string theory or any other department of physics are relevant in this argument. Do you still believe that I am categorically wrong when I said that physicists write m in both sides of ma=GmM/r^2 and then cancel m but they keep discussing mM, as you do in your post, even though they just canceled m?

    1. Dear Arnab R, are you referring to my article or someone else’s? Sorry I find it a bit difficult to track who is commenting on who’s commentary in this site. If mine yes I cant help but feel we exist in 4D the primary unity dimension being the universe expansion where New Space ( void is being produced second for second around us ) where time is homogenous & across it’s entire non ( cartesian & Euclid ) distance. This is providing the space for matter & ( our experience and measured existence of the universe centered around STR & GTR + temporal time ). This New Space is providing ( fundamental space – not spacetime ) in which ‘anything’ can move – and a mechanism which puts an upper limit on the velocity of light! Some experiments have been conducted to determine if this absolute does exist. The problem with these experiments is that they were based around GTR & STR – cartesian space! I don’t think valid actually.

      1. Sir, I commented on the article ‘WHY THE HIGGS AND GRAVITY ARE UNRELATED’..

        And Sir, isn’t that an accepted notion that we live in 4D with one Temporal dimension included ??..And it sounded that You don’t think that existence of Cartesian space is true..and why’s that ??..

  58. Thanks Alain, for your kind consideration but I don’t think we are really touching base here.

    “I did not read entirely your post, because it was too long, but I notice this affirmation : “this particular model considers the universe to exist in 2 dimensions!”
    I don’t know where you read this, but it is simply false. ”

    Forgive me but if you knew the ins and out of the De Sitter example you would have known about the 2D notion. This is not a criticism as I know you were trying to be helpful – and I really appreciate that. Thanks

    The De Sitter model does not satisfy my questions. It is a historic notion, along with other historic notions. The idea I am presenting is at variance to these – a different thought conduit – in which that tunnel offers a different light at the end of it. Hence my first question – would STR and GTR be lawful in a zero mass universe. Of course – I don’t think anybody really knows. But on the basis that STR and GTR are based on the existence of mass in order for them to work I guess the answer is no. Remove the mass and remove STR, GTR, Euclids 3D, Minkowski’s time, Hubble Zone and all the components which we depend on to make any sense of the universe. But we as physicists should be prepared to expect a fresh wave from time to time. Then be prepared to find a way of responding to it. In my thinking we exist in 4 spatial D. We can (ignore) time in the primary dimension as it is homogenous. It becomes non homogenous in the Euclid zone ( where mass exists ). Time is always a problem – but a lesser problem if one can think of a scenario where two philosophies of time exist: 1 temporal in the Euclid zone where mass must be present. 2. In the primary dimension where it is homogenous. Thats all. If anyone is prepared to consider this for a moment some exciting ideas are possible. Thanks.

  59. Prof. Strassler cancels the velocity term describing the motion of a “slow moving” object but still assumes that the object has motion: “For slow moving object p=Mv (where v is the object’s velocity) and pc=Mvc is must smaller than Mc^2. And therefore E=mc^2.”

    How can this be? The object is moving with velocity v, its energy is proportional to its v, then you remove the v from the equation by a linguistic argument but you still keep talking about slow moving object.

    When you remove v, the velocity of the object no longer exists but its energy remains. How can this be?

    In your expression E=mc^2 the object is not moving, slowly or otherwise, so this equation will be true only if the planets were to be stationary. This is an unrealistic, supernatural and unphysical case which is not worth discussing in a physics blog.

    Your argument amounts to setting v=0: E^2=m^2c^2(v^2+c^2) and set v=0 and you end up with E=mc^2.

    A slow moving object is not a stationary object and in fact, from its point of view, slow moving object must have a finite v. You cannot set v=0 and eliminate it as you move from one scale to another. If you want to talk about the velocity of a moving object you must include its velocity in the equation. If you are talking about a moving object and the equation describing the motion of that object does not have the term v, then you are talking metaphysics not physics. In physics, there are no invisible virtues that do not exist in the equation but move objects from outside the equation from the supernatural realm. I hope that you agree with this statement.

    1. Again the argument here is that the object is *approximately* stationary.

      If we take a marshmallow (25grams) then the E = Mc^2 is 2.247 e+15 joules. If I fire that marshmallow out of a cannon in space so it is whizzing around at a hundred thousand kilometers per hour (Quite fast would you not agree?) then its energy due to motion is 9.65 e+6 joules. This is a difference of 4.3 E-7% (0.00000043%)

      We can surely ignore this for the sake of making things simple in a blog dedicated to keeping things simple so laymen can get a grasp of things.

      And I find it humbling to think, no matter how fast I go, by plane, car or rocket, from light’s perspective, I’m nearly standing still.

      1. Dear Kudzu, could you do me a great favour and do the same calc to approximate an imaginary energy of a black hole: For example give this black hole a very high invariant mass the ( say based on the approximate mass of our sun 1.9891 × 10^30 kilograms and it has equatorial velocity of 1. @ 2kms ( actual ), and 2. @ 200,000kms. Is the difference remain so small as in your marshmallow example? Thank you for your time to consider this example.

        1. It’s more extreme, but still negligible.

          A solar mass black hoe will have an energy content of 1.7877e+47 joules, a truly astronomical number. At 2km/s equatorial velocity (Assuming the black hole is a sphere of mass of radius of the event horizon.) the Kinetic Energy of Rotation is truly tiny, on the scale of a quadrillionth of a percent. (Ok, worse than that.) For 200,000kms things improve, getting into the 1% area, soy you can’t neglect it.

          Of course these are rough estimates since I ignored relativistic effects (and I don’t know at all how the mass of a black hole is distributed.)

          1. SO!. even at 200,000kms a large mass has a trivial increase in variant mass! ( Hence Einstein’s advice to ignore it ), Seems unbelievable where the dynamics are so huge and at 2/3 of light speed! Particularly when a proton enjoys a larger mass increase in mass if I am not mistaken – please advise? What happens to the imagined sun mass rotating at 95% of value ‘C’? The same trivial outcome? I realise that the proton is being excited by an external force to make it move faster and is unable to for reasons which are commonly not understood. Off kilter – I can’t help but feel it is acting in some kind of torque with an invisible entity which we don’t yet know about.

            Simplistically, the motion and mass of a car we can recover some of that energy as heat when we apply the brakes!

            Referring to your much earlier comment relating to a ‘massless void’. How can such an occurrence be a Euclidian/Cartesian zone if no mass exists to provide reference frames – hence providing the above x, z, z coordinates? I will look forward to your advice. thanks

            1. Well at a third of c you can’t *really* ignore it, it’s becoming quite significant. If we have an equatorial rotational speed of 95% of c the effect cannot be ignored by even the roughest approximations, here is where the kinetic energy begins to approach and then exceed the ‘rest mass’ energy (and I believe that spacetime itself is dragged about by a large rotating mass.)

              Regarding your second question, this is something that initially confused me. Such a universe can exist as a thought experiment; as such we can apply math to it without mass and reference frames much as we can imagine working on the physics on unicorns.

              1. I had to laugh regarding physics of unicorns – lovely expression!

                Actually, The bit I am trying to fathom in this train of messages is regarding black holes ( this entity is somewhat a visible unicorn ) We currently understand it is comprised of super condensed matter of a collapsed old star. If the quantum of gravity hence generated is more relevant to Joules/sec than it’s mass. Do you think it possible that a black hole may not be as dense as we surmise – for the following account: The originating star may for instance have had a diameter of say typ 3 million km. Should this diameter be obliged to shrink to a new diameter of 5 km – a huge dimensional transition! In doing so it’s original starting momentum would have to be conserved if I am not mistaken? – in it’s new 5km diameter? If that is correct then surely it’s new equatorial rotation would be enormous? Hence the black hole is not as dense as we think and only producing a virtual gravity drain because of the kinetics involved and spinning so fast? Except at the poles of course where the kinetics are lower and the converted mass it consumes is permitted to escape as gamma radiation? And thought of as a mass to energy converter? Likewise a proton at 95% C in the LHC an energy to mass converter ? Very interested to hear your reply…thanks

                1. Well this is a tricky question to answer indeed.

                  What 8are* black holes made of? It can’t be any kind of matter we know of, are they some novel form of matter? A distortion of spacetime? Or something more exotic?

                  You are correct in noting that the angular velocity of a black hole is likely to be enormous, remember that neutron stars often spin thousands of times a second for the exact reasons you note. But what black holes are, beyond their event horizon is poorly understood, and maybe within them the definition between mass energy and kinetic energy breaks down completely.

                2. Thanks – so could they be just kinetic engines because their fixed mass it huge and their diameter is tiny and they rotation is massive

                  ? could they have the mathematical ability in these circumstances to generate all this energy? And create a gravity well light 100 light years in dia?

                3. Well I assumed that the black hole was incredibly dense mass, which it isn’t and to be frank I don’t know what it is. Some say it is an infinitely tiny point, which would thus be spinning infinitely fast, yet still have the star’s finite rotational energy.

                  So I would say until we figure out what black holes are, you can imagine them to be anything. As the professor has stated, they can be made of photons, and maybe they’re just a giant tangle of gravitons, a fold in space or who knows what.

                  But if an object is made of matter, it cannot have significant rotational energy compared to its mass. The reason is, if something spins too fast, mass gets thrown off it. Jupiter does this, it bulges at its equator significantly due to the centripetal force of its half-day spin. A star is similar, so it can only have a certain maximum rotational speed, which is not a significant contribution to its total energy.

                  Even neutron stars with their massive gravity will crack if spun too fast, so if a black hole follows these rules (But why should it?) then most of its energy will be due to the mass that formed it, whatever it turns into.

                4. Brilliant thanks…” They can be whatever you imagine them to be” yes I can understand that. And thanks yes a massive rotation would cause it to sheer itself – that’s obvious.

                  But they cannot be that mysterious as they are a part of regular nature. Mind you it is mysterious how a flower can mimic shape and pheromone of an insect bearing in mind it does not have any ability for feedback of information – to even know that the insect facsimilie it is mimicking- exists in the first place ( I don’t uphold accidental evolutionary accident – that’s an easy get out ). So there must be an information feedback somewhere along the line to cause it to deform its flower to resemble and insect which doesn’t exist in its world of supposed no information.

                  A black hole can be what we want it to be? So do you think that whatever it is – is not in isolation of another force ( not just being a black hole in 3D Cartesian space )? Returning to the concept of 4 spatial realities if the primary 1 is a framework which has a stretch velocity of 300,000kms then whatever it is and doing could be actually experiencing some form of torque against it? The velocity of 300,000kms is constant and hence it enjoys its equilibrium as the primary spatial is steady – also too reason why the suns output is regular in the case of E=mc^2?

                5. Lots of things in this comment are wrong, but I don’t have time to answer now. But black holes are *much* better understood than this comment indicates, and they most certainly are not made from novel forms of matter or something more exotic. They’re made from whatever was used to form them, combined with gravitational fields.

                6. Furthermore it must be in a wonderful state of dynamic ( kinetic ) equilibrium – so whatever they are doing they are stable to keep doing it for so long – and far more stable than any sun?? Help me out please ( no one seems to answer my questions fully )– After a sun goes supernova and is obliged to collapse under its own gravity with the possibility of forming such a black hole.

                  It is further obliged to reduce its diameter and conserve its original momentum. If the diameter is obliged to keep reducing the angular momentum is also obliged to keep increasing. So can we think that its subsequent angular rotation could reach ¾ or more velocity ‘C’? And as it starts to feed will become heavier which means its gravity increases which means it must shrink further? However, there must a point where it achieves an equilibrium somewhere along the way.

                  http://earthsky.org/space/fastest-spinning-star-ever-discovered-rotates-100-times-faster-than-sun

                  This link advises the fastest star rotates at 1,000,000 mph 447.04 kilometers / second. If you shrink this down to 100 kms diameter what will its rotational velocity be?

                  In fact do you think there is a case that the black hole is not a solid at all? On the basis that the mass it is ingesting appears to undergo a phase change just before it disappears into the mystery? Could it be just a very rapidly rotating ball of energy? And can only get to this condition if the original starting sun’s momentum was high enough in the first place – making it a black hole future candidate? ANY comments please…thanks

                7. It seems then I have missed much. It is unfortunate that is is unlikely you will do a post on black holes as I honestly have so many questions that I can find only fragmentary on unhelpful answers on.

                  Can a black hole made of photons decay via Hawking radiation only to photons? What mechanism determines this? How dense is a black hole and in what form is what formed it contained? An so much more. Is there a resource like this site that may shed some light on these questions> If what I know about black holes is like what I knew about particles, there’s a revelation and complete revision of my thinking and worldview in order.

                8. Maybe do you think there is a case for a further transition here: Sun>supernova>white dwarf> ( black hole ) which if the background conditions are right then it actually converts its mass into something else in order for it to survive as a black hole? If a candidate for a black hole has the background qualifications then to complete the transition from its historic existence is obliged to undergo this mysterious transition? Maybe it is just a ball of energy?

                  How much rest mass would we require to create a gravity large enough to influence things 50 light years radial distant? Surely that sum has been done. How many kgs of sun mass can create such gravity? We know our suns kgs and we know its gravity range approximately.

                9. Black holes/ Lightening fireballs & vortices

                  HMS Montague ( Experiences of Lightning fire ball at sea )

                  One particularly large example was reported “on the authority of Dr. Gregory” in 1749:

                  Admiral Chambers on board the Montague, 4 November 1749, was taking an observation just before noon…he observed a large ball of blue fire about three miles distant from them. They immediately lowered their topsails, but it came up so fast upon them, that, before they could raise the main tack, they observed the ball rise almost perpendicularly, and not above forty or fifty yards from the main chains when it went off with an explosion, as great as if a hundred cannons had been discharged at the same time, leaving behind it a strong sulphurous smell. By this explosion the main top-mast was shattered into pieces and the main mast went down to the keel. Five men were knocked down and one of them much bruised. Just before the explosion, the ball seemed to be the size of a large mill-stone. [9]

                  The strong smell presumably was Ozone? Please consider the note and illustration below:

                  http://www.bridgetopeaceproject.com/images_global/AWFNfix/AWFN4CsEperE/AWFN4img002.jpg

                  Numerous incidents of lightning balls suggest that maybe they could exist in the above form?? Then do you think this type

                  of condition could be created within the centre of the Black Hole? As with smoke ring dynamics the velocity of rotation

                  appears to make the ring ( tighter – smaller ) in its cross section. Then loses its small dimension as it loses energy. If this

                  phenomena is apparent to the mass converted to energy in a post supernova black hole centre – the faster it rotates the smaller it becomes if I am not mistaken? It has poles which would fit the ejection of Gamma, and the radial perpendicular kinetic’s could they not be producing the necessary extreme gravity effects? To reach across radial 50 light years? ‘A super compressed energy field vortex”? Anyone here know the math to calculate this – starting with what are the energy conditions necessary to obtain the above gravity ? – then work back to the vortex – Any thoughts please?

                10. Ah, but the professor states otherwise, so it is likely that things have moved on and that things like gravastars and dark-energy-stars are now dismissed as nonsense or outdated models. This is why I am very interested in finding out more, no matter what I imagine, the universe always finds something better to come up with.

                11. Black Hole Construction

                  ‘Large Amalgam Atom’

                  Could we consider this? Nothing more than a large atom! Where all the atomic components in the sacrificial sun are separated in the closing dynamic moments of the supernova collapse. The nuclei’s simply form the large real mass amalgam and the electrons like bees simply have no choice but to remain captive? Once separated the space the mass amalgam will require will be tiny. I have copyrighted this just in case this has not been considered historically. IF, such a scenario is in the realms of possibility what are you comments regarding the quantum of resultant gravity. Fission has not occurred because the individual nuclei’s have remained intact they have just been separated from their electrons. Nuclei fusion has occurred – hence a White Dwarf??> black hole….

                  The chemical state of this condition similar to an oxidizing gas. But unfortunately is unable to become stable in the chemical sense.

                  If you are not too busy I would be most grateful for the shortest reply. Respectfully ewj.

                12. I am not sure that would work; when density becomes high enough, and far before a black hole forms (Think neutron stars.) any electrons in the mass become so high energy that combination with protons becomes favorable and the mass is reduced to neutrons. (With perhaps a smattering of unconverted protons and suchlike hiding away.) I have heard neutron stars described as giant atomic nuclei, but I am not sure how accurate that comment is.

                13. If such a thing be possible the end mass presumably would be the same as the precursor sacrificial sun? – just that all the mass carrying particles be pressed together as a uniform homogenous lump? Regarding the kinetic energy and potential energy of this end state ( energy of the body ) could such a monster ‘by ratio comparison’ have more energy than a happy little single hydrogen or helium atom?

                  Yes I follow your insight, but are neutron stars becoming black holes? Are black holes in the process of slow development or do they just occur when the precursor conditions are right. Hence some suns will make one due their starting mass and spin etc? Also these sacrificial mass materials where presumably of hydrogen or helium which in their own right have their own atomic characteristics. But should this imagined monster be possible then, presumably it would add to the periodic table as a new atom in its own table – with its own unique qualities ( apart from incredible size )?

                  If the unimaginable compression does occur ( scientific original thinking ), energetic as the electrons maybe but simple not able to recombine with a proton – as there is no single proton to recombine with – just one big one?…

                14. Electrons would be able to combine even with a mass of proton like you suggest. (I believe that such matter, composed of rotons and neutrons squashed so tightly together is called ‘Quark–gluon plasma’, and is only theoretical.) The electrons would vanish, but the quark–gluon plasma would remain.

                  All neutron stars could in theory be isotopes of ‘element 0’, the neutron, which is included (usually by physicists rather than chemists) in some periodic tables, but since it is a far different phase of matter from ordinary atoms and may be better called neutronium or neutron-degenerate matter.

                15. This isn’t entirely right.

                  First of all, quark-gluon plasma has been observed and studied for about a decade, at RHIC and at the LHC. It’s not theoretical.

                  Second, you should not think of a quark-(antiquark)-gluon plasma as protons and neutrons squashed together. It’s what you get when you squash the protons and neutrons together so tightly, and heat them up so hot, that the protons and neutrons cease to exist. You can create such a plasma in collision of two very-fast-moving nuclei of atoms that are far up the periodic table.

                  If you squash the protons and neutrons sufficiently but you keep them cold, you will perhaps get something else — a superconductor-like material made of quarks, perhaps.

                16. Many thanks for your many replies correcting these issues; I am amazed you have the patience to sift through everything here and provide accurate information. It is a true mark of dedication.

                17. When a star collapses in a supernova, it may form a neutron star. If the original star is sufficiently heavy, that neutron star may be itself unstable to collapsing further, and nothing then stops it from shrinking down until gravitational fields are so strong that they form a black hole. A black hole is not made out of matter (though matter may be used to create it); it is best thought of as created by matter but made predominantly from gravitational fields.

                  Black holes are really not like ordinary particles, and really not like ordinary atoms, though they share (and not in a shallow way) some basic characteristics of both. You can’t put them in any existing table of particles or the periodic table and have that make any sense. They really deserve, and require, their own discussion.

                18. Yes thanks – I thought my imaginings where a bit simplistic. So we can think of it as some form of spherical plasma? Made up of its special materials and hence characteristics. However, the external information it behaves typically with known phenomena?

                19. You have to understand black holes in stages. One of the trickiest things about black holes is that time is warped there. So to say what it “IS” requires you state who is trying to figure this out: someone who is falling in to the black hole will have a completely different experience to someone who is outside and maintaining a fixed distance from it (either by being in orbit or by firing a rocket to keep from falling in.) And you also have to think about what experiments you can actually do to find out what this thing “IS”.

                  There is no “IS” without a measurement. You have to talk about what you can measure. And that then gets complicated. There have been plenty of discussions going on about the right way to think about black holes during the four decades since Hawking showed that, left to their own devices, they evaporate away. And improvements in how to think about them are still likely to occur.

                  But they are not ordinary things that you can think of as made from materials. That’s just the wrong intuition. The problem is that the right intuition is quite difficult to convey — it’s a bit of a Hydra, too, because different observers see things differently, so you need to have multiple intuitions and an understanding of how they are related — and I am not sure I’m expert enough to be the one to teach you. Maybe Susskind (who was one of my main teachers on the subject) is the best for this; I’m sure he’s thought about how to convey these ideas to the public much more than I have.

                20. The Greek historian Herodotus made a fundamental observation about Egypt when he said, “Egypt is the gift of the Nile”.

                  In fact, the Nile not only shaped the land, but it also shaped the mind of the Ancient Egyptians.

                  -They considered the Nile as a force that could both create life and destroy it. During the inundation period all the cultivable land was under water, but at the same time inundation brings with it the potential for a new life: a fertilized irrigated soil ready for the sowing of corps.! Therefore, the coexistence of opposites was a core belief in the ancient Egyptians mindset. They were not that stupid as we have Euclid of Alexandra and Pythagoras to rely on –just to mention 2 – blending Greece with Egypt here – Euclid born in Greece as you know. Understanding things, is evolutionary small steps and we cannot forget these guys minds – as they had Great Ideas and Vision.

                  In modern terms, this is called the ‘Law of Duality’. It simply states that, “There Are Two Sides to Everything”.

                  In reality, everything exists in duality ( now we are discussing entanglement !!!). Understanding a black hole ( trying ) may mean not being tied to one concept of time and search and a solution to fit it? If a duality of time exists they cannot exist in the same place ( presumably) otherwise they would cause unimaginable physical confusion.

                  So let them exist in their own respective dimensions ( disliking that work – also spacetime ). But as you rightly say physics is based on tangiable measurement. So one must ask what is the tangiable measurement of string theory? ‘With non’ but maintains theoretical credence?

                  I was very impressed that you had Susskind as a tutor – and I follow his ongoing discord with Hawking. I would love to spend just 5 minutes with him and ask him the same question. If only I could find a way of presenting it to him. “ Can a duality of time exist?” and if so in a separate dimension where time is symmetrical. Which keeps both Newton and Einstein the best of friends. Respectfully

                21. Could he be persuaded to perhaps pen something on the subject? The entire area sounds incredibly enlightening and informative.

                22. “One of the trickiest things about black holes is that time is warped there”.

                  Presumably in the sense that both parties have different relative velocities? The one who is half in it could be near the speed of light and cannot see his legs even if they were still attached to his body – or substantially longer than they were historically thanks to gravity – or actually noticeably shorter according to ‘R’ rules. So one of the observers would have longer legs than normal whilst the other had alarmingly shorter ones. There’s a paradox! In reality the observer going in his legs are actually longer but the other observer is reporting they are shorter! comically

                  Kudzu commented: “Could he be persuaded to perhaps pen something on the subject? The entire area sounds incredibly enlightening and informative.”

                23. One of the problems is gravity warps time as well as space. If I throw something into a black hole, from its point of view it falls in, but from my point of view as it approaches the black hole it seems to slow down (and redshift) Indeed the light it emits at the event horizon should be ‘stuck there’; forever not far in enough to fall into the black hole, but not far out enough to escape. So what is actually happening to the object? it can be tricky to tell.

                24. “but from my point of view as it approaches the black hole it seems to slow down (and redshift)”

                  Could it be that this red shift is not a reflection of its slowing down – but only that the light is slowing down. So the action conflicts the occurance?

                25. For a long time I thought that too, that the object merely *appears* to be slowing down. However we know gravity warps time, that time on the ground here in earth’s gravitational field passes differently from someone floating far away in space. So the object’s time does slow down from an outside viewpoint. This is ‘gravitational time dilation’ and it has been measured here on earth. See http://en.wikipedia.org/wiki/Gravitational_time_dilation for a layman’s overview.

                26. Interesting, which means you have PULLED me into thinking about gravity! So we have to think about Velocity & Gravity?……thanks for the link.

                27. Hi Matt – Happy New Year – i hope you had good one and enlightening time in Edinburgh.

                  Back to black holes – Question: can you imagine a black hole mechanism which exist as a poloid where the moving items are photons. The centre of the poloid the photons have a zero relative velocity ( in phase with each other ). In doing so the centre will have a substantial hypothetical density ( the energy and velocity locally compressed in that zone ). The outer ring the photons become rarified and separated. The substantial gravity caused by the centre puts the entire system under stress because it want to every shorten the outer diameter. It unable to reduce the diameter because the photons are already at value ‘C’ and to move closer to the centre they would have to move faster! This effect would cause the tension. This tension is then shed by way of producing huge gravity. In the same law as a proton generates mass to dump the additional energy which we impart to make it go faster. Does this make any sense?? I can send a pic or you can view this idea on you tube where i have presented although not very professionally. regards edward

                28. What you have described is a very interesting situation. If a photon was stably orbiting a large mass, what would happen to it? objects with a ‘rest mass’ emit gravitational waves, losing energy, moving closer to the center of mass and speeding up. But a photon is a massless particle with a fixed speed.

                  It would seem then that it would simply be unable to lose energy and fall towards the massive object; it would be in its lowest energy state. (But something tells me that this is likely very wrong.)

                  Of course a photon that gains energy becomes higher frequency; and if the photons in your hypothetical object became energetic enough they would start randomly forming particle-antiparticle pairs (I believe this would start to become a factor when the total energy was enough to produce positron-antipositron pairs.) These particles would then fall towards the massive object.

                  I think this would make your supposed object unstable. Do you have the link to your youtube video? It sounds fascinating.

                29. http: // http://www.youtube.com/watch?v=dD6pRrz80Wo

                  Hi Kudzu – here it is the Gravity Engine! Really sorry about its quality – first time i used movie maker and actually damages my pc and almost ceases to operate! Then almost impossible to edit it hence the failed proof reading and spelling mistakes ( so embarrassingly amateur ). Also rather patronising in its style which makes me recoil with further embarrassment.

                  I am going to rework it on another pc which can cope with this simple task unlike my 19C model!

                  In essence all I am proposing: It is a potential poloid ( like a smoke ring ). The gravity at the epicentre must cause the outer ring to contract towards it’s own epicentre. But if this was to work the particles would have to rotate faster in order to reduce its outer ring diameter. They cannot do that as they are already at value ‘c’. The entire mechanism experiences a constant internal tension. This tension achieves constant equilibrium by ridding that tension by shedding it as gravity.

                  Similarly, a proton becomes more massive ( even, if not is size ) if we impart more energy on it to move faster. It cannot, so one ( me ) thinks that the proton is under pressure. The pressure of the external energy imparted on it obliges it under the rules of nature to convert ‘get rid’ of that energy by the conversion into additional mass.

                  In the case of this hypothetical intuitive gravity engine ( black hole ). It dumps the tension by producing a massive gravity. The gravity is evidence and reflects the internal stress. Not just getting denser and denser like a super form of electron-less lead like material. It is an electrical device not far removed from a Klystron for example, which transmits microwave – but in the case of a gravity engine it is transmitting gravity.

                  And similar to a diode at ‘switch on’ where the currently flow is momentarily infinite. Only in this case the ‘gravity’ produced in virtually infinite.

                  Idiotically, one can think of it as a bird in a cage. Very often they shake and flutter to remove their stress. In doing so they become heavier momentarily during that activity! Our black hole is doing this constantly. I hope i have explained myself adequately. I eagerly anticipate your advice and critique -if for one moment you think this is technically feasible then perhaps we can turn our minds to the participation of Time in this engine – thanks.

                30. http://www.youtube.com/watch?v=dD6pRrz80Wo

                  Hi Kudzu – here it is the Gravity Engine! Really sorry about its quality – first time i used movie maker and actually damages my pc and almost ceases to operate! Then almost impossible to edit it hence the failed proof reading and spelling mistakes ( so embarrassingly amateur ). Also rather patronising in its style which makes me recoil with further embarrassment.

                  I am going to rework it on another pc which can cope with this simple task unlike my 19C model!

                  In essence all I am proposing: It is a potential poloid ( like a smoke ring ). The gravity at the epicenter must cause the outer ring to contract towards it’s own epicentre. But if this was to work the particles would have to rotate faster in order to reduce its outer ring diameter. They cannot do that as they are already at value ‘c’. The entire mechanism experiences a constant internal tension. This tension achieves constant equilibrium by ridding that tension by shedding it as gravity.

                  Similarly, a proton becomes more massive ( even, if not in size ) if we impart more energy on it to move faster. It cannot, so one ( me ) thinks that the proton is under pressure. The pressure of the external energy imparted on it obliges it under the rules of nature to convert ‘get rid’ of that energy by the conversion into additional mass.

                  In the case of this hypothetical intuitive gravity engine ( black hole ). It dumps the tension by producing a massive gravity. The gravity is evidence and reflects the internal stress. Not just getting denser and denser like a super form of electron-less lead like material. It is an electrical device not far removed from a Klystron for example, which transmits microwave – but in the case of a gravity engine it is transmitting gravity.

                  And similar to a diode at ‘switch on’ where the currently flow is momentarily infinite. Only in this case the ‘gravity’ produced is virtually infinite – constantly.

                  Idiotically, one can think of it as a bird in a cage. Very often they shake and flutter to remove their stress. In doing so they become heavier momentarily during that activity! Our black hole is doing this constantly. I hope i have explained myself adequately. I eagerly anticipate your advice and critique -if for one moment you think this is technically feasible then perhaps we can turn our minds to the participation of Time in this engine – thanks.

                31. The video was both interesting and informative. I have the following issues with your model:

                  Formation: One of the useful attributes of black holes as conventionally understood is that it’s (relatively) easy to understand how they form; being a singularity simply letting gravity bring things to a point ‘works’ (This is a simplification, but still.) Black holes can be produced in violent circumstances and the existing model is quite robust, how does a ring of massless particles form as a dense ball of matter collapses?

                  Stability: A number of points arise here; the first is your photon poloid; the particles within it are treading a very fine line, if they were slightly further in they would fall into the hole, further out they would escape. Just doing a bit of back of the envelope scribbling, your torus would be more like a very, very thin ring.

                  A ring (or poloid) of particles is not a stable situation except in very rare circumstances. This is why matter falls into black holes, it collides with itself, loses energy and falls inwards (Rather than relying on losing gravitational energy.) In our own solar system the asteroid belt is very sparse because if it were any denser, asteroids would collide and be removed. It is like the viscosity of a fliud when you stir it.

                  Photons don’t suffer from this, but have other effects that also make me question their ability to do this, notably they can scatter off each other. If the photons in the ring were all in phase, like a laser beam shot around the hole things are better, but this raises another question of how this ring formed.

                  Properties and interactions of black holes: This is somewhat related to stability. Black holes have a number of properties, they can have electric or color charge, velocity and can rotate among other thing. (especially rotation, it is likely that at their formation black holes will be rotating massively fast.)

                  The question would be how your model handles these properties, where they are stored. (This is not straightforward to explain with traditional black holes of course.) Since your hole has ‘moving parts’ as it were and an internal structure this raises questions.

                  Black holes can also merge, and of course they absorb mass and energy. Most black holes will be continuously absorbing energy from the CMB. Your model seems only to be stable while the black hole is of a fixed mass, matter falling in would alter the ‘primary gravity’ and cause the ring to collapse would it not? And I am not sure what would happen when two holes merged.

                  The biggest issue I see is that the gravitational field produced by the torus would not be spherical, but toroidal. From a distance it would appear relatively spherical, but this would become less accurate as an observer approached, or the hole was larger. At the very center of the hole there is only the ‘primary gravity’ since the torus will be pulling anything there towards it in all directions, cancelling itself out (Like how you would be weightless at the earth’s center.) It should be possible to shoot through the poles of the hole, pass through the center and emerge out the other side.

                  There is also the question of what keeps the ‘primary gravity’ in place; if the torus is producing more gravity than G1 (That is it has more energy.) then the situation of G1’s mass becomes unstable. Imagine a planet with two moons, on opposite sides of it, in the same orbit. If the planet is large, (o O o ) this is quite stable, they will both orbit roughly around the planet. But as the planet’s mass increases this istuation becomes increasingly unstable. A small moon between two large planets (O o O) is stable, but the slightest deviation in position causes it to move towards one or the other planets. It is like a pencil balanced on its tip. (See also http://en.wikipedia.org/wiki/Lagrangian_point where it is noted some points are unstable while others are stable.)

                  G1 would find itself at the mercy of G2, the slightest perturbation would knock it towards part of the torus, and it would then merge with it. Your model will need some way of preventing this.

                  All in all an interesting theory that will hopefully gain a sound mathematical foundation.

                32. Hi Kudzu – i hope you have had chance to look at my ‘you tube toy’! The particles ( whatever name ) are not orbiting in the normal sense of an atom. ( although i suppose they could be ). My engine is a separate system of unity particles ( meaning same specie ) – but unable to orbit a mass ( even if there is one ) in the conventional sense.

                  This toy system could just comprise of particles – and the dense bit ( where the mass should be ) is constructed from themselves in the epicentre – creating a virtual mass. So they think there is mass but not! It is just themselves causing a dense centre whilst they visit it – and for the very short period of time – meaning value ‘c’.

                  In the short time they visit the epicentre they are all at the same velocity and time relative the same = 0.

                  So they all contribute to forming ( construction of a mass! of the same particle type. e.g. A huge massless photon?? Then exit the centre – then they revert back to being tiny things again ( unitary photons – or whatever ) in the conventional sense.( Or other candidate particle ).

                  Or it could be a be a pure energy flux ( definition here ) which is rotating & momentarily forms ( so called dense matter – whilst in the centre. They exit – the matter still exists because behind them is a constant stream of indian particles. Then moments later they find themselves back in the centre again which maintains the status of the virtual matter ( mass ) which is not stable. ( But is because of the feed & return rate of particles causing it in the first place.

                  Any new introduction of material ( matter, mass, energy, whatever ) causes a momentary loss of equilibrium to the entire toy system ).

                  It cannot tolerate this so it dumps what is can by conversion into another state of matter/energy ( e.g. x ray/gamma etc ). But the balance of matter ingested is another issue. If it consumed endlessly it must become endlessly bigger! – which on the face of it simply does not happen. ( And not thinking of Sir Hawkings version because any loss of energy would be visible to us ). Even an invisible glimmer! (so called thermal entropy )

                  So where does the degenerate matter go?

                  One can think of the 1922 accelerator function to come up with that answer?

                  The LHC shows us great images of particles spinning off after the collision but what happens to them? They cease to be particles? – and their energy absorbed by the detector and increase in the charge on the equipment? If that is the case then I assume they are measuring it?

                  What happens to a lonely supposed Higgs particle once it has been separated from the proton? If it has so much energy potential should we not encounter some other event ?

                  If there is no background increase in the charge of the detector = too small to detect or no increase in charge? ( i hope i have used the word charge correctly? ( 125 gev^2 increase in the earth leakages of mass converted into electrical energy? i imagine would be detectable on such an elaborate piece of equipment.

                  and so on…….i look forward to your reply with interest – thanks & v. kind regards.

                33. Right. Your response answers a number of questions, and poses new ones.

                  If I understand you correctly, the particles in the torus are not orbiting it in the traditional sense, they are more delocalized, alike an electron around a hydrogen atom, which, though it spends a lot of time in a spherical shell, also spends some of its time deeper inside the atom, even inside the nucleus. When these particles are in the center of the hole they generate an effect like a mass, this produces G1, which then starts a feedback loop affecting the particles when they are in the torus.

                  The biggest objection I have tot his is I cannot think of how this situation would arise and what particles would be involved. The second biggest would be that the uncertainty principle does not allow something to have a precise velocity of 0, especially when it’s in a small area.

                  I still cannot see how this situation would arise as a mass collapses under its own gravity, at some point the particles making up that mass would have to convert into the kind you describe making up the hole, while preserving those particles information. (The particles you describe making up the hole would have to be able to store electric and color charge as well as the velocity and angular momentum of the mass and other properties.)

                  Your argument for the conversion of infalling matter is interesting, but raises some serious problems. In this case a black hole will always emit energy as it absorbs mass. That is, it would not strictly be black or be a black body. Classical black holes do not, the energy they ‘release’ is actually released by matter that has not yet fallen into the black hole, but is orbiting outside it, colliding with itself and losing energy. This is much like friction, the black hole is forcing mass together, squeezing it and making it rub against itself. It is a very rough particle accelerator. Close to the hole the mass is so hot it emits X and gamma rays. The jets emitted by many holes are also not emitted by the hole itself, but are matter that cannot enter the hole as there is already too much matter there. (Though precisely how they work is debatable.)

                  By contrast if I am understanding you correctly, your holes convert a percentage of any mass that falls into them directly into X rays and gamma rays. This means that a black hole sitting alone in space in the CMB will be absorbing that energy and converting it from high entropy microwave photons to low entropy and gamma rays. Indeed it should be possible to fire a beam of arbitrarily low energy at a hole and create a beam of high energy (but lower density) from the poles of the hole.

                  According to the laws of thermodynamics this imposes some constraints. The simplest solution would be for the black hole’s entropy to increase to balance this, but as it would eventually evaporate, this imposes some constraints on just how it can do this.

                  I am also not sure what keeps the poloid stable; the particles can migrate to the center, but then return to the poloid, but what prevents them from forming another poloid at right angles to the original? Shouldn’t they be able to form a spherical shell around the center? (Of course this would depend on the exact behavior of the particles, involved, do you have a quantitative description of this?)

                  The particles produced in collisions in the LHC decay and lose energy, eventually transforming into ‘stable’ particles, photons, neutrinos electrons, protons and the like, as well as energy to the equipment (heat.) This is a well understood process. There are indeed multiple events, this is why particles produce jets and why it can be so hard to detect the particle you want.

                  This is exemplified by cosmic rays, you will want to read this page: http://en.wikipedia.org/wiki/Air_shower_%28physics%29 A single cosmic ray, a highly accelerated particle hits a detector (our atmosphere) unleashing not one simple collision, but a cascade of particles which hit more particles and so on until all the energy is spent. The LHC detector is far more dense than the air, so the cascade is somewhat smaller and more contained, but the principle is the same. (Though cosmic ray particles can be very much more energetic than LHC ones.)

                  Your use of charge is not correct however. Energy is not electric charge, which is what we often associate with electricity. When energy dissipates it usually does so as motion, or heat as noted above. Think of rubbing your hands together, stopping your fast moving hands, releasing heat energy.

                  Also though 125Gev sounds like a lot of energy and it is, for a particle, compared to our scale, it is tiny. It is about 0.00000002 Joules. For comparison you walking down the street have kinetic energy of about 35 Joules (70Kg at 1 m/s) On the atomic scale the Higgs energy is mighty, on ours, hardly noticeable.

                34. Fantastic reply – thanks very much. I get the impression this toy black hole is part functional from your reply on the basis ( unless you are just being generous of spirit ) not to dismiss it 100% outright. You have picked up the scheme and obviously understand my thinking on its proposed operation. Maybe I could redraw more accurately the proposed flow and we just revisit ideas relating to its stability?

                  All the other refs you presented are most useful thanks – and thanks too for such a detailed reply – which I will revert back to later today – inc the LHC and loss/absorption of energy. Kind regards

                35. “The biggest objection I have to his is I cannot think of how this situation would arise and what particles would be involved. The second biggest would be that the uncertainty principle does not allow something to have a precise velocity of 0, especially when it’s in a small area.”

                  I am sure you are correct, however, sometimes laws get changed or better defined e.g. Newton vs Einstein. Also, they actually do not have to have the 0 velocity as I stated ( although I prefer it ). The zero velocity is only relative to each other. E.g. they may be moving at 300,000kms or <C = ‘all of them’ and in parallel which of course will mean that relative to each other they have no velocity.

                  So if under this condition they are all squeezed up ( for a very short temporal time – or could be forever if time is zero ). It would be necessary to think of a further and new paradox here. How can something be “moving and undergo no changes”.

                  Anyway…if they are all squeezed up to form a dense mass which rests there constantly, but is constantly being refreshed with new particles ( ones which were there a second ago for instance…) then such a mass is both constant and variable. Are we obliged to think of particles in this construct? Why not just an energy flux without having to be defined as a particle? E.g. The entire system is a field which structurally looks like a poloid and the energy is being manipulated in a way I have described but in reality nothing is actually moving!

                  “Your argument for the conversion of infalling matter is interesting, but raises some serious problems. In this case a black hole will always emit energy as it absorbs mass. That is, it would not strictly be black or be a black body. Classical black holes do not, the energy they ‘release’ is actually released by matter that has not yet fallen into the black hole, but is orbiting outside it, colliding with itself and losing energy. This is much like friction, the black hole is forcing mass together, squeezing it and making it rub against itself. It is a very rough particle accelerator. Close to the hole the mass is so hot it emits X and gamma rays. The jets emitted by many holes are also not emitted by the hole itself, but are matter that cannot enter the hole as there is already too much matter there. (Though precisely how they work is debatable.)”

                  Yes I understand this. I am a bit confused on this point though. I know that the X and G are emitted from exterior material and activities, so what actually enters the BH? Just Heat? and nothing solid ( particle )?

                  “I am also not sure what keeps the poloid stable; the particles can migrate to the center, but then return to the poloid, but what prevents them from forming another poloid at right angles to the original? Shouldn’t they be able to form a spherical shell around the center? (Of course this would depend on the exact behavior of the particles, involved, do you have a quantitative description of this?)”

                  In my mind it creates its own stability by particles ( energy ) not being able to transit faster than C ( this in a way is what maintains the pendulum swing ) . They want to – but cannot and hence resists change or decomposition.

                  “I still cannot see how this situation would arise as a mass collapses under its own gravity, at some point the particles making up that mass would have to convert into the kind you describe making up the hole, while preserving those particles information. (The particles you describe making up the hole would have to be able to store electric and color charge as well as the velocity and angular momentum of the mass and other properties.)”

                  I don’t know how to respond this insight, other than to say in brief, that if a solid sphere was collapsing at huge velocity, albeit somewhat rarified with a very thin dispersed volumetric density – I suppose a much cleverer person than I with a math imaging modeling resource may be able to find such a mechanism. To create a smoke ring has to start with an aperture of some kind is my understanding.

                36. When it comes to matter entering a black hole, you can think of it like a meteor hitting the earth. As it gets closer to earth it moves faster and starts to heat up in our atmosphere. The light escapes earth, but the meteor itself is absorbed. The situation with black holes is similar, but more severe.

                  A large amount of energy escapes the black hole, but remember the C^2 term in the energy-mass equation. Only a small portion of the energy falling into the hole (The gravitational potential energy.) has a chance of escaping as photons and not all of this manages to escape, but this is more than enough to make active black holes exceedingly bright objects.

                  I can understand the stability of the poloid if it were a perfectly still and symmetric object, but this is like a pencil balanced on its point, hard to achieve and not something you’d expect to see in nature.

                  for one thing, black holes can spin, quite fast. A poloid like this cannot have any spin since that would mean changing the speed of the particles within it, which as you note cannot be done. And there are also asymmetries; if another mass approached the black hole it would distort the gravitational landscape (as all mass (Well, energy) does.) I think this would mean the poloid would have to distort, but I do not know enough of the relevant physics to be able to say for sure.

                  The formation of smoke rings is shown here: http://tealco.net/blasterphysics.gif.jpg Basically a fluid moves through an aperture and is disturbed, curling around on itself. This happens all the time, but if the aperture is nice and symmetrical you get not a less stable ‘smoke sausage’ or more random eddy, but a ring.

                  It differes in two important respects from your poloid. Firstly the particles are moving not around the larger diameter of the ring but around the smaller diameter. (In your video not left-right but up-down.) Secondly, the production of said ring is a definitely asymmetric process; the flow is in one direction only.

                  However now I wonder, if the angular momentum of the infalling mass was conserved, could it be converted entirely to your particles? In this case a nonrotating black hole could not exist; as a star collapsed it would spin faster and faster, converting into massless particles that rotate at the speed of light. I am not at all confident in this model however, especially since I believe it predicts that objects without the correct angular momentum would collapse into something else.

                37. “but what prevents them from forming another poloid at right angles to the original?”

                  I can only surmise that it is because the poloid is organized and stable, the unidirectional flow of the material/energy cannot be easily shifted once initiated. The G1 gravity has a pattern of behavior with no other force or influence on it to vary it.

                  You also mentioned collisions of black holes – that indeed would throw up some interesting ideas! The 2 poloids are stable and at equilibrium. We introduce another one – so how could they possible combine? And after the collision have only 1 without mutual destruction? The poloids may in fact be tiny things the size of a tea cup! Until someone very clever and experience with modeling tools how could we put a size to such a thing and realize how to quantize gravity it is producing an scale it. It may the size of an atom, or even smaller, or the size of a massive sun? Surely someone who has in depth knowledge of photonic behavior for instance could create a mathematical hypothetical case. In this case determine how much gravity it is producing. Then with that information put a scale on it.

                  Maybe when 2 BH collide there is only one winner and the smaller one has its equilibrium upset and ripped apart and the 1D window closes?

                38. Well, on the size issue, the poloid, being made of light speed particles would have to be exactly as big as the event horizon, any smaller and it would collapse into a singularity. This is why I wonder about what would stop a ‘poloid’ forming like a spherical shell. However, angular momentum would stop a stable poloid from changing, I was foolish not to have noticed this. (The poloid would not be able to change the direction it spins any more than the earth would be able to suddenly start spinning north-south instead of east-west.)

                  I do not think it is possible to have ‘one winner’ when black holes merge, since if they are constructed the same, they will both affect each other. (And there’s always the case of two identical black holes meeting.) The challenge would be how the ordered poloids would combine to produce a third, larger, ordered poloid. (Traditional black holes can merge like drops of water roughly speaking, lacking any complex internal structure.)

                39. Manubo: 1. If we are currently becoming flatter, surely a good mathematician can calculate 2. When we will be flat. 3. He will also be able to inform us when that state would occur. 4 Based on our current rate of change. 5. Then with that number we would know how far we are away from the manubo centre 6. Then all other kinds of information would be possible such as, the manubo diameter, and with that information how flat the 2D ribbons are going to be?? – We may be only a few atoms thick! – or less!

                40. Two objections arise then to this. A flat universe is one with no energy in it to warp spacetime. (The energy of space itself doesn’t count, and I believe it could also contain as much energy as you want, provided that energy was perfectly evenly distributed.)

                  This means our universe, which has energy in it as particles, would have to be infinitely large to achieve this state. So no matter how fast it expands, no finite, or even increasing expansion rate will ever achieve this (Though it will get close.)

                  We know the universe’s rate of expansion is increasing and that, if nothing changes, it will become immense, tearing apart even atoms in a ‘big rip’ I don’t think this will give us a flat universe but it’s an interesting way to go.

                41. “We know the universe’s rate of expansion is increasing and that, if nothing changes, it will become immense, tearing apart even atoms in a ‘big rip’ I don’t think this will give us a flat universe but it’s an interesting way to go.”

                  You stated earlier that the matter is becoming diluted – can you explain this again please……..

                42. The universe, as far as we are aware, contains a finite amount of energy. (This is at least true for the observable universe.) We can call this quantity x. Energy is what causes the curvature of spacetime, which means that to find the average curvature of spacetime in the universe we can just divide the total amount of energy in the universe by the total volume of the universe.

                  If the universe were static then this ratio would be constant. (And the universe would be unstable, with gravity pulling everything together.) However space is expanding, this doesn’t mean just that an existing volume of space is stretching, but that the universe is getting ‘new’ space; (If a given unit of space were just stretching either we wouldn’t notice or things would get very weird.)

                  This means that in any given volume of space there is less and less matter as time passes. Galaxies are all (mostly) moving away from each other, you need a bigger and bigger volume of space to contain them all. Thus the average energy density of the universe is falling, the universe is becoming more empty, more dilute.

                  If we run time backwards we see a universe where energy density increases, where there is less and less space, until eventually we end up with a universe of no space and infinite density, the big bang. (Of course something may happen early on in the universe so it didn’t start out like that, it just appears to, but the model works rather well up to very close to the universe’s birth.)

                43. “If we run time backwards we see a universe where energy density increases, where there is less and less space, until eventually we end up with a universe of no space and infinite density, the big bang. (Of course something may happen early on in the universe so it didn’t start out like that, it just appears to, but the model works rather well up to very close to the universe’s birth.)”

                  Yes but in my thinking we run temporal time backwards – the event & historic temporal time. Maybe we have 2 conditions of enlargement: the natural and original separation of the material + the production of a space which existed before hand which was already expanding rather rapidly using our ‘c’ yardstick. And maybe because we only consider the regular spacetime where material ( energy ) exists as a visible volume.

                  Thanks – I lost track on this for a moment. Your use of the term New Space directs me back to my original intuitive meaning it difficult to be separated from this idea and function of the primary framework as I referenced in the book. I am going to try and make contact with some people at CEA fundamental research and see if I can get some opinion regards the output field dynamics ( assuming there will be some ) , ref the 2 fields of a TBH + central densificaton. To see if any of this makes any sense.

                  Is the notion of ‘New Space production’ a regular scientific realization or is it ignored as being hypothetical – or not even thought of? I know Newton thought of it as the absolute space – which Einstein discredited. Thanks

                  Yes I agree with your comments ref the prof experiment – however from his arm muscles point of view there is less gravity ( although I totally agree with your statement ). His arms are fooled into thinking there is less gravity. The same value of gravity exists but they have to do less work = less energy required. So if one ( presumably ) was to ignore the ability of the dynamic forces in this experiment and consider only the change in condition from his arm point of view the gravity has diminished. I also understand that in order to experience this so called virtual loss of gravity his arm has to remain part of the dynamic system and hold the end of the rod.

                44. The creation of new space is a well accepted scientific fact, though not usually worded in that manner. Most of the time you will hear of the expansion of space because this is nice an intuitive idea. But if a given unit of space were getting larger, stretching outwards in all directions as it were you would expect the universe to become more ‘grainy’ like a picture when you zoom in too far.

                  Of course my wording could be taken to mean that there are totally new ‘cubes’ of space are popping into existence, which is also a misleading way of looking at things. (Possibly it would be best to think of space growing like a shoot, getting longer without increasing in discrete steps.)

                45. “The creation of new space is a well accepted scientific fact, though not usually worded in that manner. Most of the time you will hear of the expansion of space because this is nice an intuitive idea. But if a given unit of space were getting larger, stretching outwards in all directions as it were you would expect the universe to become more ‘grainy’ like a picture when you zoom in too far.

                  Of course my wording could be taken to mean that there are totally new ‘cubes’ of space are popping into existence, which is also a misleading way of looking at things. (Possibly it would be best to think of space growing like a shoot, getting longer without increasing in discrete steps.)”

                  New Space = Expansion – I am sorry I don’t see this as the same engine. I take your point about grainyness – but does space have a resolution that we can currently see? ( if no New Space is being added then voids will occur between the pixels, or the grains get very rarified and what affect would this have upon Plank length ) ? My proposed intuition New Space is built upon the notion that it is being constantly added to – like a universal invisible expansive foam. ( each frame ‘pixel’ is being duplicated – by 1D quadruplication of itself. It is literally getting bigger structurally, not just the existing space occupying a greater volume and getting thinner. This may be the cause for the matter being diluted as it is not being quadruplicated dimensionally in 3D and is subject to the thinning process, which is a separate space system ( 3D spacetime is not being built it is just getting thinner).

                  I was considering a tube presentation with Graham Green where he explained that if you could remove all matter. His space presentation in this case was represented by a matrix of static cubes. In my mind this is not correct. Sure it can be represented by cubes but they don’t just sit in space doing nothing ( other than vibrating and twisting and expanding etc ).

                  According to my intuition more box frames are constantly being added second for second ( like the expansive foam above ). The feed rate of new cubes are at the value C. ( So in each direction movement is possible ), Hence light has the ability to move. It is cannot transmit through 1 frame it can only move if new frames are being produced. If no new frames where being produced ‘nothing’ could move. We can only move into a space which is already moving. In this case where new frames are being added to constantly which provides any ability of movement. Photon, a common earth slug or flicker of a candle flame. I hope I have explained this notion clearly? I sent an email today to Prof Dijkgraaft secretary at Princeton but nobody will be drawn into this idea. Science is no different than the other professions it is a closed shop with rigid politics, and a rigid way of doing things. The accountant must produce a profit and loss and balance sheet these are his rules and confinement. Over hundreds of years this is the way of conducting accountancy – but there other ways to determine and weigh the current financial situation.

                46. Things get tricky here.

                  Some have postulated that the universe does have a graininess to it, that it is made out of discrete blocks of space or some sort of network of nodes or similar. At any rate this would be at scales far too small for us to currently detect. In the main space is treated as being infinitely divisible; no smallest unit or shortest distance.

                  However as scale gets smaller the random nature of… well everything becomes more and more evident. The smooth expansion of space becomes more choppy, fluctuations in various fields become more extreme and our picture of the universe begins to break down. Eventually you reach a scale where our notions of space and distance are unreliable. At this scale we don’t really know what happens since we have no good models to describe it. It is possible that ‘blocks’ of space appear ‘instantly’ between other ‘blocks’ but I think the default view would be that the expansion of space at that scale would be very jittery and random, but still smooth. We will have to wait for evidence and theory to provide some insight on such things.

                47. So can I fantasize regards my personal view and not be shot down in flames? In absence of a good accountants practice? And primary space is ( may ) be duplicating itself internally everywhere at the value C infinitesimal frame by infinitesimal frame, each one with an individual time value 0? Into which our matter and subsequent 3D is home?

                  I have no problem with being considered a crackpot or fringe scientist – I am too old to be bothered by such critique.

                48. Your can be pretty sure you won’t be ‘shot down’ as long as you don’t overstep your boundaries and stat claiming you are right and others are wrong without any evidence. As I have said before though, having a sound mathematical basis for your ideas will go a long way to you being taken seriously.

                49. Thanks very much Kudzu – I appreciate your wisdom. When describing things I must also refrain from pulling down existing ideas to promote my own view, which would only promote the idea that I am in the crackpot community! You are completely correct.

                50. I think the biggest problem you face with your ideas is the lack of a mathematical basis. While science can be bound by tradition and method, scientists are usually the first to question everything and usually on the lookout for a new theory.

                  The problem is that it is easy to invent a concept but much harder to craft a theory. No scientist will investigate an idea unless it comes with some solid conditions where it is right or wrong. There are literally millions of way in which your theory could be utterly incorrect on the most basic level, but unless you are quite specific there is no way of telling which ones do or don’t apply.

                  It is like inventing, you can make all the drawings and advertisements you want, but nobody will take you seriously until you have a basic model that proves that your device works in reality. People will only invest as much energy into your ideas as they think they are worth, and if they have no proof that your theory won’t collapse as soon as you try and write it up then they are simply not going to value it.

                  There’s also the problem that for every email from you they get quite literally hundreds along the lines of timecube: http://www.timecube.com/ ; poorly worded nonsense that confidently states that they are wrong about everything and that obvious things (Like gravity) are wrong or made up. Your ideas can easily be confused with such things, and of course they are not going to want to respond.

                51. YES! You are absolutely spot on correct – especially using the invention parallel – years of my life experience has determined that. Simply telling people that you have invented the best ever washing machine with words is not worth a penny. In this case though I cannot see how I could invent a prototype!

                  Then I have to think how to covert in into tangiable mathematics. And even then will the mathematics be believed? As it is based on intuitive assumptions and difficult to substantiate naturally. Unless I reference things like Time dilation, atomic clocks Horizon problem etc etc. Unlike String Theory which seems to be going along quite nicely on an intuition and putting mathematics before the science, and inventing nature to fit it? That is the acrimony.

                  My notions will never be taken seriously – I realize that. Then even if they were it would just be another person’s view of the universe which competes with all the million others. And who at the end of the day will be correct will never know as most likely we will never understand the universe – just live out our lives and go mad discussing how this clock works!

                  The people with telescopes, satellites and colliders they are in the front line ( although I have serious doubts about the latter, the rest is conjecture – including in my mind String Theory.

                52. Ref time cube – yes I see what you mean a form of obsession and madness! The main purpose for me to visit this site was to receive feedback for these notions. Feedback from people who are specialists, you have helped a great deal and I thank you for that. Taking your advise further if I am to take myself seriously then as you say I have to find a way to convert them into a tangiable theory. Which I suppose is where I should direct my energy, and make a self searching decision regards am I a candidate for the time cube community, having spent all that time on the so called theory?

                  Not wanting to waste life time, I think I will just pen them in a clearest style I can find and simply post like the rest of the lay global public onto the internet, maybe the lower class of physics journals. Then get on with the next obsession!

                53. I wish you the best of luck with your efforts; it’s rare but science is littered with stories of those who built an idea from nothing that proved to be brilliantly correct. (Most recently a 15 year old’s programming work ended up on the cover of Nature.)

                54. Yes that was incredible – fate seems to favour some people – and there are many of them – including Bill Gates, and the French/American man who invented Ebay in a garage, and be born a king and not hungry in Ethiopia. Yes, Ideas are very powerful things in the realm of mankind & Intuition provides us with beliefs that we cannot justify in every case.

                55. Thankyou, I am aware of this paper, having kept a lazy eye on negative temperatures since I first heard of them several years ago. What always interested me was that though these atoms are incredibly cold in the way we usually think of temperature, they will heat up any normal matter they touch, and in doing so reach absolute zero. And we’ve known about this for decades. Incredible.

                56. “they will heat up any normal matter they touch, and in doing so reach absolute zero. And we’ve known about this for decades. Incredible.”

                  Why? What is the process? Is this another physical mystery?

                57. As I explained in another reply, it’s just the way things with negative temperature work. Something at +1K is very low energy, it ‘wants’ to take in energy from other things and cool them down, make itself ‘hotter’ (higher energy). Something at -1K is the total opposite; very *high* energy it ‘wants’ to give energy from other things and warm them up, make itself ‘cooler’ (lower energy.) This page gives a basic overview of the concepts and equations involved: http://en.wikipedia.org/wiki/Negative_temperature

                58. That depends on the density and state of the matter involved. You may recall for example that the cosmic background radiation was constantly absorbed and re-emitted by the plasma in the early universe until the temperature dropped enough for atoms to form. Matter in stars will severely impede all photons while that in gas and dust clouds presents less of an obstacle (But still a significant one.)

                59. “constantly absorbed and re-emitted by the plasma”

                  ?? Interesting , so plasma shows lowest impedance to a transiting material? As for everything else to cold and solid?

                60. Then maybe there is no opportunity for anything solid in a black hole. It would simply act as a limiter/moderator, and seriously limit its potential. Like a lump of cold stuff in the Tokomak reactor? That would seem rather absurd and cause it to fail rather quickly>?

                61. I would have to agree. I am not aware of any common model of black holes that involve an internal structure in a black hole, let alone some form of matter. (Some have postulated ‘quark stars’, more compact than neutron stars: http://en.wikipedia.org/wiki/Quark_star )

                  The problem arises with ‘degeneracy pressure’; basically how much gravity degenerate matter can withstand. Each time one degeneracy pressure is overcome matter becomes unstable and is forced to assume a more compact (And in practice simpler) form. White dwarfs are made of nuclei and electrons, when electron degeneracy pressure fails they collapse to neutron stars which are mostly neutrons. When neutron degeneracy pressure fails they may become quark stars, made of quarks (and if preons exist as constituents of quarks, we may have preon stars.)

                  A black hole would be far beyond these and it’s likely that such matter would have ‘run out of options’ (A mass made purely of electrons that exceeds electron degeneracy pressure would not be able to form a neutron star and would collapse to a black hole having no other degeneracy pressure to utilize, ignoring of course the massive electric charge and other such quibbles.)

                62. Is this model going anywhere?

                  If the supernova squashes itself into a super dense thing. Its resultant gravity sufficient to pull in millions+ of suns then – surely it must destroy the meaning of what is solid in its personal sense. The solid form simply disassembles into a pure particle phase which then have to organize themselves differently as the starting solid ceases to exist? Those particles which cannot participate are degraded or expulsed.

                  In this case a series of ‘particle only poloids’ which have a common centre. There has to be only ONE master involved which causes a system symmetry in the process otherwise we would not have any BH’s. The master is the epicenter which they all subtend. They create their own local gravity. Because the entire system is so energetic – external gravity is produced which is somewhat very insignificant compared to the internal gravity & ( sufficient to influence a light bulb output )? What do you think?

                  pdf in your email

                63. A supernova’s gravity does not increase, a black hole’s gravity is no stronger than any other object with that mass. If our sun became a black hole today Earth would continue to orbit is, unaffected. The reason it seems that gravity gets much stronger is because you can approach much closer to the object. Think of holding a bowling ball, simple enough, the weight is spread out in your hands. But now think of putting a pin under it; the weight is the same, but now there is a small volume of space where all of it can be focused.

                  But your question is something I too wonder, particles can interconvert and a supernova sees a lot of mass converted to (nearly) massless neutrinos (Whose energy due to motion greatly outweighs their mass.) so the formation of a construct of massless particles is not too far fetched.

                64. Thanks for comment – I appreciate that. I take your point – actually I am less concerned about the external affect as that is more or less understood ( although not by me –! ). It is just the notion that we have 2 mechanisms of gravity the internal and the external.

                  In the case of the earth it has a gravity which forms part of a common entity internal and external. And we deduce that it has mass and it has energy which are easy – ( easier ) to visualize. In the case of a black hole it is has a secret gravity all to itself acting within it. What happens on the outside, one could say is small interest to the BH. The internal dynamic is making it work so hard that any other consideration is irrelevant.

                  In the case of the earth it is harmonious with the external features such as our moon etc. and nothing really very special happening on the inside ( compared to a BH ). But our BH on the other hand has its work cut out having to deal with its own unique internal gravity. Hence why light cannot get out.

                  We have two manifestations the external limit of its own gravity the so called EH and 2 the orbiting bodies. The EH is the manifestation and limit of it’s internal gravity. Yes I appreciate your example of the pin affect, but I cannot help feel that this is the outcome of the external affect – focusing gravity externally to it> more below:

                  When we looked at the Prof Laithwaite demonstration, from the point of view from his arm the lump of metal was substantially lighter ( less real gravity affect ) . The only way it could experience this change was because the lump was spinning. However, the missing bit in this action is the rod which was connected his hand.

                  Once could think the boring rod is a manifold in which the forces being produced in the rotating mass has a relationship coupled with his arm ( enabling the Prof to manipulate it easier ) . In the same way Space is a manifold between 2 bodies – energetic or not. Objects could be ‘invisibly physically coupled’, nothing to do with a propagating graviton concept.

                  Gravity possibly, may not be defined as a unique particle? Gravity may simply be a manifold through space – and not space-time in the 3D sense? It may be linked with the notion that each tiny volume of space is being duplicated constantly through the medium of 0 time, this being the connectivity – the couple.

                65. “The problem there is that the main force allowing matter to escape a supernova is not centrifugal, but radiation pressure. (Basically the massive particles are given enough velocity that they can just fly away.)

                  Massless particles have no need of this energy boost since they always move at light speed. All that keeps them bounded is interactions with massive particles. (Photons in the star’s core for example are constantly colliding with, being absorbed and being reflected by atomic nuclei and electrons.)”…..

                  Sorry I missed this reply in my email and did not look at PMS web comment list – I just found it. Thank you for your advice fascinating reply…….

                  You have a lot of detailed knowledge. O K so I can discount simplistic centrifugal. I suppose it may participate in a very minor role. But the Rad’P is far more powerful. However, the RP is acting in its own was as a separator – getting rid of the big stuff! As you comment the massive particles. This is obviously extremely important part in the success of the next phase. Defensively’ centrifugal or RP as description is lesser import the main thing is that the new system in order to pass into this next transition presumably must get rid of them? Otherwise it does not inititiate?

                66. “entanglement can affect two objects any distance apart instantaneously, no matter how far away they are. (And a number of physicists were upset at this ‘spooky action at a distance’) There’s no good reason for something to not act instantaneously over any distance”,

                  I just found your comment to another participant on PMS site. Yes!

                  But why is distance always so important? Of course it is important and we live our lives by it. But surely this is only an outcome of us existing in 3D ( centered around matter ) ? If we or matter existed in 1D we can be everywhere at the same time? There is no other ‘D’ reference frame to judge where we are? I am probably barking up the wrong trees. But I can’t help but feel that much of science is centered around the ability of matter without giving poor old time any consideration.

                  For instance if we took one step do we move 1 mtr or 1 light year in 1D? How do we know? Apart from how far our legs are separated ( that being the reference frame I suppose ) so are legs would have to be invisible to do this test.

                  If we exist in a space without matter how will we ever know where we are? comically

                67. No one can answer this question: But I would appreciate your insight.

                  Supposing the gravitational feed rate to a BH suddenly changed. Eg. Everything in an entire galaxy was to be suddenly dumped into one simultaneously?? What do you think would happen?

                  At the current time is has a very relaxed time converting orbiting material. But all at once? Any ideas? Do you think it would survive?

                68. The ‘conversion’ of matter is a property of the matter itself, not something the black hole does to it. Matter that falls together heats up as gravitational potential energy changes into kinetic energy. Matter also heats up when compressed since work is done to do so Gas clouds heat up when they collapse to form stars, meteors entering earth’s atmosphere warm it (They themselves don’t warm much, but they compress the air, force it together, which heats it, which then heats the meteor.) It’s not an organized process that changes ‘normal’ matter into black hole but an ordinary physical process as basic as your hands warming when you rub them or a lead weight being warmer after it’s tossed from the roof of a building. (If you try this its temperature can increase several degrees.)

                  So if suddenly a large amount of mass was dumped onto a black hole the hole itself would not be adversely affected. What would happen would depend on how the matter was arranged, how it was moving and how quickly it could get rid of its energy and fall into the hole. The biggest problem would be matter getting in the way of itself; the event horizon is a relatively small surface area and too much matter falling in at once would cause a ‘traffic jam’ and likely a large explosion as it heated up enough to initiate fusion.

                69. Confining my enquiry to the Standard Model questions only & LHC.

                  Q1. Proton vs proton collisions have been evaluated for some time now. We have learnt much about the classification of degenerate materials liberated – more so because of the LHC work. So what is a typ post collision classification ratio of degenerate materials? i.e. Does it have a regular identifiable list and is it uniform after each & every collision? Or is it a variable particle ratio of which materials are detected. Then referring to Higgs Boson ( like ) if we continue to collide these materials how often should we detect one? Or do we have to have millions of collisions before we see one again?

                  Q2. This question is really going to be even more inane. How do we know that the liberated proton construct particles actually built the proton and existed in it before the collision? Is there any possibility that when the proton’s disintegrate, what we detect is not what makes up the proton but what it is obliged to instantly construct during and after its destruction? i.e. making its transformation from one state into the next – it chooses to break down into a lower form. ( a metamorphic change vs release material which it indicative of its original composition.

                70. Your first question is also an interesting one. in short, there is an ‘average’ collision between two protons that occurs a lot of the time with a general list of collision products. This is like flipping a coin and getting heads or tails.

                  The actual results are much more complex with hadron jets containing a whole slew of particles occurring most often, not to mention decay products of these. And the results of interest are very rare, not usual at all (The Higgs is like flipping a coin and having it land on its edge.)

                  So what we have is like a mixture of your two options, if we have a lot of collisions we can be very sure about how much of what we will get, so sure in fact that a large part of the LHC’s data is simply discarded for being too normal, too uninteresting. Millions of collisions have to be discarded for being too average before we can analyze the special, rare collisions that produce what we want.

                  Your second question needs some correcting. What we see when protons collide are not ‘liberated’ particles from inside it. quarks and gluons can NEVER be freed from hadrons. What we see is the result of much energy-mass conversion and rearrangement of the initial particles. We will even see particles that should have nothing to do with protons, like muons and energetic photons.

                  Your question then becomes ‘How do we know protons are made of quarks and gluons?’ Over the past two decades we have accrued an incredible amount of exquisitely accurate data on the precise product and possibilities of particle collisions. These correlate exceedingly well with the quark-gluon model, *exceedingly* well. This is as close to certainty as we can get in this case, if the quark-gluon model is wrong, whatever replaces it will need a LOT of strong evidence.

                71. Your response is very clear thanks, however my full understanding is still way distant to the place where it needs be. Of course I cannot in all seriousness challenge what is physically true. And I am still struggling with the idea of ‘before and after’. And after asking you this question you will refer me back to your earlier response – as I am probably asking the same question here.

                  Simplistically ( Back to classical world for a moment ) if I put a log in the fire it decomposes via combustion. What is left is metal oxide ash. The MO’s in this case did form part of the log starting composition in the natural world. However, the log’s recognized shape and structure is lost forever into hot fume & vapour of combustible material. If I capture all this airborne, heat, and MO components I will have the entire log but in its changed form. So theoretically I could reconstruct the log.

                  In the case of the PP collision if I collected all the resultant particles + energy detected could I do the same for a single proton? In particular for each of all the protons so destroyed. Or unlike the log each of the collision products differ? The log will always produce the same decomposition products ( although the ratios will vary of course ).

                72. The example you give of a burning log is actually much closer to a proton-proton collision than you’d think.

                  Firstly, a burning log will never produce the exact same products. The ash will contain varying amounts of unburnt carbon, the gases varying amounts of carbon mon- and di- oxide, nitrogen, etc. Especially variable will be the number and energy of photons emitted. The products will often have an average composition but will not be exactly the same after each burn. Of course sometimes by chance the fire will go out and the log not burn or it will be particularly smoky fire.

                  Likewise p-p collision, which will have an average collision product composition most of the time and an unusual composition on rare occasions.

                  Next comes reconstructing the initial object. For the log and the proton you will not be able to exactly reconstruct to construct the original object exactly. To do so you would need to know both their initial structures exactly (For the log down to the atomic level or further.) You would need to know what particles were emitted from where, and due to quantum effects some of that at least will be guesswork (Or more accurately the particles involved will have lost individual identity for some time.)

                  Then there is the issue of how fundamental we want to get. From the products we coud confidently say that logs in general are made of carbon, oxygen, hydrogen and so on, but could not confidently say they were made of water, carbon dioxide, elemental nitrogen, etc. Likewise we can say quite confidently protons are made of quarks and gluons, but not that they are made of pions, muons or neutrinos.

                  So in summary, after burning a log and colliding a proton we could take the average products, do the math and be quite sure of what they are made of; but it is far more complicated then just grabbing everything and sticking it back together.

                73. “since two particles may be on a course where they should classically miss, yet, due to their quantum nature, interact.”

                  Yes exactly I am trying to think that one through hence my student take on it’s available and non normal energy condition in the collider. I am assuming that the proton in this case has its mass somewhat multiplied by it’s velocity? Under what other condition in nature ( cosmological etc) can we expect to see similar conditions of free proton? or proton only dynamics like these experiments. Thanks again for not recoiling at every nonsensical idea.

                74. In the case of the tokomak fusion reactor – with so much matter rotating at high velocity with energy and angular momentum – should they not produce a local gravity?

                75. They will produce a gravitational effect since they contain energy. However back of the envelope calculations suggest that this will be greatly outweighed by the actual reactor itself, that much mass also creates gravity. Enough that we actually have sensors sensitive enough to detect it. (You may have seen one on mythbusters.)

                  The only chance this would throw up an anomaly would be if a ‘current’ of energy (Not electrical energy, but in this case kinetic energy) produced a gravitational field, and did so far more strongly than ordinary stationary energy. However if it did do this, I think we would have noticed by now in the same way we cannot help but notice electromagnetic phenomena. (At the very least the effect should throw off particle physics equations.)

                76. TOK: Can they simply subtract the mass body from the equation and then detect the new if any gravity produced?

                  Sub -273: if this is correct where does this put Max Planck/ zero point field prediction, and use of value C in his equation to determine the zero point field? I have posited ( all will refer to this as nonsense ) I am ok with that as I don’t have a reputation to lose!

                  In that the entire mathematical determination to give the ZPF a value is hanging on the hook of value C. Should nature ( New Space Production ) value change in my nonsensical opinion – everything is varied and the current ZPF may rush down or rush up! And as for the atoms if the variation is minor they may simply cease to exist as atoms and return to being their primary material – whether energy or primary particle candidates?

                77. Whether or not such a gravity anomaly can be detected depends on its strength. Gravity is a terribly weak force so our limits of detection for it are quite large, relatively speaking. The problem is, if this anomaly is too large, if it is a significant effect, then it will begin to affect other things, such as particles in a cyclotron, and it would have been detected by now. I am not sure if the limits imposed by the fact that it hasn’t been detected yet would mean such an effect would not be detectable, but we shall have to see.

                  Keep in mind also that we have been observing rapidly rotating compact masses for some time, in the form of neutron stars (Even pairs of them.) and so far they are consistent with gravity as we know it.

                  Negative temperatures should not be thought of as being below 0K; a mass with a temperature of -1K has *more* energy than the same mass at 0K, and will lose that energy to ‘warm up’ to 0K. Negative temperatures are like ‘opposite’ or ‘mirror’ temperatures; At normal temperatures having more energy makes you less ordered, with higher entropy. At negative temperatures having more energy makes you *more* ordered, with lower entropy. (This is not quite true and a big simplification, but I digress.)

                  Imagine a bunch of balls in a box, all sitting on the bottom. The bottom of the box is mesh and connected to an air hose. This is ‘absolute zero’ for the balls, they are not moving. Now imagine turning on the air house. The balls are lifted by the air and bounce around. They are ‘warming up’, becoming less ordered. If you add more air, they fly higher and higher. Atoms have no maximum energy or disorder, you can heat normal matter forever.

                  But in our system we have a box top, an energy maximum. As we add more air from the hose the balls start to get stuck near the roof, they cannot fall down. They now have a negative temperature. Adding more energy pushes them closer to negative absolute zero, where they have as much energy as they can, all stuck right at the top of the box, unable to fall.

                  What the scientists did was similar; theytook a bunch of atoms that could NOT move any way they liked, (The cooling part was just to keep them in place, you can have negative temperature systems form at any normal temperature background.) They then twisted their situation so that suddenly they were all at maximum energy. (In the ball example say, they turned the box upside down so all the balls suddenly found themselves on the top of the box.)

                  The atoms would love to release all that energy and become lower energy atoms at about 0K, but the scientists stopped them via their setup. What they did not do was produce some matter that had less energy than it would at 0K. 0K is still the lowest energy state. (And -0K the highest!)

                78. I forgot : Manubo: the acronym is ‘Massive neutrino dark matter body’. However I have changed my mind on that, as I fell into the pliers of current regular science and try and find a particle to fit every aspect of nature. My thinking a year down the road it time is master not matter which is only temporary and transitory. Leaving a dominant phenomena where matter is irrelevant.

                  “Well, on the size issue, the poloid, being made of light speed particles would have to be exactly as big as the event horizon, any smaller and it would collapse into a singularity”

                  You are probably correct – however I have some small problem with that. e.g. the serpent which eats its tail. Something is producing the phenomena that is for certain. If the gravity is so intense would it not consume itself and disappear like the serpent! But that obviously does not happen and survives?

                  So if the internal gravity has collapsed and consumed everything the only thing that could ( possibly ) be left is nothing – and if that is the case I prefer a time window exposing the primary framework and no light is being transmitted because none is being produced , or its transit is only in one direction towards it. A crucible of molten metal does not consume the crucible – ( of course given time it would ).

                  For the time being I am somewhat stuck on the notion of a poloid – release from this would be as you say to see some evidence that such a thing could exist, if only in a numerical representation using a selected good particle candidate to find out what the quantized gravity is actually produced. Working back from the scale ( value ) of the current gravity evident in the milky way and then give a dimension ( size & Value of the gravity it produces ) of such a phenomena. Maybe the poloid could be the size of a child’s marble?? I would love to find out!! To think of something which is the size of a marble and is responsible for the hold over millions of starts is quite something to consider!

                  In an earlier exchanges of ideas you mentioned that a body cannot spin at light speed because it would crack. I thought about that – when you expressed that reply – I thought that was obvious and correct. But then pondering on it – it might want to crack. But on the basis that the resident gravity could have a greater influence on the matter than the mediocre forces due to rotational velocity would always win and remain intact?

                  However surely in one’s mind one can have a situation where as a single internal entity it may not have any gravity acting on it at all in isolation – it is only experienced beyond that body ( just a transmitter of gravity which actually has a zero resident gravity ) . E.g. an incandescent bulb element transmits external light, cannot melt and remains in equilibrium. Not the best comparison I know.

                  In short and my apologies a bit repetitive the toy black hole does not have sufficient ( self ) gravity to collapse itself, what it does is generate and transmits gravity – this is the picture in my mind.

                79. This is why I should much like a post on black holes from Professor Strassler; the structure and nature of black holes is now not entirely clear to me. It appears for example that black holes are not made of particles or matter, but just fields, disturbances in the fields of the particles that made the hole, and a gravitational field. Everything has collapsed and been destroyed, leaving only the fundamentals, fields themselves. But this may not be the right way to think.

                  It will be interesting to see where this model leads. As always, a mathematical description would be the most useful. We shall just have to see.

                80. Yes that would seem logical – a small body (relative ) which has force to influence millions of stars surely it must be an easy thing to collapse itself into oblivion!

                81. Hi Kudzu

                  Regards the Tokomat fusion reactor switch on in 2020. This machine involves both energy and angular momentum of a large number of particles in a much smaller diameter than the LHC. Do you think that evidence of a local generated gravity should be evident? Does it not have the right particle conditions to suggest this? Even a small a fraction? If not does that assume that the physical presumption that these conditions are necessary are incorrect?

                  In this respect this machine is just as important if not more so than the LHC?

                82. “The CBR is indeed from our universe, roughly speaking being created not in the big bang as such, but being modified over time through interactions with matter. (Until the temperature of the universe was low enough for atoms to form.) It’s an interesting and complex subject.”

                  Gulp! Oh dear I thought the CBR was alien not of this universe ( with origin beyond HZ) In a way that helps.. then I can revert back to imagine this universe totally void of matter. And what is in it is totally unique to this entity which we refer as universe – single specie. Thanks

                83. “While the formation of a black hole (or neutron star, etc) is incredibly violent and energetic, the crushing gravity keeps at least the central mass all moving inwards at roughly the same rate, overwhelming other considerations.”

                  – its gravity wins over explosive centrifugal forces? Discussed a few months ago.

                84. Yes, thought it does not win over all of them. When a neutron star or black hole is formed a large amount of matter succeeds in escaping, overcoming the gravity of the forming object. But that which is closer to the center experiences less outwards forces and more inwards pressure and so falls inwards.

                  What interests me is the formation of a neutron star; if you look at the table in this article: http://en.wikipedia.org/wiki/Supernova#Core_collapse you can see that if a star is massive enough, then almost no mass will escape the gravitational collapse and no supernova is created (But maybe a GRB.) On the other hand sometimes a supernova occurs that leaves nothing behind, the entire star is destroyed and ripped apart, gravity totally defeated. (If you have the inkling a more professional treatment of these is preferred, wikipedia is… rather cursory in these matters.)

                85. “When a neutron star or black hole is formed a large amount of matter succeeds in escaping, overcoming the gravity of the forming object. But that which is closer to the center experiences less outwards forces and more inwards pressure and so falls inwards.”

                  So how about this for an idea.. If the powerful outward throw of the CF sheds a ‘large’ amount of matter the – system is acting as a centrifugal filter. I used this technique involved in the invention of a end of line industrial gas clean up device for glass furnaces. I could tune this mechanical device so I could determine particle sizes retained or lost. It was effectively a particle size filter. The ultra fine particles which were so small >> Supposing all the particles with mass are ejected and thrown away leaving a balance of a system which is rich with mass less particles? Where does that leave us? in the laws of particles. If photons & ( gauge bosons ) are mass less but respond to the effect of gravity maybe this end system is rich in these materials?? Assuming of course that their parent atoms have already liberated & distributed them in the energy involved in the chaotic collapse?

                86. The problem there is that the main force allowing matter to escape a supernova is not centrifugal, but radiation pressure. (Basically the massive particles are given enough velocity that they can just fly away.)

                  Massless particles have no need of this energy boost since they always move at light speed. All that keeps them bounded is interactions with massive particles. (Photons in the star’s core for example are constantly colliding with, being absorbed and being reflected by atomic nuclei and electrons.)

                  You can see this when the star explodes, the first thing to be emitted is a massive burst of neutrinos of all flavors, unlike photons they do not interact with the star’s material and so can escape freely into space. 10% or more of the star’s rest mass can be emitted this way, more energy than the star emits as light during its entire lifetime.

                  Indeed when we detect supernovae often we can detect that blast of neutrinos, despite them being so nonreactive and from so far away.

                  What is left behind, if anything when a star explodes is massive particles that did not have the energy to escape their own gravity. (Even their energy will escape as photons over time as the star cools, unless it is a black hole of course.)

                87. Ref Neutron star and supernova qualification: Jim Reeves below may be available for more detailed information regards your question what determines the classification. I have no contact with him personally but he may be inclined to respond and participate? Is a very interesting question. He is a collaborator for this kind of research which was funded by the Royal Society.

                  Gamma-ray Bursts

                  Research is conducted into Gamma-ray bursts, otherwise known as GRBs. GRBs are thought to be the most powerful explosions in the known Universe, however until recently their origins were largely unknown. Recent research is showing that some types of GRB are linked with energetic supernovae, occurring in many distant galaxies throughout the Universe, associated with regions of intense star formation. A new development includes the detection of many high redshifts bursts, including one such burst at a redshift of z=6.3, one of the most distant known objects in the Universe. The study of distant high redshift bursts is important to our understanding of the early Universe shortly after the Big Bang and to how the first stars and galaxies formed.

                  For further information contact James Reeves.

                88. “then in returning to its historic orbit emits coherent light 1.6mu in the case of C02.”

                  No! 1.6 mu coherent is NdYag – I meant to type 10.6

                89. Sorry Kudzu,

                  I missed one of your earlier responses ref black holes. I read it with interest. It was quite a long response which I appreciate. You made many interesting points with number refs which I learned from. The question in response is – Black holes without a family galaxy. Regarding a solo BH surely if it was thermally decomposing ( even with BCR ) could we still not detect its entropy? I suppose the answer will be no –otherwise this would already be in the cosmo books. Eg number 321 is decaying at.. and number e32c is dumping lots of heat and getting smaller by the second! And it will cease to be a black hole 10 million years from now which is alarmingly rapid!

                  Everything you said makes sense to me – thanks

                90. As I have noted, this is in fact our best chance of ‘seeing’ a black hole; we would want a lone black hole, far from any mass it could observe, and rather small. given the age of our universe the only possibility is a primordial black hole, formed at the big bang. (These can have a wide range of masses, from a grain of salt to solar mass.)

                  Black holes produced by ‘normal’ mean would be too large, far too large to be evaporating given the CMB, but primordial black holes could be evaporating all the time. Rough calculations suggest a black hole formed with the mass of about a large asteroid would be nearing its end about now. If such a hole were close enough for us to observe we would most certainly be able to see it, and it would have a very unique signature, growing brighter and brighter as it evaporated,emitting immense power (Being very small) before… well we’re not entirely sure what happens just before a black hole ends.

                  You’ll want to check out http://en.wikipedia.org/wiki/Primordial_black_hole#Possible_detection Astronomers are actively keeping an eye out for such things and you can be quite sure the minute one is unambiguously seen the world will know about it. (It could well be Nobel prize material since it could well lead to massive information on dark matter, the big bang, black holes in general and even GRBs.)

                91. Question 2: IF Hawkings is correct and they do evaporate – Then surely we must be able to see it ? It must be glimmering no matter how low that glimmer is ??

                92. You are correct, all black holes have a ‘temperature’ and they glow with radiation appropriate to that temperature. However there is a problem; that temperature is very low for any black hole with any sort of mass. The temperature in Kelvin is (approximately) 1.23 * 10^23 Kg/Mass

                  So the black hole as heavy as our sun (2*10^30Kg) has a temperature of roughly 0.0000000615K, barely above absolute zero. This makes them colder than space itself, which is filled with the cosmic microwave background radiation from the big bang, at a temperature of about 3K.

                  This means that most black holes are getting more energy from the CMB then they emit due to their temperature. This makes them impossible to see by emission.

                  Second, unless a black hole is VERY hot, it doesn’t emit much of anything; The total power output of a black body is proportional to the *fourth power* of the temperature (Stefan–Boltzmann law); which means if you halve the temperature the power output is only 6% Now imagine a star, think how faint it is. It has a temperature of thousands of K. To reduce it to that of a solar mass black hole (One of the warmer ones!) we need to halve its temperature about 25 times. This reduces its power output by about 10^30. To get the same output you would need a galaxy of galaxies of black holes (100 billion cubed), so you can imagine how faint they must be.

                  Finally it can be real hard to find 8anything* in space; most black holes are surrounded by dust, other stars and all sorts of debris that can block all sorts of signals. We have a hard enough time seeing the center of our own galaxy because of the dust clouds obscuring it. And many black holes are surrounded by light and hot gas as they absorb matter. (Some are among the brightest objects in the universe.) Seeing the hole under all of that is a formidable task indeed.

                  I suspect our only good chance of seeing a ‘plain’ black hole would be if a primordial (left over from the big bang.) black hole, which could be quite light, and thus quite hot, evaporated nearby, alerting us with its dying flash.

                93. Professor – however I would like to press you on the subject of the very beginning of the universe. Did it have no matter for a period no matter how short? If so would STR and GTR be lawful?

                94. If you go back far enough we just don’t know.

                  We do not know details of any period where general relativity was invalid, though surely at some point in the universe’s history a quantum theory of gravity (which might be much more complicated than general relativity) was important. And there may have been periods without any space. We have no experimental or observational access to such questions at the moment.

                  If general relativity is valid, then, at short enough distances and times, special relativity is also always valid.

                  What does it mean to have “no matter”? It depends upon *exactly* how you define matter, and as I have emphasized in my articles on matter, people do use different definitions — the term is not uniquely defined.

                95. I appreciate your comments – and thanks.

                  Ignoring the politics of science, and the possible lack of community knowledge or perhaps even consideration. Do you think that this

                  Universe could exist with 2 realities of time? 1 constant and 1 variable?

                  1. Matter/mass in my mind is anything which has 3 dimensions – or even 2, Quantum or otherwise which enjoys a name or not.

                  3×2, 2y, 7xy, and 5. Algebraic terms like these may consist of variables, coefficients, or constants all of which can have a time attribute.

                  If we do exist in a universe with 2 realities of time then all the physics books have to be rewritten from scratch? Do they not? And IF, the universe started this way ( absence of 1 above ) then we can commence argument for the case for its continued existence? Respectfully

                96. “And there may have been periods without any space”. That’s a good cognitive twist? ArgH! How can you have no space

                  When matter exists?? Or was that a typo – got me confused now……

                97. Yes I take your point but it seems that when sufficient forces ( gravity ) are available the electrons become the slaves. Actually my meaning regarding the energy output of black holes : The energy output is more than impressive by evidence of the double gamma streams. Science tells us that we don’t know anything about the inside of one because it gives us no opportunity for information. But if you think about it that is not entirely correct – it gives lots of information ( not enough mind you ). 1. We know it produces a fantastic gravitational field, 2. We know it eats atomic vapour otherwise we would not be able to see it, 3, It has 2 poles, 4, It ejects unimaginable quantum of gamma rays, 5. Its polar faces are black! 6. In the lab and in the cosmos we know how cosmic gamma is produced:

                  1. a particle can collide and annihilate with its anti-particle, ?? 2. an element can undergo radioactive decay, or 3. a charged particle can be accelerated. – I could see this one being true 4. a high-energy particle can collide with another particle

                  So which one of these conditions do you think are actually occurring inside a black hole – or all of them? Scientifically have we abandoned the notion that it just devours mass simplistically and indefinitely? Or is only converting that mass into a transmitted high energy composite wave inc radioactive to get rid of it? Lastly do you think that the vacuum energy participates in these dynamics and also do you think the vacuum energy is nothing more than the stretch of the universe actually expanding around us invisibly – as the Primary dimension – hence whatever is happening inside the BH could it involve an interaction with vacuum energy? i.e. with the very expansion of the universe? “My expression The Torque effect”?

                98. You are correct about electrons and gravity, this is ‘electron degeneracy’, the step before neutron degeneracy. The problem is that the laws of physics state that in strong enough gravity they must combine with any protons in the vicinity. The neutron and proton are on a fine balance, the neutron is only slightly unstable (It has a ten minute half life, practically forever.) If you push electrons and protons just a bit the balance shifts. (Like it atomic nuclei.)

                  Some of your points are also a bit off; black holes eat anything, and if they’re not eating, we can’t see them. Its has poles only if it is rotating and ejects gamma rays only if it is eating.

                  Gamma ray bursts can be produced by a number of things. matter-antimatter collisions are not one of them. (There’s not enough anti-matter in space.) A lot of these are to do with exploding stars, supernovae and novae that release immense amounts of energy very quickly. Black holes emit gamma rays (and more usually x rays.) when they eat something; the gas swirls around it faster and faster, hotter and hotter until it gets so hot that it glows, not with light, but with much more energetic rays. So in essence we can see black holes by friction.

                  In fact NASA’s NuSTAR project has just seen the black hole in the middle of our own galaxy devour a small gas cloud, emitting a burst of x rays.

                99. Thanks Kudzu for your continued insight – very helpful. I found this in wiki reading electron degeneracy: which I thought was very interesting and complied to my lay intuition regards the ability of instantaneous infinite compression. Also very interested to find Plank involved in its process! He keeps popping up everywhere, like Pi & ‘C’ – which encourages me and my notion regards possibility of 4 spatial realities. Which you did not comment on and was hoping you would.. Namely the condition of the universe in the first 3 seconds and it existing as a ‘single 1D face’ void of mass. Which must still exist despite the invention of mass much later. 3 seconds is a very very long time. Especially if the primary spatial can only exist as 0 or 1?

                  “Electron degeneracy pressure will halt the gravitational collapse of a star if its mass is below the Chandrasekhar Limit (1.44 solar masses [3]). This is the pressure that prevents a white dwarf star from collapsing. A star exceeding this limit and without usable nuclear fuel will continue to collapse to form either a neutron star or black hole, because the degeneracy pressure provided by the electrons is weaker than the inward pull of gravity.”

                  So it this is correct we can have an amalgam of mass particles separated from the electrons?? Is this thinking on or off the wall? Thank you for alerting me to this bit of physics.

                100. I am rather unqualified when speaking of the birth of the universe; the notion especially of dimensions is a tricky one when you have the possibility of 10 or more dimensions in string theory. I can offer no insight there.

                  Most matter is composed of fermions; particles that don’t like being exactly the same. (This is Pauli’s exclusion principle.) when you squash them together enough they refuse to compress any further, this is the ‘degeneracy pressure’ When this happens to normal matter, the link between protons, neutrons and electrons has long broken down; the electrons are not bound to nuclei and can flow through the mass like a sea.

                  So yes, matter with electron degeneracy is a bunch of massive hadrons (That may be feeling very little pressure, since they can be placed much closer together.) with the electrons flowing through it and behaving quite differently from what we usually see.

                101. I see! So what would be the conditions of such a body? Squashed hadrons?

                  In the words of Edward Witten – he states that he is actually short of 1 dimension to make all his work!

                102. So it seems that such an singularity is again in the realms of the physical mystery. So we cant predict what those conditions are as we have no experience of it?

                103. So if black holes and singularities are going to exist as a constant conjecture – I think I better use my time just thinking about the likely hood of the primary dimension – as this could be provable? Starting with entanglement……and information exchange. If we could discover by simple data search and cosmic observation any evidence that one thing is affecting another thing where the distances are just too huge consider such information being transferred at ‘c’. Starting with the Horizon Problem. If the void has a symmetrical time than surely such information is transferable as the distance becomes irrelevant?

                104. Before a few days ago I would have had to say that black holes are quite mysterious and that there is little information about what they are, however the Professor’s response suggests that I am misinformed and that we probably know a lot more about them then I can imagine. As such I cannot really answer questions about what black holes are anymore, I have been informed by a reliable source that anything I tell you is likely to be wrong.

                  Sunspot frequencies are not patterns anymore than how often you play the piano is a song. There is however a pattern IN the frequency, the solar cycle.

                105. Actually, in exchanging ideas relating to black holes – your responses have been extremely helpful, and my intuition of them improved and up dated substantially. As I was somewhat stuck on the idea ( not having really thought about them very much ) apart from being amazed by the space telescopes images of them. That they were just compressed matter. But now I realize that they are far more interesting than that – and involves the work of a Greek philosopher. Nature has to do what it has to do! And has it’s very strict laws to obey. It can only do things according to those law’s with 100% loyalty. The first law surely? must be Expansion and the 2nd ‘Equilibrium’. Everything it does is to achieve the 2nd , including black holes. So if it is out of equilibrium then I guess Sir Hawking is correct and they enjoy a thermal decomposition? However, that does not remove the intense interest in what is there clockwork?

                  Sun spots: we know the frequency of our sun spots some 2000+ years amazingly! We are also observing other suns 10 in total starting with Alpha Centauri including their own sun spots. My question was: do we know if sun spots occur at the same time? You as a competent mathematician would be able to compensate for the observational ‘C’ time lag to take that out of the equation.

                  IF, sun spots are simultaneous would have me dancing in the street – and all physics books would have to be rewritten. Why? because it would demonstrate that information is acting on them simultaneously. For example you have a football team on the field they each have a flag. There is red light nearby and when they see it light up they all simultaneously raise their flags. The speed of light ( hysteresis ) does not matter as the distances are so short in this example. In the case of the suns if a spot occurs ( noticeable solar activity of some kind ) are simultaneous with the others infers that they are being mutually disturbed at the same time. And demonstration of the primary dimension as this could be the only likely account ( information ) for that observation. In my thinking! Galileo observed the link between the moon and tides – in this case the information was gravity. The hysteresis effect is tiny because of the spatial distance of the 2 objects. Have I posed this question clearly? I have tried to contact NASA on this point but cannot find the right person to respond to it meaningfully.

                  Which on the dream sheet if the answer is yes! Then the clockwork of a black hole could be experiencing a physical relationship with it? Not to mention an even greater realization that light does not determine its own velocity in isolation! – Sorry a lot of text here…

                106. As you say, if stars light years apart were acting in synch, the physics textbooks would have to be rewritten. You can be quite sure that if any evidence of this comes to light it will be in the newspapers quite shortly, but to answer your question, no, at the present time there is no evidence that our sun is acting in synch with any other star aside from on the galactic scale, where it, like all stars, orbits the galactic center.

                107. Thanks Kudzu – “no, at the present time there is no evidence that our sun is acting in synch with any other star”

                  But has it actually been considered – or better – studied?. Regards Alpha centauri we observe it’s sun spots behavior ( ref your correct phrase solar cycle – being seasonal magnetic affect or otherwise). I have considered the solar cycle frequency of our sun but cannot find any relating to another. If I could view such a graph it would immediately be apparent if there is any evidence of ( synch )! Any refs?

                  Conjecture – Black Holes: Matter close to the black hole is near light speed and accelerating. Which infers the invisible matter entering the hole is at light speed? – or maybe in synch with the velocity conditions inside? Which could mean A black Hole is a ‘Time Zero Portal’ where 300,000kms = ‘0’ variable time. if you could uphold my 4D notion ut, x, y, x, then black holes are just black 1D spherical time holes and no light is visible because non is being transmitted? And 60% of the mass digested ( the balance dissipated by x ray ). A TZP in the sense that this window of time will be in time synch with primary dimension of ‘Ut’. Then if you could also consider my earlier possible solution to the horizon problem – then any ( matter ?) successfully entered the black hole may actually be distributed anywhere across the primary Ut space instantly. Which would make Prof Witten extremely happy as the matter may undergo a complete degeneration into strings which ( could ) be the construct of the primary dimension in the first place and void of conventional matter where STR and GTR have no applicability? The total degeneration of the SM inside the TZP is imaginable by considering Max Planks theory. If you adjust the numerical value of ‘C’ does one not have the right to claim that all the parts of an atom will dissociate.

                  Considering the LHC work, personally I don’t think they will ever exceed ‘C’ as it is not possible in this part of the universe. Maybe if the LHC was located outside the fringe of the Hubble Zone I am more confident. IF, after providing more power to the LHC one thing I could see happening is that should they apply more energy to the proton and increase it’s velocity by ( 0.001% for illustration purposes only ) I could imagine it would simply disappear without having to collide it with another. I am not sure about the safety of this – should such a single proton degenerate back into purest form of energy ( or strings ) the quantum of energy released is rather scary. And I should think about moving out of France!

                  Then regards non rotating black holes and or where matter is not being eaten, then presumably the dynamics of the final collapse of the supernova was sufficient to create such a TZP window and is held open by the same internal dynamic until it decomposes? and closes the window – assuming it is not in equilibrium because not enough cosmic background particles are sustaining it?

                  Then the reason why so much gravity is created is because the ( graviton if It does indeed have an associate particle ?) and not have an associate string? These are successfully transmitted from the BH in absence of anything else and cannot degenerate – and or pass into the primary dimension which is obliged not to have any gravity otherwise it presumably could not do which is to expand constantly? – which it does.

                108. You can be quite certain that people have indeed looked at whether the solar cycle syncs with many things, from planetary temperatures to other stars. They have probably not looked at everything, but you can’t expect that with anything. But again, it would be big news indeed if a link were found, so you can expect people will keep looking.

                  Matter near a black hole is moving fast and accelerating, but as it does so and gets more energy, like all things its acceleration is dampened and it never actually reaches light speed.. From its perspective time flows fine. This is one of the problems with black holes, close enough to them you and I will disagree about what happens, even if neither of us passes the event horizon.

                  Also, a sphere is by definition 3D, you cannot have a 1D sphere. I do not think I fully grasp your theory (Some math would be nice.) But there has been some discussion it seems as to what it means to ‘cross’ the event horizon. This video provides a nice summary, and the papers themselves were interesting (If far from comprehensible to me, you may have better luck.) http://www.youtube.com/watch?v=XE5PNbsUERE

                109. “Also, a sphere is by definition 3D, you cannot have a 1D sphere.”

                  Ref: A ‘Sphere of Time’ without matter existing in it.

                  Absolutely, in our/your classical ( Euclidian ) way of describing it. The secret is to modify our comprehension ( about time ) that such a thing could exist in 1D. A quick introduction in passing I refer briefly to a mobius ring. It, as you know only has 1 face ( boundary ). Call the thickness of the paper ( or whatever you are using to make one ) zero? Then we can argue it only has 2D. If, then we geometrically extend the width so all sides connected ( we end up with an object the – Kleinwort ), but then the entire body has only 1D or 1 face/boundary if you like. This illustration does not really transmit my understanding very well but is a starting point to help me try explain my unqualified notion. ( I also realize that Kleinwort’s are impossible in Euclidian space ).

                  But this notion is not about shape geometry it is about time itself – or a reinvented historical notion that perhaps it could exist as a symmetrical entity later discounted by Einstein and Minkowski. When the big bang was initiated it caused matter and spacetime to exist = Time 2 variable time !

                  To help me maintain this intuition in my thinking I simply imagine the big bang with its supposed tiny diameter ( ignoring the creation of spacetime and it’s complicated carryon with implication of variable time?). When that event occurred it happened in a unique moment of time, since it exits hypothetically without the ability of variable – linear space time ). As the expansion continued that same moment of time is preserved – the space is bigger but the time remains the same. The event time is preserved within it- and universally isometric in 1D.

                  Then returning to the idea of spacetime this is a separate entity within it which can only occur if matter is available. Further to my other recent comments and in thinking about Quantum relativity – this is not properly understood and it has already been reported that STR actually ceases to be a concrete theory in QM. So surely this must be indicative that before matter occurred STR had and has no validity. It is only valid when matter exists causing so called spacetime, space-time , space and time, space-time continuum. Even the definitions are ambiguous.

                  I watched your ref with great interest – 1 constant: played all the way through non stop and the second time: variable interrupting the presentation ( comically ) !! It was very interesting – however, if my car is short of petrol I need to get to the pump quickly, what happens between the time I realize when my car needs petrol and the moment I actually arrive at the pump is of lesser importance. But of course the countryside I am moving through is very interesting. I noted that the presenter only mentioned Time once – and passed over it very quickly? Science communities don’t like to talk about time as it causes so much confusion, it is not being understood – you could say entirely ‘not understood’ ( hence it paradoxes ). And with great respect to Matt he uses the phrase ‘Time gets Warped in a BH”! Which is an expression from Star Trek I think. Whereas , in my thinking it is probably THE most important thing! We need a new coat hook so people can actually meaningfully discuss time and the very important role and way it affects things. I can’t help but feel that we do live in 2D of time ref of TC + TV. The secret now is how can I prove it? ( I have mentioned one way in another comment ).

                110. Experimental confirmation

                  Gravitational time dilation has been experimentally measured using atomic clocks on airplanes. The clocks aboard the airplanes were slightly faster with respect to clocks on the ground.

                  ? extract from wiki is this true or incorrect. My understanding is that time runs slower? What am I missing here please.

                111. If we continue to accelerate to 0.99999999999999 c, for every day on board, nearly twenty thousand years pass for the observer at rest.

                  Above is my understanding.

                  In the below the observer sees the moving clock pass time faster??

                  So the clock ticks faster in gravity opposite to the above?. What am I missing here – sorry for the fundamental question?

                112. The situations are very different.

                  A clock that sits at a fixed radius orbiting a massive object has a gravitational slowing of its timekeeping relative to a clock that is very far away from that object. That’s a fact about general relativity.

                  The fact you quote isn’t relevant to this situation.

                113. Thanks Matt –very silly question – that was demonstrated in 1976! And presumably the clock runs ever faster dare I say as it moves away from the source of gravity by virtue of its flight through the gravity inverse? Or at a higher orbital distance? Meaning 1 of 2 directions of motion respective to the body – 1 valid or both valid? I am beginning to realize the interest in photons now!

                114. It does, no matter what direction it moves in, as long as that direction is ‘away from the mass’ The effect gets smaller with distance, just like gravity, so moving 2x away from the massive object results in an additional speeding up only a quarter that obtained by moving x away from it.

                115. Thanks Kudzu – got me back on the straight and narrow again. I have my mind filled with notions of time now I am tangential to it because of gravity !

                116. Hi Matt

                  Torus gravity multiplier. – A possible intuitive mechanism for a Black Hole.

                  I have attached a schematic…I don’t know if you can received or not?? Please advise. In case not here a text at attempt to describe it

                  Description: If you cannot see the schematic please advise how I can present it to you by alternative means.

                  1. A particle torus in space

                  2. The particles are rotating around its transverse at ‘C’. ( like a smoke ring – or atomic mushroom cloud to aid visualization ).

                  3. The torus has an epicentre so 100% of the particles coincide at the centre.

                  4. A gravity is inferred within the transverse section of the torus because of the particles energy and radial momentum.

                  5. Where the 100% of particles coincide at the epicentre it also causes a gravity, by their energy and radial momentum.

                  6. Because the epicentre now has a gravitational force >than the torus it is causes the reduction its diameter

                  7. As the this diameter is reduced causes the rotational radial diameter of the particles to increase velocity – hence reduce their diameter and increase gravity!

                  8. The net effect of this is to increase the epicentre gravity – and so on.

                  The entire system remains in stress as the epicentre tries in vein to pull in the outer gravity.

                  The particles cannot move faster than light because their reference time would have to t = ( – 0 ).

                  Therefore it is not velocity which is master it is Time!, or one can think in velocity if preferred – but you cannot move any faster

                  when the motion = (0) in time.

                  The entire mechanism is experiencing a torque with time. If the particles could run faster, time would HAVE run backwards!

                  Please advise what you think please.

                117. This can indeed be a bit confusing. If I move you fast, time runs slower for you compared to someone standing still. however if I put you in an airplane, away from the earth’s gravity (Or better still in space.) then time runs faster for you compared to someone on the ground.

                  You can think of gravity as accelerating you all the time; if you are in space, weightless and you want to stick to the floor, you need to move the spaceship (or spin it around) just like how you feel heavier in an elevator that’s moving up.

                  So you can imagine that everyone standing on the ground is moving, accelerating at 9.8m/s (even though they’re standing still.) so time is already running slow for us. (If you were out in space you’d see people on earth moving just a tiny bit slower than you.)

                  By putting a clock in an airplane away from gravity, it’s as if the clock isn’t moving so fast (if the plane is not moving too fast, which it isn’t.) and so time for it moves faster.

                  It’s funny to think that earth is actually slowing all of humanity down, actually applying relativity to us, just a tiny bit all the time.

                118. These statements about “nuclear matter” are not correct. As far as we know, most of what happens when matter is highly compressed, as in a neutron star, is that protons and electrons combine to form neutrons and neutrinos — it’s a transformation, the electrons are simply gone, and some of the up quarks in the proton have been combined to form down quarks. The neutrinos escape into space, and the neutrons remain. It is true there are various speculations that suggest this is not exactly what happens. But we do know supernovas create huge numbers of neutrinos (they’ve been detected) and those neutrinos were created as electrons were annihilated.

                119. I can’t follow this conversation between you and Kudzu, but there are a lot of mistaken things here.

                  First, the stage of the universe that we know we do not understand is not the first three seconds after the Big Bang began, it is much shorter than that. [We do not know the start of the Big Bang was the start of the Universe.]

                  Mass was not “invented” after 3 seconds. There were objects with mass as soon as there was space.

                  It is believed that a period of inflation preceded what people normally call the Big Bang… the period of high heat and density which has been cooling off and spreading out since then. During inflation, the universe was cold. We don’t know for sure that inflation took place, and we certainly don’t know how long inflation lasted or what preceded it.

                  We do not not know the universe had a one-dimensional episode.

                120. Fantastic – thanks for your valued input: The reference 3 secs is soured from ‘wikipia’. It seemed to me rather fantastical that such a time reference & account could ever be realistic – since we are billions of years from that point in time. The same reference also goes on to say that mass initially occurred in 13 secs after the first 3 secs !! I am sure I could make up a new statement which would be just a valid – unworthy? “Invented infers my cynicism on this statement”

                  Actually this is not the point – but only a reference for visualization. Does anybody know if we actually had a period of no matter – no matter how short a time? I know the whole subject is like a wobbly jelly which refuses to set. But if we did have a period of no mass then surely STR and GTR has no validity? And that space should then have a unitary dimension as there is no opportunity for any reference frames etc.. And Euclid to write his elements …etc.

                  In your comment below you infer Instant space = Instant mass, although you have implied some doubt about that. If a space is happy to exist without mass in <3 secs then surely that same space still exists just bigger – but with mass in it for a very long time – and hence all our comprehension of space is centered around it? thanks

                121. You get time in the early universe by computing the relation between temperature and time in the Big Bang and running things in reverse; we know what was happening at 3 seconds after the Big Bang started (not after the Universe started! we don’t know those are the same thing!) because we know what the temperature would have been at t=3 seconds and we know the laws of nature well enough to know what was happening at that temperature.

                  If Wikipedia says “mass initially occurred” then all I can say is that phrase is nonsensical and should be removed. Wikipedia articles are not reliable sources for scientific information.

                  Again, you are conflating mass and matter — big mistake! You will not be able to read science articles if you don’t understand that these terms are not what you think. I can’t answer your questions until your questions are well-defined:

                  http://profmattstrassler.com/articles-and-posts/particle-physics-basics/mass-energy-matter-etc/matter-and-energy-a-false-dichotomy/

                  http://profmattstrassler.com/articles-and-posts/particle-physics-basics/mass-energy-matter-etc/mass-and-energy/

                122. Thanks Professor – I have presented an clearer question in this train. Yes I understand the differences of terms. I was careless as they are not really that important on my road of discovery. I am more interested in the availability of dimensions. Mass/matter etc…invoke notions of space that’s all. Respectfully.

                123. “we know what was happening at 3 seconds after the Big Bang started (not after the Universe started! we don’t know those are the same thing!)”

                  Yes quite!! Interesting is it not?

                  Sir Hawking refers to the’ big crunch’ – that is ok for an idea based on the idea that everything in the universe is centered around ( matter – I have to be careful now which word I use ) – shrinkage due to a collective mass/energy centre etc. BUT maybe not that at all – but a reciprocating universe oscillation. Across its entire diameter whatever it is at that time. This could happen and with no loss of information IF, the other concept of time exists. ( all conjecture of course – Hawking idea based on known reality of matter – which weakens mine – all granted )

                  The actual dimension during these phases of oscillation is not really important. Maybe big ? maybe small? The size is almost an irrelevance. What drives these change frequencies is the expansion/stretch of the universe. It shortens then value ‘c’ changes. it lengthens and the value of ‘c’ changes. And this could be the very mechanism which initiates the creation of matter – thanks to Max Plank? And the proton in the LHC cannot move any faster than light as all things need space to move. If that space does not exist how can it move? Or, move faster than it is being created! Courtesy of the framework of expansion around us second for second.

                  I know you are not going to be drawn onto this notion which is a great disappointment ( for me).

                124. Thanks Matt, yes it realize that – mostly conjecture! “We do not know the universe had a one-dimensional episode” – indeed so. But if it did?? Would that not be more than interesting? respectfully

                125. Vacuum energy. The BBN says that the mass started to appear approximately ( 3 mins ) after the BB and the initial mass took around 13 minutes to be created hydrogen mainly…. In these 3 linear minutes ( if this number could actually be true??) then the BB had grown to a dimension of 54,000,000 kms – based on the numeric value of 300,000kms – but actually may have been substantially more. Which is quite a big space! The diameter of the solar system is approx 300,000,000,000kms to help with the visualization.

                  This space was real and existed without any matter primordial or otherwise. Theoretically it could only have had one dimension. Then in the 4th second when baryons started to appear – the original void of above figure was still stretching. Then when matter appeared we now have a Cartesian space & ability for STR and GTR.

                  How can you have a Cartesian space when there is nothing in that space except it’s 1 dimensional surface? So is there an argument which could state that the vacuum energy is nothing more than 1. Evidence of a primary dimension and 2. Provided by the very stretch of the universe still expanding around us??

                126. Thanks – but in my comment I am referring to both, mass/matter/particles – anything which has a dimension actually.

                127. I don’t know how to answer this: the phrase “mass/matter/particles – anything which has a dimension actually” is nonsensical.

                  Particles refers to the elementary constituents of all things, as far as we know today, as well as clusters of other particles that move as a unit.

                  Mass is a property that a thing or a collection of things may have.

                  Matter is (depending on your definition, and there are several) a substance made from certain types of particles arranged in definite ways.

                  Dimension is a direction in space; it is not something that particles, mass or matter have.

                  If by “dimension” you meant “size“, then mass, as a property, does not have size; elementary particles do not necessarily have size, and certainly their size is too small to measure if they have any; and matter, because it is made from collections of particles, does have size, but what definition of matter are you using? there are several. http://profmattstrassler.com/articles-and-posts/particle-physics-basics/mass-energy-matter-etc/matter-and-energy-a-false-dichotomy/

                128. Black holes are by their nature unobservable, anything you try to observe it with vanishes into it. (That is, say if you try and take a picture of it, the light gets sucked in, not the camera.) The only way we know black holes exist is by their gravitational effects, we look at a patch of sky and see something distorting space, making objects orbit it quickly, sucking in gas and so on. We can tell how big (roughly) it must be and how heavy, thus how dense.

                  And gravity is part of what black holes are; even if we could observe a ‘black hole’ with no gravity, it wouldn’t be a black hole anymore. (For one thing, what *would* it be? It wouldn’t be able to suck in light and mass, no gravity.) So no, gravity is part of the whole black hole package.

                129. Agree entirely,

                  The reason I asked was some black holes do not have associate galaxies? Ok I know most galaxies have super massive BH parents. But a BH like a sun will cause things into an orbit – would it not? So why do some BH have an orbital family? – All larger than our solar system.

                130. This question is interesting; there’s a lot out there that we can’t see via light at least, planets thrown from stars, dust clouds, and yes, black holes. Only ‘small’ (Think a few times the mass of our sun) holes are likely to ever be ejected from galaxies (Not the center of the galaxy, but holes formed from single dying stars.) and there won’t be many of them, but they should exist, along with holes of all sizes (even tiny ones, say as massive as a mountain) formed in the early universe.

                  We can see them if they pass in front of something, where they block and distort its light via ‘gravitational lensing’ Such things have been considered candidates for dark matter, but it turns out there’s not nearly enough of them for that to work.

                131. “We can tell how big (roughly) it must be and how heavy, thus how dense.”

                  Presumably when we apply normal physical rules to it. But it’s density may only be a virtual one – it has no density and be ‘0’?

                  Mass, charge and angular momentum – these apparently definite it. We have static ones so therefore we are down to 2 definitions. If the density is a virtual only, then we are then down to Charge? So how do we measure its charge? – any ideas?

                132. When we ‘look’ at black holes and calculate their size, density and soon, yes these are approximations. Imagine looking at a swooping bird far away on a foggy morning. You cannot see it exactly, but you can tell say, from how it appears next to a tree approximately how big it is (It is obviously not as big as a tree, but smaller. if you know the size of the tree and how far away it is you can make even better guesses.) from how fast it flies you can tell how heavy it is.

                  In the early days of black holes when their existence was unsure the big challenge was finding a ‘black hole candidate’ that *had* to be a black hole and not just say a neutron star. eventually we found things that were so small yet so heavy we could say ‘Whatever this is, it isn’t anything we know of.’

                  When we look at galactic centers we see heavy, small objects. Exactly how heavy and small we can’t say, but even if we assume they are as big as possible and as light as possible (while still doing what they’re doing) then they’re still too dense to be anything that doesn’t fall under ‘black hole’

                133. No — a black hole cannot merely be thought of as a large atom. Although it shares some features with atoms, and with elementary particles, too many of its other features are special and unique to black holes.

                134. Yes Matt – I have that now – thanks – I am imagining different behavior now– this was just building my visualization that’s all.

                135. Dear Professor returning your comment earlier – and thanks for keeping me on the straight and narrow.

                  “If by “dimension” you meant “size“, then mass, as a property, does not have size; elementary particles do not necessarily have size, and certainly their size is too small to measure if they have any; and matter, because it is made from collections of particles, does have size, but what definition of matter are you using? there are several.”

                  Being over zealous I caused confusion sorry for that: What I mean is anything which has a 3D or ( 2D?) volumetric displacement of Space. If it exists and we can detect it and presumably displaces it or occupies it no matter how small and we cannot measure it? Even the very short lived ones. And as the very small ones start to contradict STR is that no an indication of some other bit of missing physics and a lead us into a further understanding of some other physical influence? Respectfully.

              2. Sorry in my last message I used the wrong word. My meaning is ‘could the kinetics of the new diameter 5km’ cause a virtual increase of the supposed static mass. The gravity is real enough – and very impressive.

    2. Zeynel — I notice from your questions that you are very confused about many things where the cause is that you are not reading very carefully. I do try to write things carefully so that the details are precisely right. You said that I wrote

      “For slow moving object p=Mv (where v is the object’s velocity) and pc=Mvc is must smaller than Mc^2. And therefore E=mc^2.”

      That is not what I wrote. Had I written it, it would have been incorrect.

      I wrote “And therefore E ≈ m c^2”. Energy is APPROXIMATELY m c^2 (differing from it by a small amount).

      Everything about the rest of your comment is off base, because you misquoted me at the very start. It is important that you read with care to avoid confusions. I do occasionally make typos, and even more rarely outright errors, but in this case what I wrote is exactly right.

  60. Can you explain this sentence, I don’t understand why [for slow moving objects] “the gravitiational forces between them are proportional to E1E2=M1M2c4.” I don’t understand how you justify writing E2=M2c4 as E1E2=M1M2c4. Thank you.

    1. In a low speed system the force of gravity depends just on how much stuff is there. If we took half the earth and stuck it on the moon, the gravity between the half earth and bigger moon would still be the same.

      Two masses have two energies that must be multiplied together (E1 * E2) This makes E^2 (Which is NOT the same as E2!)

      Imagine both masses were the same size, then we have E * E = Mc^2 * Mc^2 which is E^2 = M^2C^4.

      But usually the masses are different so now we have E1 * E2 = M1c^2 * M2c^2 which is E^2 = M1M2C^4.

      Make sense now?

      1. Kudzu writes: “In a low speed system the force of gravity depends just on how much stuff is there.”

        Where do you get this from? What is derived in this post is simply that an object at rest has Energy which is equal to its mass times the speed of light squared. Where do you get that there is a gravitational force between them and it depends how much “stuff” there is? That seems like a Newtonian assumption.

        “Two masses have two energies that must be multiplied together”

        Why? Why do multiply energies?

        1. The Newtonian observation was that the more massive something was, the stronger the gravitational force it exerted. (Twice the mass, twice the gravity.)

          Einstein showed that it was not just mass that mattered, but energy (So light is affected by gravity, despite being massless.) We do not know why this is as far as I am aware, it just is, for the moment.

          Since the amount of energy determines how much gravity something has and we have TWO objects interacting, the total force between them is obtained by multiplying their energies. (Two objects of 1E each exert x force, Two objects of 2E each exert 4x force.)

          Since in a low speed, approximately newtonian system E ~ Mc^2, then the gravitational force is proportional to the two masses multiplied together.

          If we want a more accurate Einsteinian view, then things get more complicated.

          1. Dear Professor – please advise what is your thinking on this. A universe 100% void of mass. Would STR and GTR be valid in this scenario? Plus could that space be defined as a cartesion/euclidian zone with no reference frames in it?

      2. Kudzu writes: “Since the amount of energy determines how much gravity something has”

        I know only two types of energy, kinetic energy and potential energy. Which one determines gravity?

        “and we have TWO objects interacting…”

        This is a huge and unjustified assumption. In order to make this assumption you must make several hidden assumptions. What are they? Are you talking in the framework of discarded Newtonian doctrines of force or Einsteinian theories which deny force and Newtonian type interactions.

        “the total force between them is obtained by multiplying their energies.”

        How did you decide that the total force between them is obtained by multiplying? Neither Newtonian doctrine nor Einsteinian theories tell us to multiply energies. And should we multiply kinetic energies or potential energies? Why?

        1. Mass could be considered potential energy, but as Professor Strassler has discussed elsewhere, there is just energy, and the difference between kinetic energy and mass depends on perspective. (A small bit of the mass of an atom of hydrogen is due to the speed of the electron moving around in it. We usually ignore this, as well as the much, much more significant kinetic energy of the quarks in a proton. And of course when we measure the mass of say, a ball, we don’t subtract all the kinetic energy of the atoms moving about in it.)

          All energy ‘determines gravity’; if you move something very fast, it gets a lot of kinetic energy and this means gravity pulls on it harder. We could even say that energy bends spacetime and that this is gravity.

          We need to multiply the two energies because both objects curve spacetime. Imagine taking two stars. Each has its own gravitational well, a ‘dip’ in spacetime. As we move them together, what happens? Their ‘dips’ begin to merge and spacetime ‘dips’ even deeper.

          So the basic assumptions are:

          * Every object with any kind of energy has gravity.
          * The more energy of any kind, the more gravity
          * Everything with gravity affects everything else with gravity (The sun pulls on the earth, but the earth also pulls back.)

  61. “static model, because he saw that this universe would not be stable. Even a slight pertubation would increase locally mass density” – that comment is intriguing…..

  62. ” he was not happy with it?”
    No, Einstein was not satisfied by its static model, because he saw that this universe would not be stable. Even a slight pertubation would increase locally mass density, so gravitionnal attraction, attracting more mass, which in turn would increase gravitation, attract more mas and so on. At the end, the universe would collapse on this local density fluctutation.
    Then Einstein found that he could mathemetically add a constant to his equations, which is known as the cosmological constant, and it had the effect of an anti-gravitionnal force, which could balance the density and gravitation increase.
    But it was not natural. After the discovery of the expanding universe (Friedman and lemaitre models, Hubble law), the cosmological constant was no more necessary, and Einstein said some years after that it was the biggest blunder of his life. See :
    http://www.space.com/9593-einstein-biggest-blunder-turns.html

    1. “pertubation would increase locally mass density, ” Thanks Alain! yes that is right – and why is my next question? and not because of his mass/gravity reasoning…still reading your text……..not finished yet – thanks

      Cosmological constant i would like to discuss with you next.

    2. ” he was not happy with it?” Yes Alain – that is more than interesting!. A very clever man provides us with some great tools but everyone argues – including him what the universe is actually doing. Then Hubble team find it is expanding – but how does this affect mass? Apart from just proving presumably a locally constantly expanding space framework? By that i mean if Space is expanding what is expanding from ( its own fabric ) ? Or does it infer New Space is actually being produced rather than the same space but just stretching? I hope i have explained this well enough.

  63. Is there any prospect that we are only giving consideration to the origin, quality and effects of mass at the loss of consideration to an absolute which may exist – and not really thought about it too much since Michelson’s dispute & argument with Einstein then he reinventing Newton notion that it does?

  64. It seems to me that the community of knowledge must recognise the fact that mass did not exist at some stage meaning absolutely no mass – on the basis that the higgs is necessary to give anything any mass in the first place? Or is my thinking off kilter? Zero mass > Higgs > Mass? or just: Higgs >mass first?

  65. The fact that Einstein re invented the concept of a background absolute after discounting it 8 years earlier – is more than interesting! It infers that something is missing, wrong or not correctly presented ?

  66. Can anyone advise me: If we were to realise mathematically a universe which does not have any mass in it i.e. in the case of the early universe would Einstein’s STR and GTR have any relevance? 2. How would one define that massless void dimensionally ? Euclidian space? or just a zero dimensional void – despite the fact it had a regular dynamic boundary?

    1. The early universe is far from a massless void, even before the Higgs field became nonzero there was some mass around.

      Assuming that you were the lone object in an entirely massless universe (since I doubt you would last long in the early universe.) then relativity would still matter, so long as you could differentiate between reference frames.

      If you were the sole reference point in a massless void, with no way to differentiate between reference frames (so you couldn’t throw objects away from yourself, etc.) then relativity might exist, but you would have no way to detect it. (Likewise the expansion of space, etc.) This is much like how we can’t tell the Earth is orbiting the sun from its surface.

      There would be a problem defining said void since all our measuring devices are massive. Mathematically we could define a hypothetical empty void as Euclidean I believe.

      1. Thank you for clear declaration. Am curious though whether any experiment thought or real can at this time say what those localized masses were? It seems to me that when we start with Higgs field or earlier as having a source of its mass we ought to have a falsifiable explanation of what that source was before it is theory rather than not even wrong stuff.

        1. Things get interesting the further back towards the Big Bang we go. Before the Higgs field was zero Higgs particles themselves had mass, as well as a few other particles that get some of their mass through other means (Top quarks and neutrinos are two examples.) Since there was so much energy at this time, these particles were being produced wildly and the B.B. was both hot and dense.

          And of course we had things like black holes and (hypothetical) strings and in general things were quite messy. we can even see the primordial ‘mass map’ of the universe in the Cosmic Microwave background; initial, quantum fluctuations in mass-density in the early universe were blown up to cosmic scales by (we think) inflation.

          Before this thinks get murkier, our current understanding begins to break down. We have a number of theories and thought experiments on what happens before this, but as yet it’s rather difficult to even figure out which ones may be right. (Recently inflation and the big bang have even come under fire!) Some have indeed suggested the universe started off as an empty void, a false vacuum or other, more exotic states.

          1. Umm… I don’t follow this comment. You say that top quarks had mass through other means in the early stages of the big bang, as did neutrinos — what are you thinking of?

            I should emphasize that what you say in your second sentence is, in any case, somewhat speculative. We don’t actually know this experimentally; we surmise it from our equations. We don’t actually know the history of the universe well enough to be sure. Also I think you meant to write “non-zero” rather than “zero” in the fifth word of that sentence.

            1. dear Professor – how do i find someone’s comment when they have replied to me. I click on reply and then i find i am somewhere in the universe of this engine it alerts me to reply which i click – then have to scroll up and down in the hope of finding it! what am i doing wrong?

              1. In the case of a black hole with a gravity radius of 50 light years

                What is the energy of that body?

                “A supermassive black hole is the largest type of black hole in a galaxy, on the order of hundreds of thousands to billions of solar masses”.

                Our sun’s output

                3.846 × 1026 watts, = and effective gravity radius of 4.545 billion km, give or take things we can’t actually see orbiting out further. And the suns diameter of 1,400,000kms approx

                So the energy output could be 1 billion x our solar mass above?

                Hence reach the radial distance of 50 light years or more?

                This is realization:

                !!! could this be true??

                If the Earth became a black hole it would have a diameter of about 0.017 meters, about the size of a marble. Our Sun would have a diameter of about 6000 meters, just a little less than 4 miles.

                1. By ‘gravity radius’ do you mean ‘event horizon’? Gravity’s reach is infinite, all that changes is the strength.

                  Gavity is also not related to energy *output* but the total energy *in* the body. Our sun is losing mass as it burns its hydrogen fuel, so t is slowly letting go of everything orbiting it. (Though that will get worse when the sun becomes a red giant and really begins to shed mass.)

                  If the sun became a black hole, earth would not notice, at least until it got rather chilly and dark. A black hole is just very dense, it does not have more gravity than any other object equally as massive. As for energy output, black holes are very good at not outputting any energy at all, and the bigger they get, the less energy they’re willing to let go of.

                  A nice page you might like is here, it is a quick calculator that will tell you the energy content of any piece of mass: http://www.1728.org/einstein.htm

                  What you may be interested in are supernovae, which can outshine all the stars in a galaxy, or quasars (black holes eating stuff) which can output all the energy the sun will ever produce in seconds or less.

    2. Empty (no matter) space but solution of Einstein General Relativity equations is what is known as ‘De Sitter cosmological model’. See :
      http://en.wikipedia.org/wiki/De_Sitter_universe
      It was puiblished as early as 1917, and was a joke model, aimed to show that the ‘mach principle’ (geometry is driven by matter content) was not the end of the story.

      1. “It was originally proposed by Albert Einstein as a modification of his original theory of general relativity” – Yes Alain thanks very much. But what troubles me is the fact that the whole world had adopted his GTR, – he wanted to modify it and the world denied such modification! I know he continued this course – he and the world were hung by the same GTR petard. Why would “he feel” the need to change a perfect law? Yes it was ignored – if we are discussing the same case. Then despite the fact that he was greater than the beatles fame why would he want to adjust something which was perfect? – he was not happy with it?

      2. Thanks Alain – I appreciate that. He thought the universe was static – yes. And his STR and GTR are based on it. That is why he had an ongoing argument with Michelson at that time – who claimed it was not. And of course failed to prove it was by his famous experiment – thinking the univese was moving like a wind – and in hindsight of course his experiment seems rather childish now – but at the time of course some strong arguments where ongoing. Thanks also for your reference. I will read it this evening. I hope this will be an end to my enquiry. Regards ewj

      3. Dear Alain & everyone, in reading and re reading the links you kindly supplied above – I have to say it leaves me a tad unsatisfied. De Sitter Universe model says: “It models the universe as spatially flat and neglects ordinary matter. The idea that it neglects ordinary matter ( i would prefer all matter ) leaves me more satisfied. But what troubles me is this particular model considers the universe to exist in 2 dimensions! Surely this is the first error? I like the idea that it is expanding like a loaf of bread, but this assumes it is pulling the mass which exists in it, proportionally apart. Which has long since been rejected – and quite rightly I am sure. Help with this point or anyone please: My mind is somewhat stuck on the notion that if the primordial big bang with it’s zero mass – just a very high energy density, existed as a homogenous orthogonal expanding framework void, ( synonymous to an expanding chicken egg – neglecting the yolk density ) which can only ( I summise ), exist as a unity dimension which only has on face. Think of the mobius ring for example it only has 1 side, neglect the paper thickness for a second. If this unity dimension ( I hesitate to call it a dimension but know no other word for it ). Indeed existed before it created mass – then surely it still exists? Remaining as a unity dimension – in fact the Primary dimension into which is added the later Euclian 3 + temporal time once mass was created in it! IF, not then it must have undergone a mutation? I hope I am making sense here? Please bear with me on this. The De Sitter example is not appropriate with this notion. IF, the universe does exist as a primary dimension surely it is providing the opportunity not only for mass to exist in it, but something far more important: It is ( could be? ) the very mechanism which determines the velocity of light? If we consider this just for one moment all other kinds of paradoxes and phenomena may potentially be unwrapped, such as the Horizon Problem, black holes, time dilation, length shortening and the ( very creation of mass itself or pre Higgs )! Returning to light to help with a visualisation: We know the velocity is 300,000kms. I understand that no object can move – including light unless it has a space to move into! Could it be that this unity dimension is invisibly expanding orthogonally locally ( Hubble zone ) at 300,000kms. Hence when we try to make a photon in the LHC for example move faster we only successfully convert that imposed energy into new proton mass – simply because it has ‘No Space’ to move into – except the Euclidian space? IF, this unity dimension is expanding around us can we think of it as actually creating New Space second for second at the currently constant velocity 300,000kms? An independent entity but limits the behaviour of the mass it created? Causing a proton to become an ‘Energy to Mass’ converter, conversely, a black hole a ‘Mass to Energy’ converter ( the other sense ). Caused by some kind of torque effect. An actively eating black hole spews out substantial gamma energy from its poles where the angular rotation & momentum is lower. But this is moving onto another point. Returning briefly and finally to the primordial big bang, at that time the applicability of Max Plank’s constant would have a different impact on physics on the basis that the expansion of the universe was >C! We now live in a universe where light has velocity of C and determines the cohesion frequency of particles in the Standard Model. Could it be that if this expansion should vary for one second this will immediately affect everything! Such as the value of E=Mc2 power transmitted from the sun and the disruption and cohesion of all atoms? Hence the Std Model of particles can only condense into a solid form from energy when the expansion of the universe has this value. Similarly life on earth can only exist providing water is liquid and in a biological zone from the sun. Not too hot and not too cold!

        1. I did not read entirely your post, because it was too long, but I notice this affirmation : “this particular model considers the universe to exist in 2 dimensions!”
          I don’t know where you read this, but it is simply false. Original De Sitter model is indeed a four dimensional (3 for space, 1 for time) model. See for example :
          http://mathpages.com/rr/s7-06/7-06.htm
          But perhaps did you read some related work ? De Siiter model got a renewned interest in the last ~30 years, as it could descibe the early universe just after the big bang. In some toy modesl, to describne some interesting proppertiez (string related models ?), for mathematical simplification, it was perhaps reduced to two dimensions ?

          1. Hi Alain – directly from the wiki link you sent me! It states the model proposes the universe is expanding like a rubber balloon? and the mass on it move proportionally apart etc? Maybe the editor was just using this to explain its function? It does not really apply to my point of view. Pity you cannot read my whole message – I would be interested in your view and anybody else here. Thanks

            1. Yes, rubber balloon is a 2D image for what happens in our 3D+1 universe. The space time structure is stretching apart like the rubber balloon surface, and galaxies are not moving themselves, like the points on the ballon do not move on the surface, but the surface between them is stretching. And if the distance double, the strectching double, which explains that the red shift of a galaxy increase linearly with the distance (Hubble Law).
              If the galaxies were really moving, with a kinetic energy of 1/2 M*v ^2, it woiuld require much more energy than any physical process could afford…

              1. I can’t entirely agree with the last line of this answer: we have no idea how much energy is available in the universe. However, more seriously, space can stretch so fast that the distance between two very distant galaxies can grow faster than c, which we usually call the speed of light. [That’s what makes rapid “inflation” of the universe possible.] It is impossible for objects to move, relative to each other and as they pass each other, faster than c; so we cannot attribute the faster-than-c increase in the distance between galaxies (which are already far apart) as purely due to the galaxies moving through space. It is necessary to understand this as space itself carrying the galaxies along as it expands. Space is a *thing* in Einstein’s view of gravity.

                1. Yes, you are right, the most convincing argument that it is not a proper movement of galaxies is that we indeed observe redshifts of galaxies greater than one (which if it was interpreted as a classical doppler redshift would imply a velocity greater than c). The greatest redshit that I kwow for quasars are 6.4. See :
                  http://www.astronomy.ohio-state.edu/~ryden/ast162_8/notes36.html
                  If classical doppler relation, v=cz, was to apply, it would mean a velocity of 6.4 c. But the relativistic formulae for doppler effect (with relativistic composition of velocities and angles) shows this velocity would be “only” 44.000 km/s, 1/6 of c.
                  But if it is not strictly impossible to find a physical process which could afford the energy needed to accelerate the galaxy at such a speed, it is very unlikely.But indeed, there are very violent processes in the universe…
                  For the cosmic microwave background radiation, we have even a redshift of about 1000 (from a teperature of 3000 K, where electrons and protons began to form neutral atoms, and the universe became transparent, to the present tempreture of about 3K).

  67. Prof. Strassler,

    Standing in an elevator that accelerates upwards, would the gravitational influence from the earth on my body, f.i. at the moment the elevator passes a certain height be different from the gravitational influence on my body in case the elevator stands still at the same height?

      1. Hi Matt,

        I am compelled to ask you a couple of questions if may please:

        1. A small ball central inside another ball which momentarily has far greater gravity than the external one: Is the gravity additive or subtractive? The inner ball in this case is sufficient to cause the instant collapse of the outer.

        2. Can you visualise a condition where the affect of gravity is caused by an object/things which do not have mass? Externally, it acts as it does but internally is zero?

        3. The subject of light is constantly discussed if the value of light speed should vary by a very very small percent with a very very small time interval how would we know?

        4. There was a recent post originating from a NASA page here it is reporting unusual occurences relating to the ‘ether and black energy’. They are short of an explanation. Can you visualise a situation where the universe could exist as 2 entities of time 1 constant and 1 variable.? And could be expressed as ut,x,y,z + temporal time where ‘ut’ is the additional dimension which is providing the time zero but is stretching causing the value ‘C’?
        thanks

  68. Prof. Strassler writes: “law of gravity and Newton’s law of gravity are completely consistent…”

    This is absurd. Newtonian gravity is based on an occult force that acts instantaneously, this force is an intelligent virtue that knows how much each particle has mass and acts accordingly, it is occult and supernatural…

    In Einstein, there is no such occult force acting between two indivisible particles instantaneously, in fact Einstein designed his theory to eliminate the occult Newtonian force from physics… As you repeatedly mention in this blog there are no particles in the context of General Relativity and gravitation… what you call particles are all waves. Newtonian gravity and Einstein’s conception of gravity are fundamentally contradictory, how can they be consistent?

    1. When the professor notes that they are ‘consistent’ he does not mean that they are the same or that their inner workings are identical. indeed he asks ‘HOW are they consistent?’

      Consistent here is shorthand for ‘Appears to be the same under certain conditions’ When you are dealing with small, slow-moving objects Newtonian gravity, seems to work. If you look closely you see that gravity isn’t instantaneous, that speed affects mass and so on.

      Except when dealing with finicky things like GPS satelites we use Newton’s equations because they’re easier and (very nearly) consistent with Einsteins, much in the same way we still talk about sunrise though we know the earth is going around the sun.

      1. Dear Kudzu,

        Any time you repeat official physics party line as you do here you feel self-righteous and confident that you are right and you feel good. But there is always the danger that you may be a Newton apologist believing Newton’s authority without question. I have no doubt that you would be the first to criticize the old scholastic doctors of philosophy who repeated Peripatetic party line as the only truth without thinking. I am worried that you are falling into the same error by arguing by authority, of not Aristotle of course, but of Newton. I think for myself and this is how I think, let me know if you agree.

        If Newton assumed that gravity was instantaneous and he used this occult gravity in his computations of orbits and got good results then we must agree that gravity is instantaneous. But you say that now you know that gravity is not instantaneous under Einstein’s authority. If physics were to be a science, Newtonian gravity will be discarded as promoting an occult force as a natural fact and Newton demoted for faking his computations by claiming to use force in his computations. There is no reconciliation of instantaneous gravity and non-instantaneous gravity. A contradiction can only be resolved by casuistry (a very popular type of argument in physics and in this blog).

        If you look at the Principia you will see that Newton did not use a force term in his computations of orbits, all his computations are geometric. Therefore, there is no such force in nature and it is absurd to talk about Einstein’s gravitation agreeing with Newtonian gravitation (which does not exist).

        Of course, the choice is yours, you can continue to assert Newton’s authority by repeating physics mythology or question Newton and physics.

        The rest of your comment also repeats several physics myths and it is worth answering word by word, and I’ll do that in my blog.

        1. It is possible I am a Newtonian apologist, it is something we must always watch out for, biases. But I should also suggest that it is also unwise to assume that I sit here feeling a smug sense of righteousness as I type out things I believe ‘just because’ All of us must be wary we do not fall into the trap of assuming we are right :p

          If you want a brief summary of my beliefs, and (dare I say) the professors, it is this ‘Newton was wrong. but he was nearly right enough that we can use his methods most of the time.’

          When we talk of things agreeing we very seldom mean ‘agree exactly’; usually there is an additional ‘within x%’ hiding away, even if we don’t say it. If you look at previous Higgs Boson posts you will notice the caution there ‘This *looks* like the Higgs Boson’ ‘The experiments agree with it being the Higgs BUT it may just be fluke.’

          If I toss an iron ball in the air I can calculate its path and speed using Newton’s equations and Einsteins. Both agree… within far more than a millionth of a percent. When I measure the actual ball’s path, my fumbling human limitations , not to mention air resistance, introduce far more error. Then all I can say is ‘Both theories agree with the experiment within x%’

          Theories, like people seldom agree 100%. 99.9% agreement is usually good enough if it makes things easier, and later on we can tell the interested ones ‘Well no… that’s not quite right…’

          Also, action at a distance is not an ‘occult force’; we know that entanglement can affect two objects any distance apart instantaneously, no matter how far away they are. (And a number of physicists were upset at this ‘spooky action at a distance’) There’s no good reason for something to not act instantaneously over any distance, and I suggest many people would welcome new physics that allowed such things. (Opening the way for faster than light travel.) An occult force would simply be something that refuses explanation. You will see a lot of this in homeopathy where the catch-cry is often ‘We don’t know how it works, it just does. Stop asking questions.’

        2. this is a really silly comment, zeynel, and its style is unpleasant. Talk about sounding “self-righteous and confident”!

          Physics is about making predictions, not about political polemics. Call these things whatever you want (in the Hamiltonian and Lagrangian approaches to Newton’s laws one doesn’t need to talk about forces at all) — in the end, language is used in physics as a tool to help us communicate about the equations and the experiments, which are what actually get used to make predictions. And everyone in physics knows that any given set of equations can be written down in different forms, so there’s no “Party Line” about which form is better than another.

          However, your statements about instantaneous and non-instantaneous are not issues for political discussion; they can be distinguished experimentally. And if gravity were instantaneous, rather than delayed as in Einstein’s theory, I am pretty sure that the slow-down rate of the Hulse-Taylor pulsar would be different from observation. Maybe a gravity-wave expert can confirm this.

          1. Professor – actually Parks road is that the college? I had an interview their once I am sure it was Parks road. Where Sir Philip Hirst was a HOD. Many years ago! the only bit i recall is Parks road.

            On the matter of gravity ( time delay ) i.e. the earth will spin off its orbiting rotation into a tangent if the sun suddenly loses its gravity some 8? mins after this catastrophic unlikely event because of the boundaries of ‘C’. Then should we live in a universe where the value Time is homogenous across it eg. ‘0’ or 1? then presumably both events would be 100% instantaneous as if space does not occur at all?

            1. I don’t think I entirely understand your question, but no, time is not homogeneous across space; there is no universal clock that we can all use at all places in the universe. Space-time is a pretty complicated thing, in Einstein’s view of it.

              1. Gentlemen – can someone advise me how to navigate this brilliant site? Comically – i have been fortunate to receive some communication but i have absolutely no idea where to find the article/return comment for continuity purposes – except the one which appears in the email incomplete as far as in understand. When i click reply it takes me to the top home page and then have to tour down through all the text to locate any return comment. sorry……what am i doing wrong?

              2. Sir, I have got it – the replies operate outside your fine website!! Sorry I have to wear spectacles now – and did not see the small print! I am so stupid – even I am astonished by my own stupidity. You are a professor of probably the world’s top university, and as a young man you worked extremely hard, after all that effort, now in one of the most respected educational locations. – ‘Then willing’ to make yourself accessible via your fine & homely website for people who are curious about physics. A great achievement and a brilliant idea for people to make real contact with a very knowledgeable Professor – fantastic idea and thank you – which I am confident you will be rewarded shortly. Which, services extended to ordinary people such as myself provides them an opportunity to ask questions to clear their mind of doubt, about the safe scientific knowledge, and hopefully other questions which are not on so scientifically safe. And forgive me I am in that classification of person.

                Please follow my curiosity: In the time of the primordial ‘big bang’ if there was such a thing, bearing in mind this idiom was the invention of a BBC reporter in the 50’s. Can we safely assume – Pre Higgs or otherwise that the ability of mass simply did not exist? So, if we imagine for one small second a void where it does not ( where that condition may only have been for 1 second nevertheless it still occurred ?) – would you think that what existed in that void can be explained as Euclidian space – small as it may have been? In the absence of any Einstein reference frames? I know this is in the realm of philosophical physics. However, I put it to you it may be a relevant consideration. Then whatever happened in that short lived primordial void ( 1 million linear years or whatever the linear temporal time number is ) was nothing more than a place and ability of mass to happen – with or without a Higgs field!

                So if one can follow that then surely there is an argument which may state that whatever made up that void still exists – just bigger ? 40billion light years bigger? But still a void – which now contains mass existing with its own independent set of rules. But the original void is still extant today but invisible to us as we only see the mass affects in it? As you commented – physics is about questioning then providing a set of mathematical rules to prove the question.

                I look forward to your thoughts – thanks.

              3. I am thinking maybe we exist within two time identities ‘simultaneously’ and not know it – and not be argued whether we have a case for one or the other – but find a way of proving such a combination. 1. Homogenous ( symmetrical constant time TC ) Newton’s absolute background notion and 2. temporal where mass must exist providing Einstein/Minkowski ‘temporal variable t ’ – which is alarmingly elastic – but with good reason. If you can think of the (TC+TV) existing alongside each other at the same event moment would mean we would have to exist in 4 spatial realities ( fearful of the dimension word ) if the claim is true. Not to be visualized as a ‘physically’ connected tesseract but 2 independent system species. – the primary one TC provides the homogenous time & space and ability for mass to occupy ‘any space’ and be created in the first place – with or without a Higgs transfer which would infer this to be a somewhat secondary consideration.

                The net result is we end up with a Space ( involving a New Time notion) built as a ‘ simultaneous constant plus variable’. This if I am not mistaken is a mathematical realisation. I can think of a visualization on this: in aviation (my meaning reference is Pi – which is everywhere in physics much like ‘C’ more or less determines mathematically our ability to boil an egg everyday ). An aircraft must follow a flight line horizontal to gravity for passenger comfort ( curvature of earth), which is achieved with the gyro. So it theoretically is ‘constantly horizontal and enjoying level flight’. But at the same time is actually following a constant curve – which is contradictory. So the constant is the level flight ( not bending ) but the variable is the curve in this simple example. This is the first way I can explain this. In reality of course the passengers are never in level flight for the whole trip – just a sensory illusion. It is not a paradox it is possible to be straight and curved simultaneously. Einstein proved that by his simple experiment with the stone in flight from the train. Its flight is doing 2 things at the same time. Reported by the two peoples arguments one on the train and the other on the embankment. Surely we can extend this sense of relativity to time itself by combining these effects? – which could conclude Einstein’s Unification dream? And possibly give reason to TV dilation as it approaches TC at close to ‘C’. plus resolving other numerous mysteries. Thank you.

        1. I read it! Yes it’s frustrating but presumably the occult forces will be resolved in the future as we cannot jump the gun! Comprehension is achieved by very small steps of validated rules – one generation at a time. In my thinking time exists as 2 components meaning a constant + variable, but no one is prepared to even consider it, as science like a government is voted in and everything becomes political.

          1. I am curious about that assertion; how do you consider time to be two component? At rest an object experiences a steady and irreversible flow of time, and this can be reduced to nothing at light speed, so it seems to me time is one component irreversible. (Though my particular belief is that time is not a dimension and in fact has no real physical existence.)

            1. FANTASTIC!!! Thanks – yes that’s right! also very much in my mind. I think possibly Minkowski put us all on the wrong track here! Time is a variable – how can you have a variable dimension? A box has 3 Euclidian sides one can vary any of its lengths but you can’t vary the quality of the dimension. It either exists or it does not. You cannot remove or modify x, from x,y,z and still call it a uniform box.

              TVariable + TConstant. The simplest way I can explain what is in my mind and NOT use pages of text is by the following expression: Ut, x, y, z + temporal time. Where ‘Ut’: is Universal time = ‘0’ or 1. And has a dynamic which is the expansion of the universe in the hubble zone of 300,000kms as defined by our thinking of temporal time. X, y, z is our regular 3D and observed variable time which can only exist if ‘Ut’ exists first – hence my meaning that this is our Primary spatial dimension. And why I am so curios about the applicability of STR and GTR in a matter free void as per the beginnings of the BB as a plausible route to prove this 4D. = TC + TV notion.

              Horizon Problem – is maybe a good visualization? The universe is some 40 billion light years in diameter – apparently? The temp everywhere is the same. Astonishing as that is but amazingly the same at its outer reaches across this huge distance AND impossible to achieve an INFORMATION based equilibrium by our current understanding of universe! BUT it does this is astonishing. IF, we have a time constant background in the universe then distance is irrelevant. Just in thinking of Newton’s Speed = d/t. If the time is zero or 1 the speed travelled is instantaneous. This is how the universe could possibly achieve such equilibrium in this example, which hopefully is an introduction for the 4D credence?

              Light speed – With ‘Ut’ TC = 0. As TV by virtue of a moving body through x,y,z as it approaches light speed it actually is approaching time ‘0’. An introduction to this notion is atomic clocks. The moving body is approaching phase time with the background constant TC. In order for an atomic clock to read zero on its counter it is obliged to move at 300,000kms. This is another introduction to the possible validity of the notion of a Primary dimension and its dynamic and we continually ignore it – or possibly not even thought about?

              1. That relates somewhat to how I believe the universe works; to quote someone who says it better than me:

                ‘The quantum state of the universe really does evolve in time — i.e. the Hamiltonian is not zero, it truly does push the state forward in time. … A wonderful and under-appreciated consequence of quantum mechanics is that, if this possibility is right (the universe truly evolves), time cannot truly begin or end — it goes on forever. Very unlike classical mechanics, where the universe’s trajectory through the space of states can bring it smack up against a singularity, at which point time presumably ceases. In QM, every state is just as good as every other state, and the evolution will go happily marching along.

                As the quantum state of the universe evolves, it can pass through phases where it looks an awful lot like “nothing,” conventionally understood — i.e. it could look like completely empty space, or like some peculiar non-geometric phase where we wouldn’t recognize it as “space” at all. And later, through the relentless influence of the Hamiltonian, it could evolve into something that looks very much like “something,” even very much like the universe we see around us today.’

                In short, there was a time before the big bang where there well could have been no space. It’s speculative at this time, but I hope it’s true. (It’s my pet hypothesis.)

                1. Interesting – now on the trail of Hamiltonian vision.

                  Incidentally I have printed a book March 2012 near and around this subject ut,x,y,z. No one will be the least bit interested in it. But at least I have these ideas on paper. I have followed your reasoning thank you.

                  Why can we not have 2 Spaces simultaneously after all we have 3 dimensions in one of them? Then Newton and Einstein are both correct? Then should Einstein have had the notion that we exist in 2 spaces then perhaps he could have had a successful outcome to his search for the Unification???

                2. I am not entirely sure what you mean by ‘two spaces simultaneously’; I think it would be quite difficult to have two things occupy the same space, especially if the things *were* space. Putting aside multiverses aside, possibly you mean something similar to ‘branes’; 3\4D spaces embedded in a higher dimensional space, also known as part of M-theory: http://en.wikipedia.org/wiki/Membrane_%28M-theory%29

                3. To get across a river I need the ability of stepping stones. I cannot jump! The concept of parallel universe may well be applicable to the intuition of 2D of time. We can mix oil and water together forming a homogenous liquid ( x2 space-times )so they occupy the same space but they still exist as separate materials. ( forgetting the ability of chemical emulsions for the moment). Rather than parallel universes I prefer to think of an artist’s palette. The board is only providing a means onto which the paint maybe applied. ( matter in this case ). But in this instance the palette is actually creating the paint!

                4. So are you speaking of two dimensions being part of the same space, or two different, unconnected spaces occupying the same location? You say they mix which suggests to me dimensions, much like our three spacial dimensions are mixed into the three directions we know.

                5. Hi Kudzu – I have to be very careful which basic physical descriptors I use – in fear of creating confusion – then later change the way I describe it because I have found better descriptors. When we think of dimensions, like time ( in my mind ) We are somewhat fixed in our interpretation of them, and these interpretations are not necessarily clear anyway. Although concept of a dimension is clear and simply means size. I suppose my intuition could be more to do with metaphysics than physics as we are obliged to use valid physical terms which mean things. Forgive me I am sounding a bit like politician here!!

                  Intuition of the primary parent of ‘space + time’:

                  My meaning essentially, Is we have 1 primary parent unit of space + time and it is has a construct in 2D. ( which infers size which I am actually not too bothered about as relevant as it infers measureable distance ). One may visualize it as a 3D boundary + Time symmetrical isometric internal framework. This 3D space is ‘confining the captive unity of time’. The boundary in this case has a dynamic. I have no proposals for measuring that dynamic other than it is expanding outwards. But as a result of that dynamic it causes the isometric framework to stretch orthogonally at a velocity of 300,000kms. The outcome of this stretching means that the parent is creating ’New Internal Space constantly. This differs from our understanding that new space is being created at the boundary! My meaning is that space it is being created across the entire diameter in the isometric.

                  The offspring framework ‘spacetime’:

                  I use the term offspring, and not parallel. Humanities experience of spacetime is the offspring. It exists within the framework of the isometric parent. Both are valid. Occupying the same space and boundary. ( Hubble zone only in this instance = ‘C’ ). The subsequent creation of matter within the parent causes it’s unique identifiable Space ( spacetime ) and it’s relevant dimension = Cartesian/Euclidian zone).

                  Now you may feel jelly bubbles as I did when I first had this notion. Nature does things in a very regular way which contains irregularities. ‘A baby is created within the confines of it’s mother’ – I think that describes the meaning best!

                  Yes they occupy the same space, as 2 separate entities.

                6. I see. I would describe that as being two different aspects of the same space, so I guess that’s where the confusion sets in with me.

                7. Sure, description is one thing visualization is the other. The challenge is have I provided a visualization for you, and could you describe it back to me using your description. That way we both can confirm a common understanding?

                8. “In QM, every state is just as good as every other state, and the evolution will go happily marching along. As the quantum state of the universe evolves, it can pass through phases where it looks an awful lot like “nothing,” conventionally understood — i.e. it could look like completely empty space, or like some peculiar non-geometric phase where we wouldn’t recognize it as “space” at all. And later, through the relentless influence of the Hamiltonian, it could evolve into something that looks very much like “something,”

                  Could it be a simple reciprocation of ‘Being Able’ ( the realization ) of one state then reverting into another like a light it is either on or off or 1 or 0. at the moment 1 ! It can switch the ‘1’ state ‘on’ where matter can form. When the light goes off – all of the light goes off not just a part of it !! At the current time the light has been on for 14 billion years ! A big crunch this could only occur in the ‘1’ state – and should it start to collapse it may not complete its collapse because suddenly the entire state be switched off again???

                  In short the primary spatial provides a means for matter to exist or not centered around the number of 300,000kms. Things move with ‘0’ time or a variable if 1 is on!

                9. We should not think of this Hamiltonian as being something that can instantaneously affect the entire universe and just ‘turn off’ matter; it must evolve, that is, change gradually and logically from one state to the next. The big bang seems like quite a sudden change, but it started out small and followed from there. So no need to worry about mass being ‘switched off’

                  It seems however our universe will not ‘big crunch’ but will keep expanding faster and faster to a ‘big rip’ where it will become essentially an empty void (once more?)

                10. I going to study Hamiltonian theory….it sounds like I am naively promoting someone else’s idea and not know it? Having said that I am using Newton’s on the basis that space is an background absolute, I am also using Einstein’s declaration that it isn’t.

                  Yes, then on this subject I am also troubled with another notion where the Big Rip could actually be a huge expanding concentric hole caused by the centre sheering then the universe at that stage actually becomes a flat ring? Which continues to expand and become a thinner and thinner ring. And here on earth in the night sky we see absolutely nothing everything has vanished ( we are permitted the solar system but that may rip as well – probably ). But we continue to remain part of that rarified system. We continue to expand but curve back on ourselves and all our space partners reappear and get alarmingly closer. Which infers ( standard geometry of curved space ) that this system commences its collapse and finally meet up as a single lump of matter! I prefer not to think as another big bang, I prefer the switch off principal – just in thinking about Planks theory and caused by the universal adjustment to the value ‘C’. Although that is not what he said – I am just relying on his theory as a mechanism to achieve the switch off process. So in thinking of the Ancient Egyptian ‘Duality Principal’, we have 2 futures. 1. Is the rip and collapse and or 2. The switch off.

                  Evolution of anything is provided by the ability of linear time !?

                  I am worried now about my possible naivety and have to study your Hamilton reference, unless in your greater knowledge advise that whereas, these intuitions are not strictly safe – but could be plausible?

                11. The big rip does not occur at one point in space, nor expand out say at light speed, but since it is due to the expansion of space (and thus all space) itself, it occurs across the entire universe at once. This would make smaller and smaller structures unstable as the expansion increased, first galaxy clusters would fly apart, then galaxies, then stars, until eventually atoms themselves would not be stable.

                  Your ‘loop theory’ sounds interesting, but I cannot see it working as the expansion of space tends to make our universe’s spacetime flatter, as it accelerates spacetime becomes indistinguishable from a perfectly empty and flat universe whereas what you describe would seem to require a closed (curved in on itself) universe where all matter was moving away from one point (To curve around on itself and all meet at another point ‘opposite’ the initial expansion point.)

                12. It’s all conjecture but fun – the periphery of the universe is supposedly moving away faster than we are locally expanding at ‘C’. We don’t really know where we are located in the big picture – centre of to the left up or down? As the periphery moves away it is causing us to get flatter as I understand? Flatter and flatter! Until it becomes a disc anyway I guess? Then as this flat disc continues to expand the diameter – yes the rip may be catastrophic degeneration of things? But presumably what happens to the disc which now has no matter in it still exists even tho we don’t? IF, you uphold no matter no space then the whole lot goes! Agreed.

                13. The periphery of the universe is an interesting subject; according to inflation most of the universe is still inflating madly, expanding far faster than c. (The ‘calm’ patch we are in where inflation has ended can only expand through new space at a rate of c, this is a disturbing feature of inflation.) Eventually of course more and more of the universe will be moving away from us faster than c. And our position in the universe, if that’s even meaningful is not known, there’s no way to tell.

                  It’s not the periphery moving that makes space flatter, but the expansion of all space, diluting matter. When the universe was young, it was very dense, and all that matter curved space a lot. But as it has been expanding there is more and more flat nothing between the matter in our universe. As the expansion speeds up more of the universe will be empty space until it’s almost impossible to distinguish it from a truly empty universe.

                  Our universe will never become completely flat, all the matter it has now isn’t going anywhere, it’s just being diluted like a drop of dye in a swimming pool.

                14. Dilution. The distribution of matter in the universe, how it is arranged is mostly being controlled by gravity now, which is clumping together matter into stars, star clusters, galaxies and so on. But as this happens the average density of matter in the universe is decreasing thanks to the expansion of space.

                15. Forgive me in advance – this is not a glib response, nor a religious one. When the baby has spend months concerned with growth and cohabitation with its mother – it must be born or both will peril – all species including egg makers – excluding infinitesimals – they are always a problem! – that is everyone’s observation of life, we are defined by the ‘Big Rythym’ – of nature and we cannot ignore ‘any’ of its clues to get closer to a meaning not just the stars ( glamorous as they are )- that is only a part of the picture. Stepping back from the picture gives us a different view. Looking at one square inch of it how can we ever now it the picture is a portrait or a landscape? Other than to learn is comprises oil or water paint? Then spend a long time arguing if all the painting comprises one, the other or both?

                  Nature does what it does – and it’s patterns are everywhere – Darwin had his ideas and we hung our coats them for a long time. If matter is diluting ( privately that is ok ) we are ageing. That in itself infers a history – and beginning. Whether it be the BB or not. And in your earlier reference ‘ Hamiltonian’ then he is correct. And evolution must occur. This is sounding so pedantic – I’m really sorry.

                  In all of us there exists our own unique energy, which in part makes what us all individuals. In the entire evolution of homo sapiens that too has a common energy. Driving it to develop and evolve in its unique characteristics.

                  In my mind if matter is diluting that would seem a perfectly logical outcome of what it is doing, whether we understand the physics or not. Returning to my eccentric example at the top – if the universe needs to remove us because we are too old – that is what will happen and this is the way it happens. And our experience is concluded. But, I am also privately ok with the idea that this experience is not unique. Furthermore, I am also privately ok with the idea that all information has been recorded, every atom, phases of evolution, and every universe and its unique experience.

                  In the duality : Why?

                16. 1. “ Virtuality” , the quality of having the attributes of something without sharing its (real or imagined) physical form.

                  2. Photon gravity: If a black hole gravity field is so strong that light cannot escape itself, light must have mass?

                  3. If photons are the cause of a BH gravity, and yet have no mass. Then the affect they have on orbiting bodies is based upon it not having any real mass. But one which is created only by their dynamics.

                  The subsequent density is not real its affect is?

                17. I see. Firstly you must remember, gravity acts on energy, not mass. Photons have energy, so are affected by gravity and cannot escape a black hole. It may be easier to think of black holes geometrically, as warps in spacetime. If a hole is too deep, nothing can get out of it.

                  I think this is what the professor was referencing when he noted a black hole can be made of photons (Thus have no mass.) but still have a strong gravity field.

                18. , “gravity acts on energy, not mass”

                  Yes thank you I must keep this fixed in my mind. My knowledge on this is muddled. I am easier about the visualization and the 2D warp and may in fact be infinitely deep?

                19. That’s been the traditional way of looking at it, especially in the media. But it doesn’t need to be infinitely deep, just deep enough to trap lightspeed particles. Imagine a (non-flying) bug crawling on the surface of a balloon; no matter how much it crawls about it can never get off the balloon, that’s just the geometry of things. The balloon doesn’t have to be infinitely small (And thus infinitely curved.) Just as long as it’s round.

                  Likewise a black hole doesn’t need to be infinitely dense or curve space infinitely, as long as it curves it *enough* that light cannot escape it.

                20. Agreed. I like your bug Idea on the balloon. The balloon in this case the invisible universe fabric and he is only concerned with his own 3D universe as he carries out his business stuck in it’s 2D vacuum surface. The bug doesn’t realize yet, the existence of the plane of time which the whole of it’s body is immersed by function of the balloon’s existence. He wants to get to the top of the balloon and is only concerned with measuring his own relative time as he moves. His case is hopeless because the balloon is enlarging far faster than his ability to travel, but he is not too bothered because everything appears constant immediately around him. He notices strange things happening as he moves. Then refers them as Warp, Ether, Entanglement and Gravity because it is beyond his ability to imagine what can cause these observations. Although, he is more satisfied with gravity because it is something which he can measure. And he continues with his journey. Then he muses if two or more things are instantaneously synchronous as there is zero time for information to pass between them. So he uses his imagination and concludes they must be in the same place ( not even contiguous contact ) – a plane of time where measureable distance does not exist. But he remains confused because he can measure in his 3D world the distances between them.

                21. “Well I assumed that the black hole was incredibly dense mass, which it isn’t and to be frank I don’t know what it is. Some say it is an infinitely tiny point, which would thus be spinning infinitely fast, yet still have the star’s finite rotational energy.”

                  Indeed, this was my initial realisation a few days ago until we started exchanging ideas, As a result of information thanks to you & Matt I have modified my cognition. I now see them as massless entities but because of their invisible internal dynamic, they are only creating the appearance of a mass which simply does not exist. It’s only created by virtue of their massless internal clockwork. One could say an external physical illusion but its affect is more than real. Thanks

                22. Indeed, but I would be careful there too; I always thought that information was destroyed in a black hole, whatever made it didn’t matter, but it seems I am wrong here; if mass goes into a black hole, the black hole has that mass, somehow. Do not not ask me how this works, I don’t know myself, but apparently when a black hole evaporates away via Hawking radiation, it releases everything that made it. (And only that.)

                23. Synch notion of suns.

                  Actually the stars are in synch in a way caused by the uniformity of the value of ‘c’ through them all . Hence, any non uniform energy production ( sun spots ) – is my interest, as this presumably could infer the non uniformity of ‘C’ – and not simply an occult value – which is a phrase I have just learned! So if there is a small blip in the Ut TC expansion it affects everything simultaneously, and the only way of detecting it is via energy producing solar masses ( excluding nuclear power stations ) ?? Or just leave the LHC running for a few decades at full power and see if there is a reliably detected variation in proton orbital time?

                24. The problem with that is that stars are different. Imagine I had a large stove with pots of water boiling on it. Some are small, some large. Some I have just put on, others have been sitting there for hours. Now suppose I ask ‘Since the water in them is all the same as is the stove heat, are they in sync?’

                  Each pot’s boiling is partly random, partly due to water and heat and partly due to the pot’s size\boiling time. I can’t expect to see any synchronization of the bubbles in even very close pots, (if I did that would be big news) or even whether pots close to each other had boiled yet or boiled dry.

                  If the stove heat was turned up or down, synchronization again wouldn’t tell me. but if I had enough pots I would be able to work out how they boiled and what they should be doing. Then if the stove heat changed I would notice pots were not boiling as they should and would be able to calculate just what had happened.

                  The value of c, as well as other ‘universal constants’ affects many things, and we have many stars and galaxies to observe. We have looked out at the universe and determined what a star is made of, how old and heavy it is, how massive and more just by looking at its light. (For example, every element emits a certain spectrum of light, sodium gives you the orange in street lamps say, and by looking at starlight we can thus see what elements are there.) This has allowed us to ask ‘Is this constant different in that part of the universe?’

                  For the most part we can be sure the universe is the same billions of light years (and actual, time years) away as it is here, with more than 99.9995 accuracy. Section 6 of this Wikipedia page gives a brief overview of how the ‘fine structure constant’ has been measured using distant start-stuff and claims that it may have been a tiny bit different in the early universe: http://en.wikipedia.org/wiki/Fine-structure_constant

                25. Sorry – lot of little messages ….or maybe that information is already available from Nuclear power stations? I wonder if they can account for any history of unknown or unaccounted for synchronized drop in power . i.e. France -1.2 watt, UK -1.2 watt, Russia -1.2watt, USA – 1.2 watt etc: on the 15th March 1964 @ 1235.8593 hrs ? Since 1952 no matter how small, if all the NP stations records can illustrate such a synch drop then that would be more than interesting would it not, the ice cream would be the looking at sun records and if also in common synch!! ‘Wowee we have a primary dimension.’ on the basis of invisible information passing between them. There would be no way of getting out of that one!

                26. “here is where the kinetic energy begins to approach and then exceed the ‘rest mass’ energy (and I believe that spacetime itself is dragged about by a large rotating mass.)”

                  Your earlier comment above. So in short does this mean the kinetic energy can produce more gravity than it background rest mass?

                27. Yes, since gravity depends on energy and you can have as much kinetic energy as you want, any object made to move fast enough will have more kinetic energy than mass energy. (And for massless particles like photons all their energy is their momentum.)

                28. Excellent news – thanks. But in our earlier we discussed the relationship between variant and invariant mass. So we can have situation that the mass increase is trivial but the kinetic increase is massive! So the current thinking black hole centres are photons with huge momentum – gravity generators! So now what is the link between gravity and time?

                29. You may have misunderstood something; energy and mass are equivalent in many aspects. If I take a proton and accelerate it, it will get more energy and thus get more massive. Any object moving fast enough will be getting most of it’s ‘variant mass’, as you put it, from its kinetic energy, but it will act as if it weighs a lot, not as if it weighed the same as when it started.

                  For example protons in the LHC are as massive as an entire atom of tin, so only a small amount of their energy is mass, but gravitationally they act quite a bit heavier than their rest mass would suggest.

                  And everything with energy generates gravity, that’s one of the unique things about gravity. Black holes are just very dense and small so you can get very close to them where the gravity is very strong . If the sun became a black hole the earth would still orbit it exactly the same, but you could pilot a spaceship within a few miles of it, whereas the sun as it is now you start sinking *into* it at about half a million kilometers away. (Once you sink into an object gravity decreases until right in the middle of it you are weightless, roughly speaking.)

                30. “You may have misunderstood something; energy and mass are equivalent in many aspects. { I understand that from the energy mass equivalence expressions } If I take a proton and accelerate it, it will get more energy and thus get more massive. { I understand that – and potentially infinitely massive – providing availability of infinite energy } Any object moving fast enough will be getting most of it’s ‘variant mass’, as you put it, from its kinetic energy { Yes agreed } , but it will act as if it weighs a lot { Yes understood } , not as if it weighed the same as when it started { Understood – I am ok with that } For example protons in the LHC are as massive as an entire atom of tin, so only a small amount of their energy is mass { Understood } , but gravitationally they act quite a bit heavier than their rest mass would suggest { Yes understood – meaning their new kinetic energetic state ?} . And everything with energy generates gravity, that’s one of the unique things about gravity { Yes I understand }. Black holes are just very dense and small so you can get very close to them where the gravity is very strong . If the sun became a black hole the earth would still orbit it exactly the same, but you could pilot a spaceship within a few miles of it, { The hierarchy problem? } whereas the sun as it is now you start sinking *into* it at about half a million kilometers away.

                  (Once you sink into an object gravity decreases until right in the middle of it you are weightless, roughly speaking.) – Yes I understand , if I drilled a hole right through the earth I would accelerate initially at 9.8m/s at the surface some 18000 miles from the centre, and as I fall to that centre the force on me would be ( reducing ). So I am actually slowing down! the further I fall. I used to think that I would wiz past the center due to the momentum – and then arrive at the opposite face – a few metres short – so I could get from one side of the earth to the other without using an aeroplane !!

                  Actually a few years ago I had an engineering project in the Cumberland mine – it is the deepest mine in Europe some 5000 ft vertical shaft – if I recall correctly. On the basis that I am closer to the centre at that time should have I weighed myself – then the scales would have been contrary to the surface scales?? Apart from that it was an enlightening experience going deep under the biological skin of the planet!

                31. The heir achy problem has nothing to do with black holes, at least in the way you’re thinking.

                  If there was a way to drill a hole through the earth and stop air getting into it, and you jumped down that hole, you would indeed be able to ‘whiz past the center’ all the way out to the other side. What gets less on your is the force of gravity and thus how *fast* you accelerate, not your speed. If you had air in your earth-tunnel, that would slow you down quite a bit so that you would quite quickly stick in the center.

                  The earth’s radius is 6 thousand kilometers or so, even your mine doesn’t get 10 km down. (And interesting note, the rocks at the bottom of your mine still have bacterial life in them!) meaning that you’re not hitting even 1% of the distance to the earth’s core. A little rough math tells us the scales, if very accurate would be different, but even sweating, wearing different clothing, being dirty or clean, etc would change your weight by more, not to mention the fact that your weight on earth varies all over the place (Less at the equator where centrifugal force is strongest, greater under masses of dense rock or closer to sea level and so on.) Indeed, the differences of gravity across the earth can be used to make useful maps.

                32. I understand. Yes and sherpa tensing on top of everest? 30,000ft? could float away in slightest breeze!! Comically.

                  Actually the mine yields potasium and sodium salt. The strata is pre Devonian??? – a time before life. I may be confusing dates and geologic times – I knew the details at the time. The engineering techniques were extremely simplistic – so simple made me sweat with fear. I am not sure if bacteria could be present in pure salt in that era?? Could that be possible? Also – ref osmotic differential etc. I am sure it was 5000ft. vertically down and yes momentarily semi weightless. They could not drop me into the hole but the next best thing was the feeling of floating inside the shaft elevator. It was Boulby mine not Cumberland mine – sorry about that – anyway the link below.

                  Is there really bacteria so far down in life supporting material? How far down can it be found? That’s amazing….

                  Should I indeed get stuck in the centre then the first 1000 Kms I could probably jump! On my return to the surface! Comically.

                  http://en.wikipedia.org/wiki/Boulby_Mine

                  Now looking at the heir achy problem

                33. What is interesting to me is that the bacteria that seem to be in all the rocks we look at to quite a remarkable depth. They have spread there instead of being buried or dating back to the time the rocks were formed. This article gives a layman’s introduction to recent developments in the field of ‘deep biology’ Basically, if you can get any kind of water diffusing into rocks, it seems you’ll find life. http://www.sciencenews.org/sn_arc97/3_29_97/bob1.htm

                  The salt may impact this, but from what I’ve seen of life and the ubiquity of salt domes I’m guessing something would survive, even in salt.

                34. Time

                  TC + TV : Yes my meaning is co habitant and co existent, but I make statements which I am not sure about!

                  & Universe

                  The problem is, if the above is plausible, that is not the end of the story. The ancient Greeks where truly a Great race. In this sense two of it’s great philosophers where at constant discord regards the implication of dualities ( I can also think of Newton and Einstein here ) . If one is at a point where one felt uncomfortable about an understanding or analysis of something, one could imagine or propose the opposite – by extending the imagination. Then potentially establish a place of cognitive comfort. The problem with that is that this new place of comfort ALSO has a duality! So it is endless, by this they meant no matter what you think is a solid solution is tearfully nefarious – that solution has another expressible form and so on….that is really wicked! which means we could never understand anything fully!! Hence my earlier meaning of stepping stones. This concept could be extended to String theory, what is the duality form of this new notion?

                  Black hole & Gravity Fun thinking:

                  If we made a tunnel through earth as per our earlier, and as you say we evacuated 100% gas atoms. We sealed off its 2 ends 100% hermetic. Then released a heavy ball ( say equivalent tonnes of the earth nice big mass to make it interesting, and the Earth is not rotating ( angular momentum )). ( Ignoring the fact that they may want to instantaneously collapse into each other ).

                  The heavy ball accelerates initially away from the surface. It gains momentum. It then enjoys a very long time of reciprocation, returning to its original point of release etc. Going up and down passing the centre where there is zero gravity, and the ball becomes weightless momentarily. So the ball will have more gravity than the centre as it passes. Then from the balls point of view the ball will be trying to collapse the Earth !? What would make of this situation?

                  So in a black hole if we have a colony of atoms ( or sub atomic species ) they are all subtending to a common internal gravity. The entire colony of particles all pass the centre and transmit this gravity to the common centre. Surely, the outcome is they are all participating in the creation of the final quantified gravity of the entire system? It is not a perpetual gravity engine!? = the more particles = more gravity. Or less particles but greater internal velocity??

                35. In your ball example, the interesting thing is that any two masses always exert the *same* gravitational force on each other. You are exerting the same gravitational force on the earth as the earth is on you. (However while the Earth’s force is concentrated on small you, your force is spread over the entire earth.) What matters is the unbalanced force. At the center of earth there is still a lot of gravitational force, only it is all pulling in different directions to give a net unbalanced force of zero. (The earth will be trying to pull the ball outwards in all directions and yes, the ball would be trying to pull all of the earth inwards in all directions.)

                  With a black hole things are different. This is because of the vent horizon. When a piece of matter falls into a black hole it is not entering a solid body like the earth, the video link I sent you previously outlines two theories as to what may be happening, but suffice to say the mass does not move to the center of the hole and keep going out the other side.

                  However in both cases gravity adds; when we toss a ball into a hole in the ground it becomes part of the earth, its gravity adding to the earth’s like two drops of water merging. (It only adds smoothly if it’s right at the center of earth of course, if you’re standing on top of the ball buried just below the ground it will have more effect on you than any other piece of mass.)

                36. Yes very interesting – .

                  But the ball has momentum so it has more gravity on arrival at the centre than the earth and they have the same starting mass? Does that not mean the it cancels the earth gravity? – it becomes negative?

                  I love idea of entanglement and I am sure we have lot more pages to turn on this subject. And sorry to harp on about TC + TV. But in my thinking the information in order to have entanglement is by virtue of TC.

                37. Assuming we start with the ball stationary at the center of the earth, it has gravitational potential energy, energy that is converted to kinetic energy as it falls. This is why it is so hard to lift a piano upstairs, you need to give it a lot of energy to lift it away from the earth. So when the ball is at the center of the earth, moving fast, it has no gravitational potential energy, but lots of kinetic energy (And thus momentum.) so it’s gravitational effect does not change. (This is another time where we see energy and mass muddled; stored energy is usually ignored, since it is so small compared to mass energy, which is nearly the same as treating it as mass energy. So many people think that energy can come out of nowhere.)

                  Gravity is never negative (that we know of), it always attracts everything. The ‘earth’s’ gravity is itself made up of the gravity of all the rocks, trees, water and so on that compose it. So adding the ball to the earth for anyone not very, very close to the ball would be the same as making the earth a little bit heavier. At the center of the earth there would be a (near) perfect balance, the ball’s gravity field would mix symmetrically with the earth’s and it would be impossible to tell the two apart. Far before then though the difference would be too tiny for anyone to measure.

                38. Thanks – I need to learn more about the dynamics of gravity generation otherwise I am going to irritate you which I don’t want to do. Ok so it is additive to the outer system.

                  But should the inner ball cause the immediate collapse of the outer? Is it still additive?

                39. Simply put, anything will attract anything else towards it, so you can often see what will happen by drawing the system and then drawing lines between the bits of the objects involved.

                  The ball would be trying to collapse the earth, that is, pull all of it inwards towards itself. (But we’re already imagining an earth that doesn’t immediately collapse the hole in it because of all the earth trying to collapse itself.)

                  And if the ball is in the air, falling towards the earth remember that the earth is also falling towards *it* (Both are pulling on each other.)

                40. Yes . But I changed the rules in this a little and didn’t explain – my thinking was the hole exists ( quantum hole was in my mind actually but I don’t want to go there for a minute as I just need to fix the big picture first). The earth has it’s hole a particle comes along and flies through it ( under the original hypothetical conditions with its ends open now).

                  At the centre of the hole where it passes the earth wants to collapse and at that moment is the net gravity still additive or subtractive? I suppose additive because it realizes it is now more dense than it was a second ago so is obliged to get smaller. The 1 small dense ball will pump up the gravity of the outer ball, then everything external to the outer ball will also be pulled in closer.

                  Then if we have zillions of balls all the same all passing the same route like a machine gun bullets as far as the external matter is concerned its gravity is now increasing by the second, because the train of balls are moving so fast and they cannot see them and are curious why its gravity is changing so dramatically? Its gravity is not real in the sense if you measure its normal conditions. But as a result of this activity it behaves like it should be bigger or more energetic.

                  Then if we take our finger off the trigger the gravity returns to normal?

                41. Assuming that we’re firing a line of really fast bullets through the earth (say a beam of neutrinos.) the beam has its own gravity, pulling things towards its center. As more and more balls enter the earth yes, gravity increases as the gravity of the beam ends up int he same place as the gravity of the earth. Eventually the bullets will pass the center of the earth.

                  If they whiz out the other side then they will take their gravity with them away into space. (And if they were really massive you could be dragged behind them by their gravity.) If they stay then they’ll be part of the earth becoming part of it and part of it’s gravity.

                  (Think of the asteroid that wiped out the dinosaurs, it had its own gravity and it went pretty deep into the earth when it hit, but now it is part of the earth, its gravity in the same place as earth’s gravity.)

                42. Ok that’s good thanks. Incidentally – looking at the picture of the globe and the Gulf of Mexico 1.6M/kms area. Do you not think its shape is rather interesting and for a site for the big killer pair of asteroid’s arriving in the same moment?

                  So in a black hole could we? Have a scenario where our bullet photons are passing the epicentre continuously from all 360deg , like a poloidal smoke ring? This poloidal has a small inner diameter, so small one could think of it as a closed hole. Could this then cause our virtual gravity? Being poloidal provides the opposing poles. There is nothing to drag away because there is nothing to drag away? Or the object so small the photons or any other particle cannot get close enough. In any moment the entire field is enjoying a time of absolute zero. On the basis that the photons are moving at ‘c’.

                  If that could possibly be the case then it is a Time Zero Portal! Then thinking about the Russkind firewall the opportunity for ‘instantaneous & synchronous’ ( so called entanglement )behavior across it – passing from 1 time frame into the next. The event horizon being nothing more than a time interface.

                  Then thinking about the LHC – maybe as a research tool it is not complete, in the sense that we have only cut a slice through the poloidal. If we completed the entire structure into one where 1 quadrant of each ring formed a unity manifold with all the others. So as the protons was orbiting it’s own ring it would meet in the common point as all the other rings. They would not collide as their relative velocities are zero.

                  What is your thinking on this ?

                43. As far as I am aware only one asteroid hit to wipe out the dinosaurs.

                  I am also not aware of ‘virtual gravity’; gravity is gravity is the curving of spacetime. Other things can act like gravity (Acceleration, attracting opposite electrical charges…) but are not gravity.

                  And I am not sure what you are asking about the black hole and poloidal; a drawing or some equations would be very enlightening.

                44. Thank Kudzu,

                  Actually I did post a hypothetical schematic – but I can’t observe it anywhere in the historic train of comments here. I am not sure where it went. I will try again, I am aware of the other ‘gravity’ affects as I used electrostatic to agglomerate <10mu particulates vented in the exhaust from glass production furnaces. Simply by causing the exhaust to follow two separate exhaust routes. In each of the arms the particles where transit through opposing electrostatic charges. Then the exhaust arm converges into a single exhaust. Then the opposite charged particles agglomerated. Making them easier to capture. This was subject to novel research I was conducting in Bologna and Murano. I conducted a similar trials with British Glass where I injected charged particles of larger waste particles into the upstream exhaust..and so on.

                  If, we had 100 LHC’s so their rings are joined forming a big donut ( torus ). On the inside of the donut where all the particles are passing simultaneously they will all be at the same speed respectively. But the concentration of particles could be substantial and compacted into a small size. Because they are accelerating moving towards the centre – would this cause a gravity effect? 1. Where they coincide or within the torus manifold ? or, 2 within the torus itself? Interested to hear what you think..

                45. The direction a particle is accelerating has no effect on the gravitational force it produces. However it would feel to it as if a large mass were in the center, pulling it inwards and holding it in ‘orbit’ in this ‘hyper LHC’ you envision. The gravity of the particles would however be outweighed by the machine itself, which would have an interesting profile. (Assuming you stacked a whole lot of LHCs on top of each other in a long cylinder.)

                46. Please see attached…this is what I have in mind.

                  Gravity torus multiplier

                  Particles cause a second inferred gravity in the epicentre where 100% of the particles meet in the same direction

                  gravity at the epicentre pulls in the torus gravity. ( Just like 2 planets ) which have energy and momentum.

                  As the torus must reduce its dimension the particles must move faster which means they must reduce radius –

                  This makes the situation worse!

                  Hence a gravity multiplier! It want’s to increase forever ! ( like a diode ‘switch on’ infinite runaway current )

                  Particles if moving at ‘C’ must have a t= ( 0 ). If they move faster than light !!! then their time reference must become t = ( – 0 )!!

                  Which I think is impossible.

                  Therefore the entire system is in stress and equilibrium.

                  The entire system is experiencing a ‘TORQUE with TIME’.

                  If the particles could move faster their relative would become t = -0 . No problem – but they can’t! Even the LHC has proven that.

                  Hence everything in the background of the universe is against a background framework of T = 0 or 1. This is the ultimate limit

                  Not velocity.

                47. You made a comment which has been going around my mind. You stated that bacteria are found at depth in the Earth – and came to be there without assistance from tectonic transit? How far down recorded ? In my experience working in the Boulby mine the fresh cut salt even at a short distance from the surface of 5000ft was hot. I did not measure it but high 50C+? they can advise me of course. It was very hot in the road spaces and had to be cooled by a second shaft otherwise the workers would simply die in the heat and working naked were in not for a single covering with a thin pair of overalls.

                  We know anaerobic bacteria can survive without oxygen. We also know the composition of dna some of it excludes the requirement for oxygen. IF, the bacteria came to simply come into creation there – then surely they can do that on other mixed material planets at depth? Energy is available from the background, and materials from the salts from the surrounding – maybe even synthesis of oxygen from the salts ( not NaCl )? For example in the case of rice husks the composition of its ash is 90%+ pure amorphous silicon Si02– I know this from work in India. The rice getting the silicon from silicic acid from the water. That is why they refuse to rot easily – as with other grasses which have their lifestyle in water, and their seeds properly packaged resists water and may freely float away.

                  So maybe the criteria is local energy not oxygen to create life? – in which case all the planets could have it? Just not flying or walking around on the surface?

                48. As far as we currently know (This is a rather new area of biology) there are bacteria as deep as we have measured, so long as the rocks contain pores with water. They are all anaerobic- not requiring oxygen, and rather slow growing despite the heat (Some take between months and years to divide,.) They are not flourishing, but are surviving. Many seem to get a lot of their energy from chemicals, using oxidizing salts and acids like H2SO4 or Fe3+ to oxidize things like sulfides.

                  It seems very likely that they infiltrated the rock from the surface rather than life starting in rock itself. (Though some have argued otherwise.) This means that while many planets could sustain them, less could produce something that could evolve into them. This is why exactly where and how life formed is important; microbes today are not the fragile first life that formed, but already quite complex and robust.

                  Oxygen itself is not a criteria for life, the first life definitely lived without it and it is a toxic waste product to many things. As far as we know of what is needed for life is polymerizable carbon compounds (Amino acids, sugars) a solvent (e.g. water) some and some form of energy (e.g. oxidizable sulfide.) Life’s needs may be far broader or more specific, we’ll know when we find some not on earth.

                49. Wiki extract:

                  Within the framework of established knowledge of physics and cosmology, our universe could be one of many in a super-universe or multiverse. Linde (1990, 1994) has proposed that a background space-time “foam” empty of matter and radiation will experience local quantum fluctuations in curvature, forming many bubbles of false vacuum that individually inflate into mini-universes with random characteristics. Each universe within the multiverse can have a different set of constants and physical laws. Some might have life of a form different from ours; others might have no life at all or something even more complex or so different that we cannot even imagine it. Obviously we are in one of those universes with life. [9]

                  http://t0.gstatic.com/images?q=tbn:ANd9GcTpskyAvk4BDAiaxwzwvgFR0t8b3SFApM48NgM_CQPVCYKQzGwCzQ

                  This is how I visualize the above extract! And it is a natural shape – I think based around Felix Hausdorf , zero dimension and fractals? One could present an opinion that its shape is harmonious with the background framework behavior – dare I mention the entanglement. The shapes we see do not develop in isolation. They linked to something else?

                50. Not in a while and not for physics. (My specialty is carbohydrate chemistry, and I’m nothing special their either.)

                51. Hi Kudzu,

                  Initial searches realize 435 nuclear power stations ( a lot of recording !). 80,000 scientists involved with the ‘1’ LHC research machine vs me with 435 research machines ! Comically. I am sure if the phenomena existed – unexplained energy interruptions they would have noticed it. However, this is not always the case. Their minds would be focused on “why is our power down, and adjust the moderator!”? Vs “can anyone in the office advise – has the speed of light varied in the last minute”? Comically! I am pretty certain that such a search would be futile on the basis that they all exchange information and such a phenomena would have come to light many decades ago…. However, that recording is there to be examined should they have not considered it? There must be things relating to the operation of NPS which maybe even the specialists have not researched and any physical power eccentricity may be written off as a poor quality fuel?. Similar to radar return blips ( background noise & sea scatter etc ). It is ignored generally and nothing more than a minor irritation. ( dated radar systems ).

                52. Well any variation would turn up everywhere at once; which would be pretty noticeable, especially given that NPS do exchange staff and information quite regularly. And any variation in physical constants would have to be slight, or we might find our atoms suddenly not working. (Remember that physical constants affect *everything*)

                53. YES! Absolutely agreed.

                  But a – or + 0.00000000009 watt synchronous? Yes it would cause affects everywhere but how would we know except? Petrol in our cars combustion, storms as you correctly note, ‘Aurora borealis’ + southern equivalent etc. Movement of a proton in the LHC. But the change may be quite tiny as above and the time interval even smaller?

                54. Yes, any change that has escaped us would be tiny, trillionths of a percent or less, especially if it is constant change. (Happening over billions of years.)

                  Also, I forgot to mention that it seems we now have evidence of a photon being entangled with another photon that has already been measured and destroyed. http://www.livescience.com/19975-spooky-quantum-entanglement.html (That’s a brief introduction of course, the actual write-up is far more interesting, if opaque.)

                55. Thanks for the link..reading it.

                  The thrust of my enquiry relates to a very very short time interval change. Just a blip! And such a blib may be random or regular but how would we know? Do we have a station somewhere in the world which is constantly is measuring the speed?

                56. We have atomic clocks which are incredibly accurate and running a lot of the time. If you are talking about momentary changes that then reverse then these clocks should register them, though they might get bogged down in the background noise. (The slightest shake can make them go crazy, you’d be amazed at how insulated they have to be.)

                57. Yes good point atomic clocks. But I am not so sure if they would be so accurate. And as you say extremely delicate and sensitive – especially the temp.

                58. “(The ‘calm’ patch we are in where inflation has ended can only expand through new space at a rate of c, this is a disturbing feature of inflation.)”

                  Just returning to your above.

                  “Expand through new space”?

                  Intuitive example as you read this text light is transmitting information from your PC monitor to your eyes at value ‘C’. But in the one minute you have been reading this text the universe has actually expanded creating ‘ 1 minute linear value of ‘New Space’ – Between the monitor and your eyes – not ‘just’ expanding. This is another point relating to other realities of time and notional mechanism for the upper limit of ‘C’. Hence the LHC would only have the ability to exceed the value ‘C’ should it be located outside the Hubble Zone where New Space is being created faster. New Space and its plane has a time zero. But we are only really aware of the measurable linear time units light has taken to travel from your PC to your eyes.

                59. When I am talking about the expansion of our ‘calm patch’ I am speaking about how inflating space is being converted to non-inflating space. The problem is when inflation starts it causes space to expand faster than c, but when an area of space *stops* inflating it can only do so at light speed, so the non-inflating space can never ‘catch up’ with the part of the universe still inflating. Meaning that at the ‘edge of the universe’ there is always going to be inflation blowing the universe up to massive sizes. You might want to see this brief description of ‘eternal inflation’ http://en.wikipedia.org/wiki/Eternal_inflation Alan Guth has also written an excellent paper on this.

                  And I do not understand how you expect even a hypothetical LHC to exceed c, this is the universal speed limit, expanding space or not.

                60. “The problem is when inflation starts it causes space to expand faster than c,”

                  “And I do not understand how you expect even a hypothetical LHC to exceed c, this is the universal speed limit, expanding space or not.”

                  Sorry you have confused me on the above.

                61. Firstly, inflation makes space expand faster than light, so once it starts there is no way to stop it. (It can only be stopped at the speed of light it seems.)

                  Secondly, you state that “Hence the LHC would only have the ability to exceed the value ‘C’ should it be located outside the Hubble Zone where New Space is being created faster” There is no way to exceed c that we are aware of, expanding space or not.

                62. Thanks – yes. This is my point relating to the existence to the other dimensional plane ( dimension ut,x,y,z + t). My claim is that the velocity of light does not necessarily determine its own velocity. We are all locked into a specific way of trying to make sense of everything by our existing rules, and observational product very often has contradictions or lack of understanding and clear description. There was a post last night from NASA ref black energy and the ‘ether’! and unexplained occurrences between them.

                  This was a Victorian notion originating as far as I know from Newton. Then Michelson – morley set up their ingenious experiment to find out if it had motion. The term was completely discounted by Einstein but then he reinvented – hence the cosmological constant! Then your ref to De Sitter.

                  If for one minute we could think of such a thing would that help us to understand more? And try and see if it makes any sense of things – the above for instance?

                63. You will often see ‘old’ notions bought back, sometimes because we find out that there are loopholes that mean it’s still possible, other times it refers to something that is not at all the same, but is a bit like something old. (Such as transmutation, alchemy; we know you cannot turn lead into gold using magic potions, but you can in a particle accelerator or via radiation. We call this transmutation even though it is not the same as the old middle ages nonsense.)

                64. If instantaneous entanglement is real irrespective of distance only time (‘0’ time ). Does this not make you think of the horizon problem – and maybe support the concept of the 4th spatial dimension?

                65. Not particularly; entanglement is an interesting and complex field and I have not seen anything in the math that would suggest such things.

                  And we need to be careful what we mean when we say ‘dimension’; many times it is used when we really mean ‘field’;

                66. I hope you received my schematic of the LAC. If the particles are neutrons or whatever their descriptor/ability, they will have energy and acceleration. Moving at ‘normal c’ or Big C? they pass as individual particles a common internal point ( much like an atomic explosion mushroom torus ). The torus in this case may be moving as a system or not. The main point is if one has an exceptionally energetic torus and so intense that if forms a very tight diameter which it cannot adjust ( relax and become a larger diameter).

                  The frequency of particles arriving and exciting this common point is so high ( their individual contribution ) to the net gravity may be small. But because the arrival interval is infinitesimally short x the total number of particles passing in 1 unit of time the net contribution could be massive?

                  What do you think?

                67. I did not receive the schematic. However I think I grasp what you are saying.

                  At any point in space the average gravity depends on the average density of energy there. So at any point in your torus there would be a balance, particles arriving as quickly as they leave. The total gravity would depend on how ‘dense’ the particle beam was.

                  Also, each particle would not have a ‘small’ contribution because of moving so fast, but a *short* one. A black hole speeding past earth at very nearly light speed would still exert its full gravitational force on us, but if it passed quickly enough there wouldn’t be time for it to have much effect.

                68. Yes.

                  In the case of the Gravity Multiplier Concept, the particle beam could have a massive diameter, and with it massively dense as all the particles are confined into a small convergence. The relative velocities would be parity there. But as you correctly say they would pass through the convergence very quickly. However seeing there is a constant stream the net effect is that the gravity contribution is constant. Considering one particle in the epicentre for a tiny fraction of time – it would have to return a fraction of moment later.

                  Because this epicentre has a gravity its own right and being central to the torus – which has its own weaker gravity – the epicentre will pull on its diameter ( trying to shrink it ). As it pulls it into the centre – the particles orbiting will have to increase their orbital velocity – as the diameter of the torus wants to get smaller. ( think smoke rings )

                  The net effect of this will cause the density ( or visits ) to the epicentre to increase – so on and so forth infinitely …

                  I sent another schematic but have no idea how to transmit a jpeg here.

                  The unsatisfactory equilibrium is caused by the fact that the orbiting matter cannot move faster than light. Because in order to so then time will have to = ( -0 ). Which will mean time will have to run in minus figures or run backwards. So in my thinking this mechanism is experiencing a Time Torque. Therefore time is the master not velocity.

                  Regards the deep set bacteria, I would be surprised if they need to eat! On the basis that there is so much abundant background heat. And the fact that they don’t grow very fast is because ( maybe ) they are in a form of thermal stasis – opposite to frozen??

                69. returning to black holes as gravity engines. The particles in the torus are streaming thro the epicentre at C. This stream converges into the common centre and must densify ( if only momentarily ). 100% of all the particles participating in the construction of the torus pass through this epicentre. The particle beam would be massive, dense and at light speed. The relative time between the particles would be zero.They would create a gravity which in turn would cause the diameter of the torus to reduce. This in turn would cause the velocity of the particles to move faster than light – which they cannot do. So the entire system must be under constant stress. It wants to get smaller but cannot! The net effect ( possibly ) is to create an infinite ( vertical curve ) gravity. What do you think?

                70. Happy New Year Kudzu i hope you a good one. By the way can you imagine a universe without matter? If so can advise what you intuition tells you? regards

                71. And a happy new year to you also.

                  I can imagine a number of universes without matter, or even particles. You would need to be more specific as to other conditions. (Whether or not space was expanding and how say.) It is indeed a fun exercise to toy with these things and many physicists have, often in far greater detail (And with more mathematical rigor) than me.

                  What interests me is how often my intuition proves to be wrong, especially when it turns out my ideas have been tried by others. If anything I have learned to trust math far more than myself.

                72. Can you imagine just one universe without matter? I know you are going to say yes – in imagining that what remains in that space in your mind?

                  I have another intuition regards other universes – and have a image representation – also on you tube which I have named MANUBO. Funny word to use. But maybe we can have a look at this after you have considered the gravity engine?

                  Yes I take your point – but toys evolve with time – with the better understanding of materials and production techniques.

                73. I would very much like to see this other video of yours, i cannot seem to locate it on the channel which houses your gravity engine video.

                  A universe without matter could be many things; it could contain a bath of low energy radiation, it could be the end result of a universe like our own where all matter has decayed. (If the proton were usntable say.) Or it could be a universe without any particles whatsoever. It could even lack fields (Or consist of a field that has the same value everywhere.) As I said, you would need to be rather specific about what universe we are dealing with.

                74. Thanks for your kind interest – I will email a link later today….I think I may have cancelled the wix site for it. I put on the internet a couple of years ago…. And should if still around should searchable under ‘MANUBO’…. I will have a look…. Failing that send you construction files which I originally used.

                75. Dear Kudzu,

                  This was the manubo website I was building then removed it – now changing it. A year down the road I feel compelled to modify it (embarrassed and to correct the spelling mistakes everywhere and modify the core notions ).

                  The concept of Manubo illustrations attached. Essentially it proposes a Mother Universe comprised of 1 framework ( a ball for presentation purposes of ‘Ut’ ). The universes are produced much like a mother producing children – they accelerate away from the centre becoming flatter – until eventually the centre shears and becomes a very flat ribbon. This ribbon then continues it’s motion and direction around the Manubo body. Until eventually it is obliged to enter the opposite pole, where all its matter is returned to energy. The process continues indefinitely producing new and recovering old universes. ( The dynamics at the centre of this body are not far removed in my mind from my toy black hole concept. And in a way duplicate the same physical phenomena. But much bigger than the biggest black hole ever imaginable!

                  When existing as flat ribbons ( maybe as horizontal layers of contiguous branes ). At that point our star partners have long since disappeared and what we see in the night sky ( if not permanent night!) is other new stars and planet bodies. This realization in a way could provide the next evolution opportunity of man – assuming the milky way remains intact?

                  The next evolution would be very important – whatever man is and to remain a unique species. The reason being that the old universes are consumed at the opposite pole from which they were produced. In order to avoid this we would have to jump ship – onto a ( neighbour brane ) which was moving in the opposite direction in order to indefinitely eliminate entering a pole and complete extinction.

                  Manubo is not a unique body and could exist in their zillions ( like stars in our own HZ ) each one having zillions of universes ending up like layers of onion skin being produced and recycled endlessly.

                  By the way regards the my toy black hole concept you said how/what could cause the creation of a poloid? I don’t know is the short answer, but every bath tub drain in the southern hemisphere the water rotates in same direction due to the invisible influence of a relative huge body called the Earth. Similarly, the influence on matter finding itself in a potential black hole may well be under such a similar kind of dynamic – and one which we are unaware of?

                  These are the basic notions – thank you for showing interest in these ideas. regards

                76. I believe with your bathtub analogy you are referring to Coriolis force. Sadly this is not what causes water to swirl down a drain. This is the snopes page for the myth: http://www.snopes.com/science/coriolis.asp though it has been treated more thoroughly by physicists elsewhere on the internet. In a nutshell water swirls because it has some random motion that is amplified as it drains.

                  Something similar poses some severe problems for forming a structured black hole. The mass involved is already in the grip of a powerful dynamic, that of angular momentum. As the mass falls in it spins faster and faster. A neutron star may spin thousands of times a second, black holes much, much more. (Matter falling into the hole likewise makes it spin as it adds its angular momentum.)

                  So any model that poses an internal structure for black holes will have to account for this. In general a theory that posits something like this poloid of yours is not to be taken seriously unless there is a rigid mathematical basis for it, or experimental evidence.

                  I am now quite interested in this Manubo theory of yours, it seems to have promise. I am guessing that if we were to visualize the Manubo as a poloid in 3 dimensions then we could visualize the daughter universes as flat 2D ribbons emerging from (say) the top, curving over like lines of magnetic flux around a loop of wire eventually meetig at the bottom pole and being destroyed? It raises many, many (many) questions and I shall have to wait until a complete description is up to ask them.

                77. “I believe with your bathtub analogy you are referring to Coriolis force. Sadly this is not what causes water to swirl down a drain. This is the snopes page for the myth: http://www.snopes.com/science/coriolis.asp though it has been treated more thoroughly by physicists elsewhere on the internet. In a nutshell water swirls because it has some random motion that is amplified as it drains.”

                  Thanks for the link- I sort of regret referring to this as a play example – because now I am obliged to revisit it! And my mind is somewhat overtaken with the TBH. Actually on occasions I have tried and succeeded in causing it to rotate in its opposite sense. But then after a short time it reverts back to its preferred flow direction – this as you refer may be caused by the drain geometry. This would mean that I would have to create a test drain and test it around both hemispheres! Which I am most likely not going to do!

                  Yes I agree the TBH will not gain any kind of enthusiasm in the real world until such time as it could be demonstrated by some model. Which would include, select and predict particle behavior in this scenario.

                  However many speculative concepts are investigated on the basis of law of curiosity. The DoD invests millions in this kind of curiosity driven research – one such area of research which I was following was how to control an insect flight direction remotely by implanting ( connecting to and external Tx/Rx chip which was stuck on the insects head. A scarab beetle in this case. It also included the production of insects to increase their adult size. They succeeded in both. A. by keeping the insect generations in a high oxygen atmosphere simulating the carboniferous Oxy content – when insects where enormous ( larger than a man in some species ) and 2. By conducting invasive brain surgery of the insect to connect the electrodes!! The work done in California and I enjoyed a long private email discussion the Prof leading the research. The result was a 13% increase in insect size and good indication of flight control and flight initiation – but not very accurate – much like a cheap radio controlled aeroplane.

                  I prefer to think that black holes and universes are not that mysterious – just we have not used enough intuition. In the same way regards the dynamics involved with flower/insect mimic. If you research this there are some incredible examples ( mostly orchidae ). Thank you for showing interest in Manubo, we know that our universe is not symmetrical – and could say taking up a flat posture – Why? If that is true then there must be a force on it causing it to change/determine shape. Most speculatively of course maybe taking up its position at a pole? In preparation for the next phase and a big empty hole to appear!!

                  “(Matter falling into the hole likewise makes it spin as it adds its angular momentum.)”

                  That is very interesting – ( like a man pushing a child on a swing ? can I think of like that?

                78. The DoD have more money than sense in my opinion. When you investigate the possibility of psychics for several decades, despite constant failure you lose any credibility on that front.

                  I am not sure our universe is that asymmetric; I know that one of the big questions inflation ‘answered’ was why our universe was so flat, so featureless, so symmetric. Inflation acted to stretch out and smooth over the quantum irregularities that otherwise would have been very visible.

                  It is not quite like a man pushing a swing.You will want to check this video out: http://www.youtube.com/watch?v=UZlW1a63KZs As spinning collapses it has to spin faster. So it would be almost impossible for a black hole, neutron star or white dwarf not to be spinning incredibly fast when formed. (They slow down over time though.)

                79. http://www.youtube.com/watch?v=MHlAJ7vySC8

                  Thanks – yes a simple demonstration and quite powerful! This is Prof Leithwaite I used to follow his lectures back in the late 70’s – he had some fascinating ideas. I met him once at the Royal Institution of Great Britain where he gave a discourse of the same subject.

                  His style was captivatingly honest – unlike Kaku ( sorry Kaku!) ( then as now physics was a scary subject for the general public ) One of his ideas which you will remember was the train without wheels using magnetic levitation ( linear conductor motor ). He got somewhere with this and I think the Japanese invested in a track in Tokyo. This concept caused him a lot of TV popularity at the that time.

                  Back to TBH – revisiting this bit of physics – thanks for the link. Makes me think a bit deeper into the dynamics associated with the poloid – it could be experiencing another degree of rotation. 1. The particles in the ring and, 2 the entire poloid!

                  Question: 1. The individual particles are rotating at the speed of light as, 2. Converting at a common epicentre which determines that the diameter must continually reduce 3. The entire poloid system is also rotating at the speed of light? First I imagined it as a static mechanism but this may not the be case? What would you make of that?

                  So if the particles are responsible for the primary generation of gravity at the dense centre, the rotation of the entire system may cause it to be a gravity multiplier?

                  Prof laithwaite demonstrates ( the 40lbs mass ) is technically lighter. Of course it is not but from his arm muscles point of view it is and requires less joules of his energy to manipulate it.

                80. The behavior of the individual particles in your poloids would depend on exactly how they were behaving. If they were discrete particles with a well defined location this could be difficult, since they would be moving at light speed already, and rotation would attempt to increase or decrease their speed. If on the other hand they were in a similar situation to an electron in an atom, with a less well defined position then I think the whole poloid could be rotating.

                  The problem with the Professor’s example is that the rotation doesn’t affect gravity itself, but the balance of forces. A better analogy would be a motor, where a spinning magnet creates an electric flux. If this was the case for gravity it would likely involve a completely novel particle and\or aspect of gravity.

                81. I suppose a universe as ours would continue to be bombarded with CBR, strictly speaking I suppose we could not legally remove that as its origins are not of this universe. So what would the CBR be moving through where matter used to exist which created our local spacetime?

                  Question:

                  CBR: if you were to use Max Planks theory, could you use it to predetermine the energy of such a particle? By that I mean can you change any value in it to achieve such a result? E.g. Work the equation backwards and change values in it?

                82. Well now things get interesting.

                  In a universe that follows physics like ours the presence of the CBR means the presence of matter.

                  The CBR is indeed from our universe, roughly speaking being created not in the big bang as such, but being modified over time through interactions with matter. (Until the temperature of the universe was low enough for atoms to form.) It’s an interesting and complex subject.

                  In our universe any particle-antiparticle pair of sufficient energy can become any other particle-antiparticle pair. The big bang initially created a lot of particle-antiparticle pairs and the CBR is part of this (Photons being their own antiparticles.) This means that if the early universe was filled with just a pure mixture of one kind of particle and antiparticle, all the other possible kinds will be able to appear (roughly speaking.)

                  This could be avoided if the baby universe only produced particles (Not sure how this would work.) or if particle-antiparticle pairs couldn’t interconvert or if everything was finely balanced and all the matter and antimatter produced were in equal amounts and perfectly annihilated. (The fact that our universe had ‘extra’ matter was an interesting problem in physics.)

                83. Ref the ‘ toy black hole’ theory: You said you were unable to identify a candidate particle to participate in this system Of course likewise. However, just to kick of a stream of ideas how about this: If the TBH has a construction of particles with mass then presumably we have a short list to choose a candidate – and eliminate as unusable. In the alternative if the TBH is made up of massless particles we also have a list short list to choose from. If massless photons then the entire centre – also does not have mass just the concentration of photons producing a dense massless core?? But because they have the energy + momentum they produce the initial gravity so intense to initiate the entire device and cause a poloid? For example when a shell leaves a large gun barrel it usually produces a ring of energetic gun smoke. In the TBH and small step for the imagination, if in the final collapse of the supernova where there must be incredible dynamics being ever submitted to a decreasing diameter. In this confusion of behavior there must be a natural background ( initiator force ) of nature which is determining the final outcome, which is consistent otherwise we would not have so many black holes. So what happens in the last minutes before the black hole is switched on? That is an interesting muse! The last minute of shrinking chaos before it becomes stable and form one. The example of an initiator force in the bath tub is the rotation of the earth. So no matter how huge and powerful these things are, must remain subject to something external. Otherwise we would only have one black hole and a freak of nature?

                  Another step… to form a theoretical poloid one requires an aperture ( such as a gun barrel example ). Then this idea you may find interesting.. How about that a tiny aperture in this case is where 3D ceases to exist caused by concentration of dimensionless particles. Just 1 dimension ( which brings me back to ‘Ut’. ) If the 3D are stripped away then the degenerate material is entering a 1D sink where it may be rotating or stationary. We have a 1 dimensional hole – into which the energy, or photons may be moving – that perhaps could cause the initial concentration of energy into a focussed point. Once densified and focussed in this way causes the intense gravity. The chaotic competition around this aperture are then organised into a cyclical & laminar flow by its presence then becomes self sustaining like a jet engine. The power turbine is driving the compressors and will continue to accelerate until self destruction if not limited by the physics of fuel air combustion and bearings.

                84. One of the strengths of the ‘gravitational collapse model’ of black holes is that it dampens chaotic dynamics, much like how all massive objects in the universe are spheres. (A rock can be any shape, asteroids are roundish and planets even moreso.)

                  While the formation of a black hole (or neutron star, etc) is incredibly violent and energetic, the crushing gravity keeps at least the central mass all moving inwards at roughly the same rate, overwhelming other considerations.

                  In general he less symmetric something is, the harder it is to create. A sphere is the most symmetric 3D object and thus the easiest to create, all of its constituent particles simply have to attract each other. (Or it seeks to minimize the surface area-volume ratio or…) By contrast an egg shape can be formed by collapse only if one direction in space sees less collapse than the others (Which is not how physical laws work; meaning we’d need at least two processes to do this.)

                  So your poloid must form despite the fact that a collapsing sphere is simpler. (Indeed the formation of a black hole is very similar to water going down a drain. If you trace the path of a particle in both situations it traces out a spiral towards the center. However the drain has somewhere for the matter to go.)

                85. I used the word ‘charge’ nervously when attempting to explain the toy black hole as a hypothetical mechanism. If the particle stream is a photon or Gauge boson cocktail then the word charge may be appropriate? In as much as the photon is a carrier of electromagnetism. Could it be that when the particles are concentrate and densified forming its so called mass less core – that the charge in this zone becomes rather more than substantial + the creation of substantial gravity as the so called charge has to convert into another form in order to rid itself of it to maintain its equilibrium. Eg:

                  I recall once when I visited a paper mill. The web of paper as it was produced in the process of manufacture collected static charge picked up mainly from the drying phase over huge heating rollers. This charge was then conserved by the huge end roll which may have been more than a kilometer + of paper wound onto it. To walk in the vicinity of this roll was dangerous for various reasons. One of them being the fact that the roll would user a passerby as a route to earth producing a substantial spark which could jump a meter air gap. The roll was wound onto a metal roller with metal support. But for some reason it was not earthing the roll? And for some other reason airmail paper was the worse candidate for this. I suppose just being thinner and able to wind more paper length onto the roll. So it was acting like a huge capacitor.

                  Then as the gauge boson particle/s exit the core they return to the outer ring – only to return millionths of a second later with more charge which they have picked up in their transit outside the core. So the photons are pumping the core in a similar why as laser resonator is pumped. In this example the HV raises the outer electrons to a new orbit , then in returning to its historic orbit emits coherent light 1.6mu in the case of C02.

                  So the toy black hole could it not be converting this charge into gravity? So we have 2 dynamics 1. gravity produced in its true sense by the energy behavior of the photons in this mechanism and 2. Immense Electrostatic generator (van der graff style )?

                86. I could not find any references to “ Synchronicity of Suns” which presumably means there is non, not considered, or non published?

                  I have considered Hamiltonian briefly I am not sure how it can have any applicability to a universe of 2D time. But was alerted to his reference of infinitesimal places – which I am going to revisit. Also his equation negates time – which has also caught my attention.

                  In your earlier comment you requested some math to assist with the visualisation of a 1D TC framework. You will have to forgive me if the below is not relevant as I am not a professional mathematician – and I am presenting this off the cuff.

                  In the Parent Space which I propose it has 1 boundary and the internal framework has a time constant being ‘0’ or ‘1’ the 1D boundary moves outward and our locale is being stretched at 300,000kms creating new parental space in its framework ( not an ethereal wind ). So this intuition is promoting the idea that our parent space has 2D not 3D even though it could be a sphere ( hence liking to the mobius/Kleinwort phenomena ). What is happening to the boundary is of lesser consideration for the moment and only thinking of the notion and ability of a time constant.

                  2-dimensional Euclidean space (R2) where we examine Euclidean (x, y) and polar (r, θ) coordinates (which are undefined at the origin). Thus x = r cos θ and y = r sin θ and also r2 = x2 + y2, cos θ = x/(x2 + y2)1/2 and sin θ = y/(x2 + y2)1/2. Suppose we have a scalar field which is given by the constant function 1, and a vector field which attaches a vector in the r-direction with length 1 to each point. More precisely, they are given by the functions

                  s_{\mathrm{polar}}:(r, \theta) \mapsto 1, \quad v_{\mathrm{polar}}:(r, \theta) \mapsto (1, 0).

                  Let us convert these fields to Euclidean coordinates. The vector of length 1 in the r-direction has the x coordinate cos θ and the ycoordinate sin θ. Thus in Euclidean coordinates the same fields are described by the functions

                  s_{\mathrm{Euclidean}}:(x, y) \mapsto 1,

                  v_{\mathrm{Euclidean}}:(x, y) \mapsto (\cos \theta, \sin \theta) = \left(\frac{x}{\sqrt{{x^2 + y^2}}}, \frac{y}{\sqrt{x^2 + y^2}}\right).

                  We see that while the scalar field remains the same, the vector field now looks different.

                  IF, the prospect of a TC background is hypothetically ok – then current laws I suggest have to be considered differently – hence opening up a can of worms! We know STR has a problem in quantum physics – which to my mind is an indication that something is wrong or not correctly presented or understood – which infers we need more information about something to satisfy our lack of understanding?

                87. Where did you look for references? You’d be utterly amazed at how much there is out there, often tucked away in journals and online databases. Unless you have access you’re likely to miss some of the best stuff. All most people have access to is google and related search engines which will only help you access content that’s both online and available. As an example I did work on fluorosucroses to inhibit bacterial growth, good luck finding anything about that without knowing how to browse some pretty complex databases. (For one thing what you’re after may be called say, ‘non gravitational interaction of distant stars’ or something totally different. Finding previous work can be a real pain.)

                  You are correct in that STR (and GTR) and QM do not get along, this has long indicated to most physicists that we need a bigger theory that contains both of them. Your explanation is certainly something for me to think over, though I do not feel qualified to comment on it at present.

                88. Thanks – yes searching is absolute pain. I like your search phrase and will use it. I have sent an email to the AEA public office – to ask in what form and if available the NP daily outputs. The AEA are generally good at keeping records like the MoD. So I will just await their response. Being lazy hopefully the records will turn up?? However, it would be the most boring study? I hope it will be graphical report.

  69. Professor Strassler:
    Doesn’t the non zero vacuum expectation value (vev) of the Higgs field contribute to the vacuum energy? in this way isn’t there a connection between higgs vev and gravity?

    1. No. The non-zero expectation value does NOT contribute.

      What does contribute is the potential energy of the Higgs field. That is a *function* of the Higgs non-zero value v. But it is not v itself that contributes to the vacuum energy; it’s the potential energy as a function of v. In supersymmetric theories, you can have fields that have expectation values but contribute exactly zero potential energy.

      Moreover, if the Higgs field had zero expectation value, it STILL would have potential energy that contributes to the vacuum energy.

      So there is no logical relation between the two.

      Finally, there’s no connection between the potential energy from the Higgs and gravity — because gravity reacts to ALL sources of potential energy, and there’s nothing special about the Higgs. For instance, there is potential energy that is generated by the strong nuclear force [this is a complicated subject] and there is potential energy due to quantum fluctuations of all the fields of nature — and gravity reacts to all of them, as well as to the potential energy from the Higgs field, without regard to where they came from.

      1. This is a very interesting comment. I must admit that I did not made a clear distinction between vev and higgs potential energy as regarding its contribution to the vacuum energy.
        But I am less convinced by the last part of the argument. There is in my opinion a fundamental distinction with other sources of potential energy, the fact that the higgs field is said to permeate the entire universe, even the vacuum.
        So its contribution to the vacuum enrgy is certainly largely dominant compared to other (known) sources, and it is hard to see how it could be reconciled with the known value of the vaccum energy (or cosmological constant) which is much much lower (10^60 order of magnitudes ?, I don’t know exactly), and it is dominant for the total energy of the universe, hence its gravitionnal (or geometric) properties…

        1. Ref fusion:

          Just read this in a wiki lead. Am I mistaken or is this contradictory:

          At nucleus radii distances the attractive nuclear force is stronger than the repulsive electrostatic force. Therefore, the main technical difficulty for fusion is getting the nuclei close enough to fuse.

          If the nuclear force is stronger then why don’t we have fusion a daily regularity?

      2. Professor Strassler:
        Thanks for the reply, It helps clear some misconceptions. It is true that gravity reacts to all type of potential energy in the same way as with Higgs but its also interesting to note that (as “Alain” said) the contribution of Higgs field is dominant compared to other (known) sources. This leads to a problem because simple estimates show that the contribution of Higgs field to the vacuum energy density is many many orders of magnitude larger than the measured value of the vacuum energy density and we don’t know how to account for such a mismatch, there must be something (or some mechanism) to cancel this large mismatch.

  70. You wrote:
    “Now it is true that the W and Z particles, the quarks, the charged leptons and the neutrinos must get their mass from a Higgs field. It’s not possible for them to have masses any other way.”

    Have you explained somewhere else how we know which particles will get their mass from the Higgs field?

    Incidentally, the 125 GeV mass of the currently detected Higgs (which you refer to as curiously small) is fine for the Standard Model according to John Parsons from Columbia’s CERN team. But not for GUTs of course.
    As usual, this article was very informative and cleared up a lot of the haze! Thanks.

    1. I have NOT explained this yet. It is tricky mathematically (typically one learns this only well into a quantum field theory class.) But I think I can explain it for electrons; it’s an article under construction. So stay tuned.

      Curiously small refers to the hierarchy problem: I haven’t explained that properly yet either, but you can get a taste at http://profmattstrassler.com/articles-and-posts/particle-physics-basics/the-hierarchy-problem/

  71. Prof. Strassler,

    You say gravity is universal. But you also say that the Higgs and gravity are unrelated, Which seems logic to me if their sources are different. But then, how can you say that gravity is universal if it is not related to everything?

    1. ‘Universal’ does not mean ‘related to everything’ but rather ‘is general’ Gravity does not pull of positive particles different from negative ones, nor does it disappear on Saturdays or on the sun.

      Think of a universal adapter that allows you to make an appliance much more general, that is it will work the same way in many different situations.

      1. Universal refers to (see above) the notion of complete generality. The source of gravity is unknown as yet, but as stated above this complete generality does not apply to all things in universe. Why?

        1. Now that is a question that we as yet don’t know the answer to, at least completely.

          Some things simply can’t be universal, that’s not the way things work. Other things could in theory, but this wouldn’t last long. Others could be, but we’ve experimentally determined that they’re not. Possibly when (if) we can answer that question we will know a lot more about the universe than we do now.

  72. The charge inside an electron / positron is uniformly distributed. Can it be taken as evidence that it is fundamental and indivisible and so the smallest possible (smallest in the sense of energy rather than Compton radius) carrier of the isolated charge

    1. Electrons and positrons appear to be point particles, and measurements have been made that place upper limits on their size if they have one, but this is not ironclad proof that they are in fact no just really, really small composite particles.

      The charge in a proton appears to be uniformly distributed from a distance and only measurements at smaller scales show this to not be true. It is possible that our elementary particles aren’t and that most certainly would be interesting.

  73. So, in the beginning there would be bosons which can go from place to place and electrons which can go from place to place but no place for them to go to or from and nothing to do at those not extant places. Tto localize anything requires an observation which can change a state at some point in space time but there are no observers because everything that can inform is moving at vspeed of light and this has no position to localize in space and time
    So how does giving the Higgs boson mass energy change that in absence of giving rest mass to fermions so that there can be things related to each other?

  74. Sir, we only have an indication of a Higgs particle. My concern is – the specie they detected may be nothing more than a dipole fermion? Mimicking a higgs? I am nervous about the release of information now from LHC because their claim earlier that they exceeded the velocity of light – thanks to a $1 signal cable! My comfort is that the real information was released and admitted an error. At the current time the higgs remains hypothetical. Any any discussion relating to higgs is also hypothetical also to String theory – we are only discussing ideas about ideas. It can drive one mad. kind regards ewj

  75. Dear Pr Professor Strassler,

    As often your articles are very interesting and very challenging. Thanks for giving us all these deep views in High Energy Physics.

    You say that Higgs field has nothing to do with gravitation. But don’t you think that with a vev of about 246 GeV and so the enormous density energy of this field which permeates all space should dominates the energy content of the universe and then its geometry and gravitionnal behaviour ?

    Alain

  76. “… Einstein’s gravity … does pull on … dark matter …” — not according to Milgrom.
    Consider “Focus Point Supersymmetry Redux” by Feng, Matchev, and Sanford
    http://arxiv.org/pdf/1112.3021v3.pdf
    What are the essential differences between focus point supersymmetry (FP SUSY) and non-FP SUSY? What does FP SUSY (as opposed to non-FP SUSY) help to explain?

    1. While being skeptical is certainly “natural”, it is not the opposite of naive. Skepticism and naivety don’t go together well, but that does not make them opposites. You will see naive skeptics every day, doubting well-established concepts and not doubting flimsy theories.

    2. It is natural for all individuals to be naive as this is the easiest option; this can be readily apparent when an expert ventures out of their field. Skepticism is something acquired through experience and effort. Many experts lack it, but some develop a general skepticism that they can apply anywhere.

      I myself am not an expert in many things but I have learned for example that anything in a newspaper should be taken with a kilo of sodium chloride.

  77. Dear Matt,

    You made the statement (and I have seen this statement being made by other physicists):

    “So the mass of the earth, or the mass of the sun, would change, but not enormously, if there were no Higgs field”

    Numerically, if in the nucleus you would neglect the Higgs generated quark masses, this wouldn’t change the proton or neutron masses much.
    However, the kinetic energy of the quarks enters as mc2/sqrt(1-v2/c2) and here for v close to c the contribution can be much higher than the bare mc2.
    So my question is: can we really neglect the quark masses or is the impact on physics actually to big to make such a simple statement?

  78. Do we know for certain that the elementary particles with rest mass can not in actuality also consist of more fundamental mass-less particles contributing to their perceived mass just through their internal kinetic energy and binding energy, like protons and neutrons, and thus not need any Higgs field to explain their mass? How is this ruled out?

    1. Sure. The only evidence against it is that (A) the current theory works pretty well assuming they’re fundamental and (B) there has never been a hint of substructure in the currently believed to be fundamental particles. But there’s a precedent: hadrons were treated as fundamental particles until the 1970s.

  79. Dear Matt,
    When we go beyond GR and into quantum gravity theories and beyond into the realms of the cosmic grid, ( I am inspired by Frank Wilczek here), an exotic substance that permeates the emptiness is a must to give the properties to the vaccum as we perceive it through the standard model and the general relativity. Wilczek calls it the grid, or you can call it the Brahman from Hindu philosophy or ether (a bad choice due to historical association with the luminiferous ether) or what ever. While this material undoubtedly carries a lot of energy (vaccum energy), this energy also does not influence the energy-momentum tensor of gravity directly although gravity itself manifests through the space time created from this grid.
    Higgs excitation is a phenomenon that can arise in many quantum systems as the natural resonance frequency/excitation of the condensate (such as ultra cold rubidium atoms). Since Higgs field is a property arising from the nature of this exotic grid, and this grid gives rise to space time, understanding the origins of Higgs field is fundamental to understanding the grid itself and thereby space-time and gravity itself.
    What is your take on this

  80. Sorry about the double post, I’m having internet connection problems. But I have another concern: Correct me if I’m wrong but you seem to be contradicting yourself in the article. You state: “The Higgs field, which has spin 0, only interacts directly with elementary particles and fields that also participate in the electromagnetic and weak nuclear forces.” But then you go on to say that the Higgs field does not give mass to protons and neutrons. But these nucleons participate in the electromagnetic force as the quarks that make them up are electrically charged 1/3 e and 2/3 e.

    1. In regards to your first post, it’s entirely possible there are other Higgs particles out there, and if you check out the post on the known (apparently) elemental particles (If the Higgs field were zero) you’ll see that several are shown. (Reducing down to the one we see due to various interactions.) The thing is, even if there are different Higgs particles, they’re still *Higgs* particles and therefore must have certain characteristics. (There may be other types of ‘photon’ out there say, but if they have spin 1/2, they’re not photons anymore.)

      Thus even if we discover more Higgs particles, they’re not going to do anything different, just do what we know the Higgs does more complexly.

      On your second question, the Higgs field does give *some* mass to protons and neutrons, that is the rest mass of their quarks, about 1/100th of the mass of the proton or neutron. The bulk of the mass is due not to the Higgs but due to the binding energy holding those quarks together. Saying the Higgs gives no mass is wrong, but a very close approximation. Checking out the post on ‘What is a proton’ will shed some light on this.

      1. As an addendum, the binding energy is negative, but not all of it can be converted into energy that can escape the system. When a rock falls to the ground gravity turns potential energy into motion that is eventually dissipated as heat, (photons) this radiates away and the system loses energy (And thus mass.)

        But when three (hypothetical) free quarks meet, the binding energy is converted into particles (gluons, quarks, anti-quarks) that cannot escape. Three free quarks would be heavier than a proton (Most of their mass being potential energy.) but only some would be released on binding.

  81. Professor Strassler:

    Is it possible that if additional Higgs Bosons are found, beyond the 125 GeV conventional one that has been found, that they might provide mass to the gluons and also, possibly, dark matter?

  82. Professor Strassler:

    Is it possible that if additional Higgs Bosons are found, beyond the conventional 125 GeV one that has been found, that they might provide mass to the gluons and also, possibly, dark matter?

  83. So is it thought that we have already discovered all of the particles which are massive because of the The Higgs field?

    By “The” Higgs field I mean the one thought to be associated with the recently discovered H-boson.

    I ask this because you of your comment about dark matter getting its mass elsewhere.

    1. These issues are in the class of things we’re pretty sure of but not certain (there are loopholes) so with regard to this it would be best to let experiment tell us the answers, rather than impose our ideas on nature.

  84. Rezso,

    Good thought. That field is a Kahler modulus and can affect 4d physics, including (e.g.) its vacuum expectation value setting four-dimensional gauge couplings in D-brane constructions. Key: though it is a scalar field and its vacuum expectation value is important for low energy physics, the Higgs boson of the standard model does not arise in this way in string compactifications.

    Cheers,
    P

  85. (technical comment)

    Professor Strassler,

    correct me if I’m wrong, but I think that in string theory, there is a connection between gravity and scalar fields. If I start with the Einstein-Hilbert action in D dimensions and integrate over one of the compact extra dimensions, then the dimensional reduction will generate a massless scalar field. The vacuum expectation value of this scalar field controls the size of the extra dimension. So maybe there is a deep connection between the Higgs-boson and gravity.

    1. 1) In string theory, everything is related to everything. That’s probably true in any complete theory of the universe. So I’m sure that gravity is related to the Higgs boson in some sense, but it is certainly not the one that comes immediately to most people’s minds.

      2) The massless scalar field that you will obtain by integrating gravity over a compact extra dimension will not have the right quantum numbers to be a Higgs field — this is certainly not an option.

      3) Certainly people have considered more complex possibilities in which the Higgs field and gravity are part of a larger structure in higher dimensions. However, that does not change the fact that there is no connection between gravity’s pull on energy and the Higgs field’s role in giving mass to some of the particles of nature.

      4) In string theory, most particles (not the ones we know about, but others predicted by the theory) are strings in high vibrational states, or Kaluza-Klein partner particles, and these objects most certainly don’t get their mass from the Higgs field.

        1. Right — the scalar you mentioned will not participate in the weak interactions (any more than gravity does) and therefore when it has a non-zero value it cannot give mass to the W and Z particles (or to the electron, etc.)

    2. Uuuh Reszo,

      you are talking about quite dangerous things that are better not mentioned here, ts ts ts …
      And I dont know why this is, but I somehow feel pinged by this comment 😀

    1. The view of gravity as a ‘curvature’ of spacetime (itself a rather simplistic analogy of what relativity says gravity is.) is incompatible with QM’s view of it as being made of particles. We know therefore that is is incomplete (Much like Newton’s theories are incomplete.) but it is still useful.

      This is why we use a lot of scientific concepts, such as say gravity relying only on mass, They’re not accurate, but they’re useful analogies or approximations. Even if we do come up with a QM description of gravity it’s likely that you’ll still see the old ‘curved space’ description used because it’s simpler to explain to the layman.

  86. Thanks Matt. This was a really important clarification.

    Another question: Nobody is confident about how to reconcile gravity with quantum theory, yet everybody seems certain that gravity will be explained in terms of ‘spin-2 particles’. Where does this certainty come from?

    1. The only things you need in order to know that gravity will exhibit (not “be-explained-in-terms-of”, for this explains nothing) spin-two gravitons is

      1) the world is quantum mechanical
      2) gravitational waves in Einstein’s theory are those of a spin-two classical field

      As long as these two things are true, it’s guaranteed; for in a quantum mechanical world, all waves are made from quanta (which is what particles are — ripples of minimal height), quanta all must have integer or half-integer spin, and the properties of the waves follow from the spin of their quanta. So you can infer the quantum numbers of quanta from the properties of the corresponding waves, without knowing anything else. You can infer from Maxwell’s equations for electric and magnetic fields that if there is quantum mechanics in the world, electromagnetic waves will be made from spin-one quanta (namely photons.)

      To make a full theory of quantum gravity, however, you need a vastly more complete understanding of how all the fields of nature interact with each other. This is where all the mathematical challenges arise. It is the same for understanding what photons do when they interact with matter; you need much more than Maxwell’s equations for that! Quantum electrodynamics (QED) was quite challenging to develop, long after people knew about photons. And quantum gravity will be that much more difficult — but spin-two gravitons will be a part of it, unless quantum mechanics has to be completely revised even for ordinary gravitational waves.

      1. If course one does not exclude the continue look for foundational attributes to describing reality just that one does it right. Thanks

  87. Can we then go further and say that general relativity has nothing essential to do with ‘particles’ at all; much less with the value of mass we assign to those particles?

    That is, GR is concerned with the distribution of energy-momentum throughout spacetime, and its connection with the geometrical structure of the latter. That we can coherently ‘divvy up’ the energy-momentum into ‘particles’ (elementary or otherwise; massive or otherwise) is of no fundamental concern to this theory.

    1. That is certainly true of Einstein’s theory of gravity, which makes no reference to particles at all.

      It is unlikely to be true of the full quantum mechanical theory of gravity to be found in nature, however, where modifications of Einstein’s theory may well depend in detail on the quantum nature of the world.

    2. When G.R. was published, it had only recently been established that atoms have a nucleus; protons and neutrons hadn’t been discovered, and Heisenberg and Dirac were schoolboys. That only makes it all the more remarkable, to my mind 🙂

    1. Are you talking about what we know? or what various people speculate about?

      Because in the theories where gravity is an emergent phenomena (and there are many of them) the answers are as diverse as the theories themselves.

  88. Well , but what is the causal physical mechanism , i mean what is the cause of the word ( determined ) in 2nd line , i am beyond equations , i talk reality as i know that there are some other equations that say otherwise.

  89. Now you say G.R.gravity is all about energy and momentum , then IF spacetime is really bending under E and P then what mechanism affect spacetime forcing it to bend if E and P are in some locality ?

    1. Einstein’s Equation (also written down by Hilbert) essentially says:

      The properties of space and time are determined by the quantity of energy and momentum to be found there.

      More specifically there is a Riemann tensor that is determined by the properties of space and time
      There is an energy-momentum tensor that is determined by the presence of energy and momentum

      And a particular quantity built from the Riemann tensor is required by the Einstein-Hilbert equation to be equal to the energy momentum tensor, times a constant.

      Gravitational fields are simply manifestations of the shape of space and time; so it is true that gravitational fields are determined by the presence of energy and momentum. Masses of objects appear nowhere in the equation.

      Like all things, the fully correct statement is a bit more complicated (since gravitational fields also carry energy and momentum) but this is the basic idea.

      1. “Gravitational fields are simply manifestations of the shape of space and time; so it is true that gravitational fields are determined by the presence of energy and momentum. Masses of objects appear nowhere in the equation.” …

        “So the proton’s and neutron’s masses do not come predominantly from the Higgs field. [Experts: There is a subtlety here, having to do with how the Higgs field affects the confinement scale; but even when it is accounted for, the statement remains essentially true.]” …

        “You can make a black hole entirely out of photons, in principle.” …

        Hello Prof.

        I am sure you have seen this simulation and maybe even better one, (which I would love to get my hands on if you have any links?). http://upload.wikimedia.org/wikipedia/commons/0/03/Black_hole_lensing_web.gif

        It is a galaxy passing through a black hole. And it clearly illustrates you first comment above as to how gravitational fields shape space and time.

        Your third comment, of how a black hole can be constructed entirely out photons, is what I have been trying to convey, unsuccessfully (due to my poor use of the English language) here for some time. “Spinors”, “trapped energy spheres in very tiny spaces”, once I even called them Fermi spheres which you quickly corrected me, thank you.

        If you go back to that simulation and reduce it down to at or below Planck’s scale, and assume the galaxy to be a plane wave of photons and the black hole, well being a very, very, very, very tiny black hole (also made from “entirely out of photons”). Look at the shape of the light going around the black hole, … from flat plane to a precise circle when it coincides with the black hole, i.e. Fourier transform, the full spectrum I may add.

        My point, you can take you first comment and cross out the word gravitational and just say, “fields are simply manifestations of the shape of space and time;” in general, universal.

        The tough part is your second comment about confinement. I once asked if, is understanding confinement the “Holy Grail” of final getting a unification theory postulated? And you replied, no. So allow me to rephrase my question … If one can figure out how these potential “spinors”, “micro black holes”, “trapped photons”, (graviton?) get confined with a stability (life) of potentially infinite time could it not explain the gravitational field and the evolution of the other fields (forces) derived directly from this phenomena illustrated in the simulation?

  90. Why wouldn’t there be any atoms without the Higgs field? My first guess is that electrons would be massless, hence move at the speed of light, hence not sit around in orbitals. What’s the real reason?

  91. http://profmattstrassler.com/articles-and-posts/particle-physics-basics/mass-energy-matter-etc/mass-and-energy/

    In that article you called the energy locked away in objects mass ” mass energy”, is the “mass energy” that comes from the higgs mechanism the same “mass energy” as a system of two photons going different directions(they have 0 momentum frame, hence a rest mass)?

    Basically what I’m asking is there a fundamental “mass energy” from the higgs mechanism and an emergent “mass energy” from the interactions(potential) and motions(kinetic) of the the systems constituents. Are they the same thing in terms of inertia and gravity? Is it good to differentiate between the two?

    1. They’re the same thing. The difference is whether the system of objects whose mass you’re thinking about moves as a unit and can really be called an “object” in and of itself. The two photons from, say, a Higgs particle’s decay form a system whose mass we can talk about, as I did in the post you mentioned. But of course they’re going to go flying apart across the universe and they will have very different futures. By contrast, when we talk about the mass of an atomic nucleus we are talking about the mass of a system that might hold together, as a unit, for seconds, centuries, or billions of years. So a system of two photons is not really what we think of as an object, while an atomic nucleus really is. The same is true of an electron; it’s an object of its own. Even though a Higgs particle or a Z particle exists for a very short time in human terms, it lives thousands of times longer than its vibrational frequency (ν = m c2/h) which I like to call its “heartbeat”, so it too forms an object whose mass can be discussed just like that of a nucleus.

      Gravity will pull on these forms of energy in the same way **as long as they are in the same place**. When a Higgs particle decays to two photons, there is no immediate change in the gravitational effects, because the energy and momentum stored in the vicinity of the Higgs particle have not changed. Once the two photons begin to separate by great distances, however, then the gravitational effects will change, because the location of the energy and momentum has changed.

      1. Sir, I understand that Einstein when referring to mass was to advise students to totally ignore the implication into physics invariant mass! This statement steeply troubles me. I understand that the invariant mass is a subject in itself when considering how it interacts with particular reference to gravity implications. Can we consider for one moment Black Holes? We know it is impossible to exceed the value of ‘C’. From earlier para submitted on your fine site you may have picked up that my notion is that light does not determine its own velocity but is determined by the mechanism of expansion of the local universe which just so happens to have this constant opening arbitrary velocity but as a separate unity/dimension? IF, this for one moment could be true then the spin of a black hole may indeed be experiencing a torque against it !! Let us further imagine for one moment that this torque indeed limits the max radial velocity of such a ( rest mass i.e. place it on a static scales ) heavy object. But because this body has a high rest mass and may indeed be rotating at very high sub ‘C’ values a few interesting things may occur: 1. Because of its spin value ( high angular momentum ) will actually cause its mass to increase – we both know that is correct. I cannot calculate that as i don’t have time or experience with doing such calculations – but in principal it should be pretty straight forward – invent a huge static mass and multiply by its angular momentum – something like that? So we now have 2 clear conditions a. the invariant mass and b. its actual – virtual mass which is variant. 2. Should a very heavy object such as this could it indeed be creating a virtual black hole as a deformable ( shrinking body )? By that I mean its invariant mass is inadequate to cause a very deep gravity well sufficient to pull in all the stars in a galaxy. But because of its angular momentum it is actually providing a much greater gravity which is? 3. IF, such a torque does exist then as it rotates the invariant mass will necessitate it to get heavier. This increase in mass/gravity will undoubtedly cause it to become smaller as the variant gravity will simply cause it to shrink in diameter. IF, this is the case then the angular momentum will also have to increase – which means greater radial velocity. 4. IF, my notions have any basis and the universe does indeed exist as its own entity as a zero dimensional framework creating New Space at the rate of 300,000kms then our black hole has very strict physical limits. Which means it cannot get any smaller as its momentum will simply be too high set against the torque affect of the Primary dimension which i refer to as ‘Ut’ ( universal time ). 5. So we have a situation where the invariant mass in this instance is obliged to increase but cannot. So the next thing which must happen is for the new mass which this black hole is ingesting must be converted back into energy to simply get rid of it ! Which we observe as gamma radiation from the poles – where the angular momentum is much lower than its equator. It is a mass to energy converter – nothing more complicated than that. Likewise a proton must increase its mass as it approaches the value of ‘C’ because we are dumping a lot of energy into it to increase its momentum – but cannot. – Our black hole must convert the mass its eating to energy – this is its only option as it simply cannot get any heavier? I look forward to your reply kind regards ewj

Leave a Reply

Search

Buy The Book

A decay of a Higgs boson, as reconstructed by the CMS experiment at the LHC

Related

Recently, the first completed search for what is nowadays known as SUEP — a Soft-Unclustered-Energy Pattern, in which large numbers of low-energy particles explode outward

POSTED BY Matt Strassler

POSTED BY Matt Strassler

ON 03/15/2024

On April 8th, 2024, a small strip of North America will witness a total solar eclipse. Total solar eclipses are amazing, life-changing experiences; I hope

POSTED BY Matt Strassler

POSTED BY Matt Strassler

ON 03/14/2024